You are on page 1of 82

2017-18 100 &

op kers
Class 12 T
By E ran culty
-JE Fa r
IIT enior emie .
S fP r es
o titut
Ins

MATHEMATICS
FOR JEE MAIN & ADVANCED
SECOND
EDITION

Exhaustive Theory
(Now Revised)

Formula Sheet
9000+ Problems
based on latest JEE pattern

2500 + 1000 (New) Problems


of previous 35 years of
AIEEE (JEE Main) and IIT-JEE (JEE Adv)

5000+Illustrations and Solved Examples


Detailed Solutions
of all problems available

Topic Covered Plancess Concepts


Tips & Tricks, Facts, Notes, Misconceptions,
Determinants Key Take Aways, Problem Solving Tactics

PlancEssential
Questions recommended for revision
17. DETERMINANTS

1. INTRODUCTION
Development of determinants took place when mathematicians were trying to solve a system of simultaneous
linear equations.

a1 x + b1 y =
c1  b2c1 − b1c2 a1c2 − a2c1
E.g.  ⇒ x= and y =
a2 x + b2 y =
c2  a1b2 − a2b1 a1b2 − a2b1

a1 b1
Mathematicians defined the symbol as a determinant of order 2 and the four numbers arranged in row
a2 b2
and column were called its elements. Its value was taken as (a1b2 − a2b1 ) which is the same as the denominator.
a b
If we write the coefficients of the equations in the following form then such an arrangement is called a
c d
determinant. In a determinant, horizontal lines are known as rows and vertical lines are known as columns. The
shape of every determinant is a square. If a determinant is of order n then it contains n rows and n columns.

a1 b1 c1
a1 b1
E.g. , a2 b2 c2 are determinants of second and third order respectively.
a2 b2
a3 b3 c3

Note: (i) No. of elements in a determinant of order n are n2. (ii) A determinant of order 1 is the number itself.

1.1 Evaluation of the Determinant using SARRUS Diagram

 a11 a12 a13 


 
If A = a21 a22 a23  is a square matrix of order 3, the below diagram is a Sarrus Diagram obtained by adjoining
a a32 a33 
 31
the first two columns on the right and draw dark and dotted lines as shown.

The value of the determinant is (a11a22a33 + a12a23a31 + a13a21a32 ) − (a13a22a31 + a11a23a32 + a12a21a33 ) .
1 7 . 2 | Determinants

a11 a12 a13 a11 a12

a21 a22 a23 a21 a22

a31 a32 a33 a31 a32

Figure 17.1

3 2 5
Illustration 1: Expand 9 −1 4 by Sarrus rules.  (JEE MAIN)
2 3 −5

Sol: By using Sarrus rule i.e. ∆ = (a11a22a33 + a12a23a31 + a13a21a32 ) − (a13a22a31 + a11a23a32 + a12a21a33 ) we can expand
the given determinant.

3 2 5 3 2

-1 4 9 -1

2 3 -5 2 3

Figure 17.2

3 2 5
Here, =
∆ 9 −1 4 ⇒ ∆= 15 − 36 + 90 + 16 + 135 + 10= 230
2 3 −5

x2 − x + 1 x − 1
Illustration 2: Evaluate the determinant :  (JEE MAIN)
x +1 x +1

Sol: By using determinant expansion formula we can get the result.

x2 − x + 1 x − 1
we have, = (x2 − x + 1)(x + 1) − (x + 1)(x − 1) = x3 + x2 − x2 − x + x + 1 − x2 + 1 = x3 − x2 + 2
x +1 x +1

2. COFACTOR AND MINOR OF AN ELEMENT


Minor: Minor of an element is defined as the determinant obtained by deleting the row and column in which that
a11 a12 a13
a a23
element lies. e.g. in the determinant D = a21 a22 a23 , minor of a12 is denoted as M12 = 21 and so on.
a31 a33
a31 a32 a33

Cofactor: Cofactor of an element ai j is related to its minor as Ci j = ( −1)i + j Mi j , where ‘i’ denotes the ith row and
‘j’ denotes the jth column to which the element ai j belongs.
Now we define the value of the determinant of order three in terms of ‘Minor’ and ‘Cofactor’ as

D = a11M11 − a12M12 + a13M13 or D = a11C11 + a12C 12 + a13C13


M a them a ti cs | 17.3

Note:
(a) A determinant of order 3 will have 9 minors and each minor will be a determinant of order 2 and a determinant
of order 4 will have 16 minors and each minor will be determinant of order 3.
(b) a11C21 + a12C22 + a13C23 =
0 , i.e. cofactor multiplied to different row/column elements results in zero value.

Row and Column Operations


(a) Ri ↔ Rj or Ci ↔ Cj , when i ≠ j; This notation is used when we interchange ith row (or column) and jth row (or
column).
(b) Ri ↔ Ci ; This converts the row into the corresponding column.
(c) Ri → Rki or Ci → kCi ; k ∈ R; This represents multiplication of ith row (or column) by k.
(d) Ri → Rik + Rj or Ci → Cik + Cj ; (i ≠ j); This symbol is used to multiply ith row (or column) by k and adding the
jth row (or column) to it.
2 −3 5
Illustration 3: Find the cofactor of a12 in the following 6 0 4  (JEE MAIN)
1 5 −7

Sol: In this problem we have to find the cofactor of a12, therefore eliminate all the elements of the first row and
the second column and by obtaining the determinant of remaining elements we can calculate the cofactor of a12.
Here a12 = Element of first row and second column = –3

2 −35

6 4
M12 = Minor of a12 ( −3) =6 0 4 = = 6(–7) − 4(1) = −42 − 4 = –46.
1 −7

1 5 −7

( −1)1+ 2 ( −46) =
Cofactor of ( −3) = −( −46) =
46

Illustration 4: Write the minors and cofactors of the elements of the following determinants:

2 −4 a c
(i) (ii)  (JEE MAIN)
0 3 b d

Sol: By eliminating row and column of an element, the remaining is the minor of the element.
2 −4
2 −4
(i) ; M11 = Minor of element (2) =  ( −1)1+1 M11 =
= 3 ; Cofactor of (2) = +3
0 3
0 3

2 −4
M12 = Minor of element (–4) =  = 0 ; ( 1)1+ 2 M12 =−
Cofactor of ( −4) =− ( 1)0 =0
0 3

2 −4
M21 = Minor of element (0) =  = −4 ; ( 1)2+1 M21 =−
Cofactor of (0) =− ( 1)( −4) =4
0 3

2 −4
M22 = Minor of element (3) =  = 2 ; ( −1)2+ 2 M22 =
Cofactor of (3) = +2
0 3
1 7 . 4 | Determinants

a c
a c
(ii) ; M11 = Minor of element (a) =  = d; ( −1)1+1 M11 =
Cofactor of (a) = ( −1)2 d =
d
b d
b d
a c
M12 = Minor of element (c) =  = b ; ( −1)1+ 2 M12 =
Cofactor of (c) = ( −1)3 b =
−b
b d

a c
M21 = Minor of element
= (b) = c ; ( −1)2+1 M21 =
Cofactor of (b) = ( −1)3 c =
−c
b d

a c
M22 = Minor of element
= (d) =  a ; ( −1)2+ 2 M22 =
Cofactor of (d) = ( −1)4 a =
a
b d
2 −2 3
Illustration 5: Find the minor and cofactor of each element of the determinant 1 4 5 .  (JEE ADVANCED)
2 1 −3

Sol: By eliminating the row and column of an element, the determinant of remaining elements is the minor of the
element. i.e. Mi× j and by using formula ( −1)i+ j Mi× j we will get the cofactor of the element.

4 5 1 5 1 4
The minors are M11 = = −17 , M12 = = −13 , M13 = = −7
1 −3 2 −3 2 1

−2 3 2 3 2 −2

= M21 = 3 , M22 = = −12 , M23 = = −6
1 −3 2 −3 2 1

−2 3 2 3 2 −2
M31 = = −22 , M32
= = 7 , =
M33 = 10
4 5 1 5 1 4

The cofactors are:

( −1)1+1 M11 =
A11 = M11 = ( −1)1+ 2 M12 =
−17 , A12 = −M12 = ( −1)1+3 M13 =
13 , A13 = M13 =
−7

( −1)2+1 M21 =
A21 = −M21 = ( −1)2+ 2 M22 =
−3 , A22 = M22 = ( −1)2+3 M23 =
−12 , A23 = −M23 =
6

( −1)3+1 M31 =
A31 = M31 = ( −1)3+ 2 M32 =
−22 , A32 = −M32 = ( −1)3+3 M33 =
−7 , A33 = M33 =
10

3. PROPERTIES OF DETERMINANTS
Determinants have some properties that are useful as they permit us to generate the same results with different
and simpler configurations of entries (elements).
(a) Reflection Property: The determinant remains unaltered if its rows are changed into columns and the
columns into rows.
(b) All-zero Property: If all the elements of a row (or column) are zero, then the determinant is zero.
(c) Proportionality (Repetition) Property: If the all elements of a row (or column) are proportional (identical)
to the elements of some other row (or column), then the determinant is zero.
(d) Switching Property: The interchange of any two rows (or columns) of the determinant changes its sign.
M a them a ti cs | 17.5

(e) Scalar Multiple Property: If all the elements of a row (or column) of a determinant are multiplied by a non-
zero constant, then the determinant gets multiplied by the same constant.

a1 + b1 c1 d1 a1 c1 d1 b1 c1 d1
(f) Sum Property: a2 + b2 c2 d2 = a2 c2 d2 + b2 c2 d2
a3 + b3 c3 d3 a3 c3 d3 b3 c3 d3

a1 b1 c1 a1 + αb1 + βc1 b1 c1
(g) Property of Invariance: a2 b2 c2= a2 + αb2 + βc2 b2 c2
a3 b3 c3 a3 + αb3 + βc3 b3 c3

That is, a determinant remains unaltered under an operation of the form Ci → Ci + αC j + βCk , where j, k ≠ i ,
or an operation of the form R i → R i + αR j + βRk , where j, k ≠ i

(h) Factor Property: If a determinant ∆ becomes zero when we put x = α , then (x − α ) is a factor of ∆ .

(i) Triangle Property: If all the elements of a determinant above or below the main diagonal consist of zeros,
then the determinant is equal to the product of diagonal elements. That is,

a1 a2 a3 a1 0 0
0 b2 b3
= a=
2 b2 0 a1b2c3
0 0 c3 a3 b3 c3

a11 a12 a13 C11 C12 C13
( j) Determinant of cofactor matrix: ∆ = a21 a22 a23 then ∆1 =C21 C22 C23 =∆2
a31 a32 a33 C31 C32 C33

where Cij denotes the cofactor of the element aij in ∆ .

PLANCESS CONCEPTS

By interchanging two rows (or columns), the value of the determinant differs by a –ve sign.
If ∆ ′ is the determinant formed by replacing the elements of a determinant ∆ by their corresponding
cofactors, then if ∆ =0 , then ∆1 =0 ,else ∆ ′ =∆n−1 , where n is the order of the determinant.
Vaibhav Gupta (JEE 2009 AIR 54)

a b c
Illustration 6: Using properties of determinants, prove that b c a = (a + b + c)(ab + bc + ca − a2 − b2 − c2 )
c a b (JEE MAIN)


Sol: By using invariance and scalar multiple property of determinant we can prove the given problem.

a b c a+b+c b c
∆= b c a = b + c + a c a [Operating C1 → C1 + C2 + C3 ]
c a b c+a+b a b

1 b c 1 b c
= (a + b + c) 1 c a = (a + b + c) 0 c − b a − c [Operating (R 2 → R 2 − R1 and R 3 → R 3 − R1 )]
1 a b 0 a−b b −c

= (a + b + c){(c − b)(b − c) − (a − b)(a − c)} = (a + b + c)(bc − b2 − c2 + bc − (a2 − ab − ac + bc)) = (a + b + c)(ab + bc + ca − a2 − b2 − c2 )


1 7 . 6 | Determinants

−α2 βα γα
2
Illustration 7: Prove the following identity αβ −β γβ = 4α2β2 γ 2  (JEE MAIN)
2
αγ βγ −γ

Sol: Take α , β, γ common from the L.H.S. and then by using scalar multiple property and invariance property of
determinant we can prove the given problem.
−α2 βα γα
2
∆= αβ −β γβ
αγ βγ −γ 2
−α α α
Taking α , β, γ common from C1 , C2 , C3 respectively ∆ = αβγ β −β β
γ γ −γ

−1 1 1
2 2 2
Now taking α , β, γ common from R1 ,R 2 ,R 3 respectively ∆ = α β γ 1 −1 1
1 1 −1

−1 1 1
2 2 2
Now applying R 2 → R 2 + R1 and R 3 → R 3 + R1 we have ∆ = α β γ 0 0 2
0 2 0
0 2
Now expanding along C1 , ∆ = α2 × β2 ( −1) × γ 2 ( −1) = α2β2 ( −1)γ 2 (0 − 4) = 4α2β2 γ 2
2 0
Hence proved.

α β γ β µ φ
Illustration 8: Show that θ φ ψ = α λ θ  (JEE ADVANCED)
λ µ ν γ ν ψ

Sol: Interchange the rows and columns across the diagonal using reflection property and then using the switching
property of determinant we can obtain the required result.

α β γ α θ λ
L.H.S. = θ φ ψ = β φ µ (Interchanging rows and columns across the diagonal)
λ µ ν γ ψ ν

α λ θ β µ φ β µ φ
2
( 1) α λ θ =α λ θ = R.H.S.
=( −1) β µ φ =−
γ ν ψ γ ν ψ γ ν ψ

a a2 1 + a3
Illustration 9: If a, b, c are all different and if b b2 1 + b3 =
0 , prove that abc = –1.  (JEE ADVANCED)
2 3
c c 1+c

Sol: Split the given determinant using sum property. Then by using scalar multiple, switching and invariance
properties of determinants, we can prove the given equation.

a a2 1 + a3 a a2 1 a a2 a3 a a2 1 1 a a2
D = b b2 1 + b3 = b b2 1 + b b2 b3 = b b2 1 + abc 1 b b2
c c2 1 + c3 c c2 1 c c2 c3 c c2 1 1 c c2
M a them a ti cs | 17.7

1 a2 a 1 a a2
[C1 ↔ C3 in 1st det.]
= ( −1)1 1 b2 b + abc 1 b b2
1 c2 c 1 c c2

1 a a2 1 a a2
= ( −1)2 1 b b2 + abc 1 b b2 [C2 ↔ C3 in 1st det.]
1 c c2 1 c c2

1 a a2 1 a a2 1 a a2
= 1 b b2 + abc 1 b b2 = (1 + abc) 1 b b2
1 c c2 1 c c2 1 c c2

1 a a2
= (1 + abc) 0 b − a b2 − a2 [R 2 → R 2 − R1 and R3 → R3 − R1 ]
0 c − a c2 − a2

b − a b2 − a2 1 b+a
= (1 + abc) (expanding along 1st row) = (1 + abc) (b – a) (c – a)
c−a c −a 2 2 1 c+a

= (1 + abc) (b – c) (c – a) (c + a – b – a) = (1 + abc) (b – a) (c – a) (c – b)

⇒ D = (1 + abc) (a – b) ( b – c) (c – a); But given D = 0

⇒ (1 + abc) (a – b) ( b – c) (c – a) = 0; ∴ (1 + abc) = 0

[since a, b, c are different a ≠ b, b ≠ c, c ≠ a ]; Hence, abc = – 1

a + b + 2c a b
Illustration 10: Prove that c b + c + 2a b = 2(a + b + c)3  (JEE ADVANCED)
c a c + a + 2b

Sol: Simply by using switching and scalar multiple property we can expand the L.H.S.

a + b + 2c a b
Given determinant = c b + c + 2a b
c a c + a + 2b

Applying C1 → C1 + (C2 + C3 ) , we obtain

2(a + b + c) a b 1 a b
2(a + b + c) b + c + 2a b = 2(a + b + c) 1 b + c + 2a b
2(a + b + c) a c + a + 2b 1 a c + a + 2b

R1 → R 2 − R1 and R3 → R3 − R1 given

1 a b
2(a + b + c) 0 b + c + a 0 = 2(a + b + c) ⋅ 1{(b + c + a)(c + a + b) − (0 × 0)} = 2(a + b + c)3
0 0 c+a+b

Hence proved.
1 7 . 8 | Determinants

a2 + 1 ab ac
2
Illustration 11: Prove that ab b +1 bc =1 + a2 + b2 + c2  (JEE ADVANCED)
2
ac bc c +1

a2 + 1 ab ac
2
Sol: Expand the determinant ab b +1 bc by using scalar multiple and invariance property.
2
ac bc c +1

a2 + 1 ab ac
2
L.H.S.= ab b +1 bc ; Multiplying C1 , C2 , C3 by a, b, c respectively
2
ac bc c +1

a(a2 + 1) ab2 ac2


1
= a2b b(b2 + 1) bc2 ; Now taking a, b, c common from R1 , R 2 , R3 respectively
abc 2 2 2
ac b c c(c + 1)

a2 + 1 b2 c2 1 + a2 + b2 + c2 b2 c2
abc
= a2 b2 + 1 c2 = 1 + a2 + b2 + c2 b2 + 1 c2 [C1 → C1 + C2 + C3 ]
abc 2 2 2 2 2 2 2 2
a b c +1 1+a +b +c b c +1

1 b2 c2 1 b2 c2
= (1 + a2 + b2 + c2 ) 1 b2 + 1 c2 = (1 + a2 + b2 + c2 ) 0 1 0 [R 2 → R 2 − R1 and R3 → R3 − R1 ]
1 b2 c2 + 1 0 0 1

= (1 + a2 + b2 + c2 ) (1.1.1) = 1 + a2 + b2 + c2 = R.HS.

Hence proved.

PLANCESS CONCEPTS

|AB| = |A||B|
The value of the determinant is the same when expanded by any row or any column. Using this property
it is easier to expand determinant using a row or column in which most zeroes are involved.
Vaibhav Gupta (JEE 2009 AIR 54)

4. SYMMETRIC AND SKEW SYMMETRIC DETERMINANTS


4.1 Symmetric Determinant
a h g
A determinant is called Symmetric Determinant if ai j = a ji , ∀ i, j e.g. h b f
g f c
M a them a ti cs | 17.9

4.2 Skew Symmetric Determinant


A determinant is called a skew symmetric determinant if ai j =−a ji ∀ i, j for every element.
0 3 −1
E.g. −3 0 5
1 −5 0

Note: (i) det |A| = 0 ⇒ A is singular matrix (ii) det | A | ≠ 0 ⇒ A is non-singular matrix

PLANCESS CONCEPTS

The value of a skew symmetric determinant of an even order is always a perfect square and that of an
odd order is always zero.
Vaibhav Krishnan (JEE 2009 AIR 22)

5. MULTIPLICATION OF TWO DETERMINANTS


(a) Multiplication of two second order determinants as follows: (as R to C method)

a1 b1 l1 m1 a1l1 + b1l2 a1m1 + b1m2


× =
a2 b2 l2 m2 a2l1 + b2l2 a2m1 + b2m2

(b) Multiplication of two third order determinants is defined.
a1 b1 c1 l1 m1 n1
a2 b2 c2 × l2 m2 n2 (as R to C method)
a3 b3 c3 l3 m3 n3

a1l1 + b1l2 + c1l3 a1m1 + b1m2 + c1m3 a1n1 + b1n2 + c1n3
= a2l1 + b2l2 + c2l3 a2m1 + b2m2 + c2m3 a2n1 + b2n2 + c2n3
a3l1 + b3l2 + c3l3 a3m1 + b3m2 + c3m3 a3n1 + b3n2 + c3n3

Note:
(i) The two determinants to be multiplied must be of the same order.
(ii) To get the Tmn (term in the mth row nth column) in the product, Take the mth row of the 1st determinant and
multiply it by the corresponding terms of the nth column of the 2nd determinant and add.
(iii) This method is the row by column multiplication rule for the product of 2 determinants of the nrd order
determinant.
(iv) If ∆ ′ is the determinant formed by replacing the elements of a ∆ of order n by their corresponding co-factors
then ∆ ′ =∆n−1 . ( ∆ ′ is called the reciprocal determinant).
2
0 c b
Illustration 12: Reduce the power of the determinant c 0 a to 1. (JEE MAIN)
b a 0
Sol: By multiplying the given determinant two times we get the determinant as required.
2
0 c b 0 c b 0 c b b2 + c2 ab ac
2 2
c 0 a = c 0 a c 0 a ⇒ ba c +a bc
b a 0 b a 0 b a 0 ca cb a + b2
2
1 7 . 1 0 | Determinants

2
a2 + x2 ab − cx ac + bx
x c −b
2 2
Illustration 13: Show that ab + cx b + x bc − ax = −c x a . (JEE ADVANCED)
2
ac − bx bc + ax c + x 2 b −a x

Sol: By replacing all elements of L.H.S. to their respective cofactors and using determinant property we will obtain
the required result.

x c −b
Let D = −c x a
b −a x

Co-factors of 1st row of D are x2 + a2 , ab + cx , ac − bx . Co-factors of 2nd row of D are ab − cx, x2 + b2, ax + bc and
co-factors of 3rd row of D are ac + bx , bc − ax , x2 + c2
∴ Determinant of cofactors of D is

x2 + a2 ab + cx ac − bx a2 + x2 ab − cx ac − bx
c 2 2
D = ab − cx x + b ax + bc = ab + cx b2 + x2 bc − ax = D2
ac + bx bc − ax x 2 + c2 ac − bx ax + bc x2 + c2
2
x c −b
(Row interchanging into columns) = −c x a ( Dc = D2 , D is third order determinant)
b −a x
2
a2 + x2 ab − cx ac + bx
x c −b
2 2
Hence ab + cx b + x bc − ax = −c x a
2
ac − bx bc + ax c + x 2 b −a x

6. SOME STANDARD DETERMINANTS

1 a a2 a b c 1 1 1
2 2 2 2 2
2
(i) 1 b b =(a − b)(b − c)(c − a) (ii) a b c = a b c2 = (a − b)(b − c)(c − a)(ab + bc + ca)
1 c c2 bc ca ab a3 b3 c3

2 3
a bc abc a a a 1 1 1
2 3
(iii) b ca abc= b b b = abc (a − b)(b − c)(c − a) ; (iv) a b c = (a − b)(b − c)(c − a) (a + b + c)
c ab abc c c2 c3 a3 b3 c3

a b c
(v) b c a =−a3 − b3 − c3 + 3abc
c a b

p+ q 2 r r
Illustration 14: Evaluate the determinant =
∆ qr + 2p r 2r , where p, q and r are positive real
numbers.  q + pr qr r (JEE MAIN)

Sol: Taking r common from C2 and C3 of the given determinant using scalar multiple property and then
expanding it using the invariance property we can evaluate the given problem.
M a them a ti cs | 17.11

p+ q 2 1
We get ∆ = r qr + 2p r 2
q + pr q r

Applying C1 → C1 − qC2 − pC3

− q 2 1
We get D = r 0 r 2 = −r q(r − 2q) = r q 2 − r q . ( )
0 q r

a b c
Illustration 15: Let a, b, c be positive and not equal. Show that the value of the determinant b c a is negative.
c a b
 (JEE ADVANCED)

Sol: By applying invariance and scalar multiple properties to the given determinant we can get the required result.

a b c a+b+c b c
D = b c a ; then D = a + b + c c a [C1 → C1 + C2 + C3 ]
c a b a+b+c a b

1 b c
= (a + b + c) 1 c a [Taking (a + b+ c) common from the first column]
1 a b

1 b c
= (a + b + c) 0 c − b a − c [ R 2 → R 2 − R1 and R 3 → R 3 − R1 ]
0 a−b b −c

= (a + b + c+ ) [(c − b)(b − c) − (a − b)(a − c)] = (a + b + c+ ) [bc + ca + ab − a2 − b2 − c2 ]


1
= −(a + b + c) (a2 + b2 + c2 − bc − ca − ab) = − (a + b + c)(2a2 + 2b2 + 2c2 − 2bc − 2ca − 2ab)
2
1
= − (a + b + c) [(a + b − 2ab) + (b + c − 2bc) + (c2 + a2 − 2ac)]
2 2 2 2
2
1 2 2 2
= − (a + b + c) [(a − b) + (b − c) + (c − a) ]  … … (i)
2
 a, b, c, are positive ⇒a+b+c>0
 a, b, c are unequal ⇒ (a − b)2 + (b − c)2 + (c − a)2 > 0  … …(ii)
∴ From (i) and (ii) , ∆ < 0 .

1 cos2 (α − β) cos2 (α − γ )
Illustration 16: Show that
= ∆ cos2 (β − α ) 1 cos2 (β −=
γ ) 2sin2 (β − γ )sin2 ( γ − α )sin2 (α − β) (JEE ADVANCED)
cos2 ( γ − α ) cos2 ( γ − β) 1
Sol:.By Putting β − γ = A , γ − α =B , α − β = C and then by using switching and invariance properties we can prove
the above problem.

1 cos2 C cos2 B
We can write ∆ as, ∆ = cos2 C 1 cos2 A (Note that A + B + C = 0.)
2 2
cos B cos A 1
1 7 . 1 2 | Determinants

Using C2 → C2 − C1 , C1 → C3 − C1 we get

1 − sin2 C − sin2 B 1 − sin2 C − sin2 B


∆ cos2 C sin2 C cos2 A − cos2 C cos2 C sin2 C sinBsin(C − A)
2 2 2 2 2
cos B cos A − cos B sin B cos B sinCsin(B − A) sin2 B

1 sin2 C sin2 B
= ( −1 ) cos2 C
2
− sin2 C sinB sin( −A)
2
cos B sinC sin(B − A) − sin2B

[ cos2 A − cos2 B =sin(A + B)sin(B − A), A + B =−C,C + A =−B ]; = sinC sinB [ ∆1 ]

1 sin2 C sinB
2 2
where =
∆1 cos C − sin C sin(C − A) Using R 2 → R 2 − R1 and R 3 → R 3 − R1 we get
2
cos B sin(B − A) − sinB

1 sinC sinB
2 2
∆1 = − sin C −2sin C sin(C − A) − sinB
2
− sin B sin(B − A) − sinC −2sin2 B

But sin (C – A) – sin B = sin (C – A) + sin (C + A) = 2 sin C cos A and sin (B – A) – sin C = 2 sin B cos A
1 sinC sinB
Therefore,
= ∆1 sinCsinB= ∆2 where ∆2 sinC 2 −2cos A
sinB −2cos A 2

Applying R 2 → R 2 − sinC R1 and R3 → –sinB R1 we get

1 sinC sinB
=∆2 0 2 − sin2 C −2cosA − sinBsinC = (2 − sin2 B)(2 − sinC) − (2cos A + sinBsinC)2
0 −2cos A − sinBsinC 2 − sin2 B

= 4 − 2sin2 B − 2sin2 C + sin2 Bsin2 C − [4 cos2 A + 4 cos A sinBsinC + sin2 Bsin2 C]

= 4 sin2 A − 2sin2 B − 2sin2 C − 4 cos A sinBsinC

= 2sin2 A − 2 [sin2 B + sin2 C − sin2 A + 2cos A sinBsinC]

But A + B + C = 0 implies; sin2 B + sin2 C − sin2 A =


−2cos A sinBsinC
∴ ∆2 =2sin2 A ; sinB∆1 sin2 C sin2 B∆2
Hence, D = sinC=
= 2sin2 A sin2 Bsin2 C = 2sin2 (α − β)sin2 (β − γ )sin2 ( γ − α ) .

Illustration 17: Prove that the following determinant vanishes if any two of x; y; z are equal
sinx sin y sinz  (JEE ADVANCED)
∆ = cos x cos y cos z
cos3 x cos3 y cos3 z

Sol: Taking cos x, cos y, and cos z common from first, second and third column using scalar multiple and then using
the invariance property we can prove the given statement.
M a them a ti cs | 17.13

tanx tan y tanz


Here, ∆ =cos x cos y cos z 1 1 1
cos2 x cos2 y cos2 z

tanx tan y − tanx tanz − tan y


= cos x cos y cos z 1 0 0 (C3 → C3 − C2 , C2 → C2 − C1 )
2 2 2 2 2
cos x cos y − cos x cos z − cos y

tan y − tanx tanz − tan y


Expanding along R 2 ; ∆ = − cos x cos y cos z
cos y − cos x cos2 z − cos2 y
2 2

sin(y − x) sin(z − y)
cos z.sin(x − y) cos x.sin(y − z)
= − cos x cos y cos z cos x cos y cos y.cos z =  … (i)
2 2 2 2
sin(x + y).sin(x − y) sin(y + z).sin(y − z)
sin x − sin y sin y − sin z

cos z cos x
=sin(x − y)sin(y − z) = sin(x − y)sin(y − z)[sin(y + z)cos z − sin(x + y)cos x)]
sin(x + y) sin(y + z)
1
= sin(x − y)sin(y − z)[{sin(y + 2z) + sin y} − {sin(y + 2x} + sin y}]
2
1 1
= sin(x − y)sin(y − z)[sin(y + 2z) − sin(y + 2x)]
= sin(x − y)sin(y − z)2cos(x + y + z)sin(z − x)
2 2
= sin(x − y)sin(y − z)sin(z − x)cos(x + y + z)
π
Clearly, ∆ is zero when any two of x, y, z are equal or x + y + z = .
2
Hence proved.

7. SYSTEM OF EQUATIONS
7.1 Involving Two Variables
a1x+b1y+c1 = 0
a2x+b2y+c2 = 0

Consistent Inconsistent
System of equation has solution System of equation has no solution
a1 b1 c1
= =
a2 b2 c2
Unique solution Infinite solution
a1 b1 a1 b1 c1 (Equation represents parallel disjoint lines)
= or a1b2 a2b1 0 = =
a2 b2 a2 b2 c2
(Equation represents (Equation represents
intersecting lines) coincident lines)

Figure 17.3

∆1 ∆2 b1c2 − b2c1 a2c1 − a1c2


Solution to this system of equations is given by x = ,y = or x = ; y=
∆ ∆ a1b2 − a2b1 a1b2 − a2b1

b c1 c a1 a b1
where ∆1 = 1 , ∆2 = 1 and ∆ = 1
b2 c2 c2 a2 a2 b2
1 7 . 1 4 | Determinants

7.2 Involving Three Variables

a1 x + b1 y + c1 z =
d1 a2 x + b2 y + c2 z =
d2 a3 x + b3 y + c3 z =
d3
To solve this system we first define the following determinants

a1 b1 c1 d1 b1 c1 a1 d1 c1 a1 b1 d1
∆ = a2 b2 c2 , ∆1 =d2 b2 c2 , ∆2 =a2 d2 c2 , ∆3 =a2 b2 d2
a3 b3 c3 d3 b3 c3 a3 d3 c3 a3 b3 d3

Now following algorithm is followed to solve the system (CRITERION FOR CONSISTENCY)

Check value of 

 0  0

Consistent system and Check the values of 1, 2 and 3


has unique solution
1 2 3
x= ;y= ;z=
   Atleast one of 1, 2 and 3 1= 2 = 3=0
is not zero
Put z = t and solve any two equations
Inconsistent system to get the values of x and y in terms of t

Figure 17.4

This method of finding solution to a system of equations is called Cramer’s rule.


Note:
(a) If ∆ =0 and ∆1 =∆2 =∆3 =0 , then system of equation may or may not be consistent:
(i) If the value of x, y and z in terms of t satisfy the third equation then system is said to be consistent and
will have infinite solutions.
(ii) If the values of x, y, z don’t satisfy the third equation, then system is said to be inconsistent and will have
no solution.
(b) If d=
1 d=
2 d=
3 0 , then system of linear equations is known as Homogeneous linear equations, which always
possess at least one solution i.e. (0, 0, 0). This is called trivial solution for homogeneous linear equations.
(c) If the system of homogeneous linear equations possess non-zero/nontrivial solutions, and ∆ = 0. In such case
given system has infinite solutions.
We can also solve these solutions using the matrix inversion method.
We can write the linear equations in the matrix form as AX = B where

 a1 b1 c1  x d1 
     
A = a2 b2 c2  , X =  y  and B = d2 
a b c3  z  d 
 3 3    3
Now, solution set is obtained by solving X = A-1B. Hence the solution set exists only if the inverse of A exists.

Illustration 18: Solve the following equations by Cramer’s rule x + y + z = 9, 2x + 5y + 7z = 52, 2x+ y – z = 0.
 (JEE MAIN)
M a them a ti cs | 17.15

Sol: Here in this problem define the determinants ∆ , ∆1 , ∆2 and ∆3 and find out their value by using the invariance
property and then by using Cramer’s rule, we can get the values of x, y and z.
1 1 1
Here ∆ = 2 5 7 (Applying C2 → C2 − C1 and C3 → C3 − C1 )
2 1 −1

1 0 0 9 1 1
∴ ∆ 2=
= 3 5 1 (–9 + 5) = – 4; ∆1 =52 5 7 (Applying C2 → C2 + C3 )
2 −1 −3 0 1 −1

9 2 1 1 9 1
∴ ∆1 =52 12 7 =−1 (108 – 104) = –4; ∆2 =2 52 7 (Applying C1 → C1 + 2C3 )
0 0 −1 2 0 −1

3 9 1 1 1 9
∴ ∆2 =16 52 7 =−1 (156 – 144) = –12 and ∆3 =2 5 52 (Applying C1 → C1 − 2C2 )
0 0 −1 2 1 0

−1 1 9
∴ ∆3 = −8 5 52 (Applying C1 → C1 − 2C2 ) =−1( −52 + 72) =−20
0 1 0

∆1 −4 ∆ 2 −12 ∆3 −20
∴ By Cramer’s rule=
x = = 1 ,=
y = = 3 and=
z = = 5
∆ −4 ∆ −4 ∆ −4
∴ x = 1, y = 3, z = 5

4 3 6 6
Illustration 19: Solve the following linear equations: + −1 and
= − −5 
= (JEE MAIN)
x +5 y + 7 x +5 y + 7

1 1
Sol: Here in this problem first put = a and = b and then define the determinants ∆, ∆1 and ∆2. Then by
x +5 y+7
using Cramer’s rule we can get the values of x and y.
1 1
Let us put = a and = b then the 2 linear equations become
x +5 y+7
4a + 3b = – 1  …(i)
and 6a – 6b = –5  …(ii);
Using Cramer’s Rule, we get,
x y 1 a b 1
= = ⇒= =
−1 3 4 −1 4 3 6 + 15 −20 + 6 −24 − 18
−5 −6 6 −5 6 −6

a b 1 −1 1
∴ = = ⇒ a = and b =
21 −14 −42 2 3
1 1 1
∴ a= − ⇒ = − ⇒ 2 = – x – 5 ⇒ x = –7
2 x +5 2
1 1 1
b= ⇒ = ⇒ 3 = y + 7 ⇒ y = –4
3 y+7 3
1 7 . 1 6 | Determinants

Illustration 20: For what value of k will the following system of equations possess nontrivial solutions.
Also find all the solutions of the system for that value of k.
x + y – kz = 0; 3x – y – 2z = 0; x – y + 2z = 0.  (JEE ADVANCED)

Sol: Here in this problem first define ∆. As we know that, for non-trivial solution ∆ =0 .
So by using the invariance property we can solve ∆ =0 and will get the value of k.
For non-trivial solution, ∆ =0
1 1 −k 2 0 −k + 2
⇒ 3 −1 −2 =0 ⇒ 2 0 −4 =0 [R1 → R1 + R3 , R 2 → R 2 − R3 ]
1 −1 2 1 −1 2

Expanding along C2 . ⇒ –(–1) [–8 – 2 (2 – k)] = 0 ⇒ 2k – 12 = 0 ⇒ k=6


Putting the value of k in the given equation, we get,
x + y − 6z = 
0 … (i)
3x − y − 2z = 
0 … (ii)
x − y + 2z =
0  ... (iii)
x
(i) + (ii) ⇒ 4x – 8z = 0 ∴ z=
2
Putting the value of z in (i), we get x + y – 3x = 0 ∴ y = 2x
t
Thus when k = 6, solution of the given system of equations will be x = t, y = 2t, z = , when t is an arbitrary number.
2

Illustration 21: Solve the following equations by matrix inversion.


2x + y + 2z = 0 2x – y + z = 10 x + 3y – z = 5  (JEE ADVANCED)

Sol: By writing the given equations into the form of AX = D and then multiplying both side by A −1 we will get the
required value of x, y and z.
2 1 2   x   0 
    
In the matrix form, the equations can be written as 2 −1 1  y  = 10 
1 3 −1  z   5 
    
2 1 2  x 0
     
∴ AX = D where A = 2 −1 1 , X = y  , D = 10 
1 3 −1  z  5
    
⇒ A −1 (AX) = A −1D ⇒ X = A −1D  … (i)

2 1 2
−1 adjA
Now A = ; =| 2 −1 1 = 2(1 – 3) – 1(–2 –1) + 2(6 + 1) = 13
|A
|A|
1 3 −1
 −2 3 7  −2 7 3  −2 7 3
    −1 1  
The matrix of cofactors of |A| is  7 −4 −5 . So, adj A=  3 −4 2=; A  3 −4 2 .
13
 3 2 4  7 −5 −4   7 −5 −4 
     
 −2 7 3  0   0 + 70 + 15   85 / 13  x  85 / 13 
1    1        
∴ from (1), X =  3 −4 2 10  = 0 − 40 + 10  =−  30 / 13 ; y  =  −30 / 13 .
13 13
 7 −5 −4   5   0 − 50 − 20   −70 / 13 z   −70 / 13
          
85 −30 −70
⇒ x= ,y= ,z=
13 13 13
M a them a ti cs | 17.17

PLANCESS CONCEPTS

In general if r rows (or columns) become identical when a is substituted for x, then (x − a)r −1 is a factor
of the given determinant.
Anvit Tawar (JEE 2009 AIR 9)

7.3 Some Important Results


The lines: a1 x + b1 y + c1 =
0  … (i)
a2 x + b2 y + c2 =
0 … (ii)
a3 x + b3 y + c3 =
0  ... (iii)
a1 b1 c1
are concurrent if, a2 b2 c2 = 0
a3 b3 c3

This is the condition for the consistency of three simultaneous linear equations in 2 variables.
2 2
(a) ax + 2hxy + by + 2gx + 2fy + c = 0 represents a pair of straight lines if
a h g
2 2 2
abc + 2fgh − af − bg − ch =0 =h b f
g f c
x1 y1 1
1
(b) Area of a triangle whose vertices are (xr , yr ) ; r = 1, 2, 3 is : D = x2 y 2 1 . If D = 0 then the three points
are collinear. 2
x3 y 3 1

x y 1
(c) Equation of a straight line passing through (x1 , y1 ) & (x2 , y 2 ) is x1 y1 1 = 0 .
x2 y2 1

(d) 
If each element of any row (or column) can be expressed as a sum of two terms, then the determinant can be
expressed as the sum of the determinants.
a1 + x b1 + y c1 + z a1 b1 c1 x y z
E.g., a2 b=2 c2 a2 b2 c2 + a2 b2 c2
a3 b3 c3 a3 b3 c3 a3 b3 c3

It should be noted that while applying operations on determinants at least one row (or column) must remain
unchanged i.e.
Maximum number of simultaneous operations = order of determinant – 1

PLANCESS CONCEPTS

Always expand a determinant along a row or a column with maximum zeros.


To find the value of the determinant, the following steps are taken.
Take any row (or column); the value of the determinant is the sum of products of the elements of the row
(or column) and the corresponding determinant obtained by omitting the row and the column of the
elements with a proper sign, given by ( −1)p + q where p and q are the no. of row and the no. of column
respectively.
Vaibhav Krishnan (JEE 2009 AIR 22)
1 7 . 1 8 | Determinants

8. DIFFERENTIATION AND INTEGRATION OF DETERMINANTS


f1 (x) g1 (x)
Let ∆(x) = , where f1 (x), f2 (x), g1 (x) and g2 (x) are functions of x. Then,
f2 (x) g2 (x)

f1′ (x) g1′ (x) f1 (x) g1 (x) f1′ (x) g1 (x) f1 (x) g1′ (x)
∆ ′(x)
= + Also,
= ∆ ′(x) +
f2 (x) g2 (x) f2′ (x) g2′ (x) f2′ (x) g2 (x) f2 (x) g2′ (x)
Thus, to differentiate a determinant, we differentiate one row (or column) at a time, keeping others unchanged. If we
write ∆(x) = [C1 C2 ] , where Ci denotes the ith column, then ∆= ′(x) [C1′ C2 ] + [C1 C2′ ] , where Ci′ denotes the
R1  R ′   R1 
column obtained by differentiating functions in the ith column Ci . Also, if ∆(x) =
  , then ∆ ′(x)=  1  +  
R 2  ′
 R 2  R 2 
Similarly, we can differentiate determinants of higher order.

Note: Differentiation can also be done column wise by taking one column at a time.

If f(x), g(x) and h(x) are functions of x and a, b, c, α, β and γ are constants such that

f(x) g(x) h(x) ∫ f(x)dx ∫ g(x)dx ∫ h(x)dx


∆(x) =a b c , then the integration of ∆(x) is given by ∫ ∆(x)dx =a b c
α β γ α β γ

sin2 x logcos x logtanx π /2


Illustration 22:=
If ∆(x) n2 2n − 1 2n + 1 , then evaluate ∫ ∆(x) dx .  (JEE MAIN)
1 −2log2 0 0

Sol: By applying integration on variable elements of determinant we will solve the given problem.
π/2 π/2 π/2

∫ sin2 x dx ∫ log cos x dx ∫ logtanx dx


sin2 x logcos x logtanx π /2 0 0 0
2 2
We have,
= ∆(x) n 2n − 1 ; ∫ ∆(x) dx
2n + 1 = n 2n − 1 2n + 1
1 −2log2 0 0 1 −2log2 0

π π
− log2 0
4 2 1 −2log2 0
π 2 π
= n2 2n − 1 2n + 1= n 2n − 1 2n + 1 = × 0 = 0
4 4
1 −2log2 0 1 −2log2 0

xn
sinx cos x
nπ nπ dn
Illustration 23: If f(x) = n! sin cos , then show that {f(x)} = 0 at x = 0. (JEE ADVANCED)
2 2 dxn
a a2 a3

Sol: By applying integration on variable elements of the determinant we will solve the given problem.

dn dn dn
n (xn ) (sinx) (cos x)
x sinx cos x dxn dxn dxn
nπ nπ dn nπ nπ
We have, f(x) = n! sin cos ; {f(x)} = n! sin cos
2 2 dx n 2 2
a a2 a3 a a2 a3
M a them a ti cs | 17.19

 nπ   nπ  nπ nπ
n! sin  x +  cos  x +  n! sin cos+
 2   2  2 2
nπ nπ  dn  nπ nπ
= n! sin cos  n {f(x)} 
; = n!
= sin cos 0
2 2  dx  2 2
  x =0
a a2 a3 a a2 a3

PROBLEM-SOLVING TACTICS

f1 (x) f2 (x) f3 (x) f1′ (x) f2′ (x) f3′ (x)


Let ∆(x) = b1 b2 b3 , then ∆ ′(x) = b1 b2 b3 and in general
c1 c2 c3 c1 c2 c3

f1n (x) f2n (x) f3n (x)


∆n (x) = b1 b2 b3 where n is any positive integer and f n (x) denotes the nth derivative of f(x).
c1 c2 c3

f(x) g(x) h(x)


Let ∆(x) =a b c , where a, b, c, l, m and n are constants.
l m n

b b b

∫ f(x) dx ∫ g(x)dx ∫ h(x) dx


b a a a
⇒ ∫ ∆(x) dx =a b c
a l m n

If the elements of more than one column or rows are functions of x then the integration can be done only
after evaluation/expansion of the determinant.

FORMULAE SHEET

a1 b1 c1
b c2 a c2 a b2
(a) Determinant of order 3 × 3 = a2 b2 c2 = a1 2 − b1 2 + c1 2
b3 c3 a3 c3 b3 b3
a3 b3 c3

a11 a12 a13


a a23
(b) In the determinant D = a21 a22 a23 , minor of a12 is denoted as M12 = 21 and so on.
a31 a33
a31 a32 a33

(c) Cofactor of an element ai j = Ci j = ( −1)i + j Mi j


1 7 . 2 0 | Determinants

(d) Properties of determinants:

(i) Reflection property: Ai× j = A j×i

(ii) All-zero property: If all the elements of a row (or column) are zero, then the determinant is zero.
(iii) Proportionality (Repetition) Property: If all the elements of a row (or column) are proportional
(identical) to the elements of some other row (or column), then the determinant is zero.
(iv) Switching Property: The interchange of any two rows (or columns) of the determinant changes its sign.
(v) Scalar Multiple Property: If all the elements of a row (or column) of a determinant are multiplied by a
non-zero constant, then the determinant gets multiplied by the same constant.
a1 + b1 c1 d1 a1 c1 d1 b1 c1 d1
(vi) Sum Property: a2 + b2 c2 d2 = a2 c2 d2 + b2 c2 d2
a3 + b3 c3 d3 a3 c3 d3 b3 c3 d3

a1 b1 c1 a1 + αb1 + βc1 b1 c1
(vii) Property of Invariance: a2 b2 c2= a2 + αb2 + βc2 b2 c2
a3 b3 c3 a3 + αb3 + βc3 b3 c3

That is, a determinant remains unaltered under an operation of the form Ci → Ci + αC j + βCk , where j, k ≠ i ,
or an operation of the form R i → R i + αR j + βRk , where j, k ≠ i .

a1 a2 a3 a1 0 0
(viii) Triangle Property: =
0 b2 b3 a=
2 b2 0 a1b2c3
0 0 c3 a3 b3 c3

∆ ∆ ∆
(e) Cramer’s rule : if a1 x + b1 y + c1 z =
d1 , a2 x + b2 y + c2 z = d3 then x = 1 , y = 2 , z = 3
d2 and a3 x + b3 y + c3 z =
where ∆ ∆ ∆

a1 b1 c1 d1 b1 c1 a1 d1 c1 a1 b1 d1
∆ = a2 b2 c2 , ∆1 =d2 b2 c2 , ∆2 =a2 d2 c2 and ∆3 =a2 b2 d2 .
a3 b3 c3 d3 b3 c3 a3 d3 c3 a3 b3 d3

∆1 ∆2
And if a1 x + b1 y + c1 =
0 and a2 x + b2 y + c2 =
0 then x = y= .
∆ ∆
b c1 c a1 a b1
Where ∆1 = 1 , ∆2 = 1 and ∆ = 1
b2 c2 c2 a2 a2 b2
a1 b1 c1
(f) (i) lines a1 x + b1 y + c1 =
0 , a2 x + b2 y + c2 =
0 and a3 x + b3 y + c3 =
0 are concurrent if, a2 b2 c2 = 0
a3 b3 c3
a h g
2 2
(ii) ax + 2hxy + by + 2gx + 2fy + c =0 represents a pair of straight lines if h b f =0
g f c

x1 y1 1
1
(iii) area of a triangle whose vertices are (xr , yr ) ; r = 1, 2, 3 is : D = x2 y2 1
2
x3 y3 1

x y 1
(iv) Equation of a straight line passing through (x1 , y1 ) & (x2 , y 2 ) is x1 y1 1 = 0
x2 y2 1
M a them a ti cs | 17.21

f1 (x) g1 (x) f ′ (x) g1′ (x) f1 (x) g1 (x) f1′ (x) g1 (x) f1 (x) g1′ (x)
(g) If ∆(x) = =
then ∆ ′(x) 1 + or +
f2 (x) g2 (x) f2 (x) g2 (x) f2′ (x) g2′ (x) f2′ (x) g2 (x) f2 (x) g2′ (x)

f(x) g(x) h(x) ∫ f(x)dx ∫ g(x)dx ∫ h(x)dx


(h) If ∆(x) =a b c then ∫ ∆(x)dx =a b c
α β γ α β γ

Solved Examples

JEE Main/Boards 3 3 3
⇒ R.H.S. = pqr(a + b + c − 3abc)  .… (ii)

Example 1: Prove that From eq. (i) and (ii), we get

pa qb rc a b c ∴ L.H.S. = R.H.S.
qc ra pb = pqr c a b . Use p + q + r = 0.
rb pc qa b c a Example 2: Prove that the determinant

x sin θ cos θ
Sol: By using the expansion formula of determinants we
− sin θ −x 1 is independent of θ.
can prove this.
cos θ 1 x
pa qb rc
L.H.S.= qc ra pb =
Sol: Simply by expanding the given determinant we can
rb pc qa prove it.
ra pb qc pb qc ra x sin θ cos θ
pa − qb + rc We have, − sin θ −x 1
pc qa rb qa rb pc
cos θ 1 x
= pa(a2qr − p2bc) − qb(q2ac − prb2 ) + rc(pqc2 − r 2ab)
−x 1 − sin θ 1 − sin θ −x
=x − sin θ + cos θ
3 3 3 3
= a pqr − p abc − q abc + b pqr − r abc3 1 x cos θ x cos θ 1

= pqr(a3 + b3 + c3 ) − abc(p3 + q3 + r 3 ) x( −x2 − 1) − sin θ ( −x sin θ − cosθ) + cos θ ( − sin θ + x cos θ)

 p+q+r =0 … (given) = −x3 − x + x sin2 θ + sin θ cosθ − sin θ cos θ + x cos2 θ

(p + q + r)3 = 0 = −x3 − x + x(sin2 θ + cos2 θ) = −x3 − x + x

⇒ p3 + q3 + r3 ‒ pqr = 0 ⇒ p3 + q3 + r3 = 3pqr Thus, the determinant is independent of θ .

⇒ L.H.S. = pqr(a3 + b3 + c3 ) − abc(3pqr) x+a x x


3 3 3
⇒ L.H.S. = pqr(a + b + c − 3abc)  .… (i) Example 3: Solve the equation x x+a x =0,
a≠0. x x x+a
 a b c b c a
R.H.S. = pqr a −b +c 
 c a b a b c  Sol: We can expand the above determinant by applying
the invariance and scalar multiple properties, and hence
= pqr[a(a2 − bc) − b(ca − b2 ) + c(c2 − ab)] we can easily solve this problem.
= pqr[a3 − abc − abc + b3 + c3 − abc]
1 7 . 2 2 | Determinants

x+a x x 2 −1 3
We have, x x+a x =0 Here, ∆ = 1 1 1
x x x+a 1 −1 1

Operation: C1 → C1 + C2 + C3 =2(1 + 1) + 1 (1 – 1) + 3 (–1 – 1) = – 2,


9 −1 3
3x + a x x 1 x x ∆ x =6 1 1 =9(1 + 1) + 1(6 – 2) + 3 (–6 – 2) = –2
3x + a x + a x =0 ⇒ (3x + a) 1 x + a x =0 2 −1 1
3x + a x x+a 1 x x+a
2 9 3
Operating R 2 → R 2 − R1 , R 3 → R 3 − R1 ∆ y =1 6 1 =2(6 – 2) – 9(1 – 1) + 3(2 – 6) = – 4
1 2 1
1 x x
We get (3x + a) 0 a 0 =
0 2 −1 9
0 0 a ∆ z =1 1 6 =2(2 + 6) + 1(2 – 6) + 9(–1 – 1) = –6
1 −1 2
a 0
⇒ (3x + a) 0 ⇒ a2 (3x + a) =
= 0 By Cramer’s Rule
0 a
∆x ∆y ∆z
a x =
= 1 ,=y = 2 ,=
z = 3
⇒ 3x + a = 0, [ a ≠ 0] ⇒ x = − ∆
3 ∆ ∆
Hence Proved.
Example 6: Show that
Example 4: Solve, using Cramer’s rule 3x ‒ 2y + 4z = 5;
a + b + 2c a b
x + y + 3z = 2; ‒x + 2y ‒ z = 1
c b + c + 2a b = 2(a + b + c)3
Sol: By defining D, D1, D2, D3 and by using Cramer’s c a c + a + 2b
Rule we will get required result.
Sol: By using invariance and scalar multiple property
3 −2 4 we can expand given determinant and can prove it.
D= 1 1 3 = −5
−1 2 −1 2(a + b + c) a b
∆ 2(a + b + c) b + c + 2a
= b
5 −2 4 3 5 4 2(a + b + c) a c + a + 2b
D1 = 2 1 3 = −33 , D2 = 1 2 3 = −13
[C1 → C1 + C2 + C3 ]
1 2 −1 −1 1 −1
1 a b
3 −2 5
= 2(a + b + c) 1 b + c + 2a b
=D3 =1 1 2 12
1 a c + a + 2b
−1 2 1
1 a b
D1 −33 33
x =
By Cramer’s Rule,= = , = 2(a + b + c) 0 b + c + a 0
D −5 5
0 a c+a+b
D2 −13 13 D3 12 −12
y
= = = ; =
z = =
D −5 5 D −5 5 [by R 2 → R 2 − R1 and R 3 → R 3 − R1 ]

= 2(a + b + c)[1{(b + c + a)2 − 0}]


Example 5: Solve the following system of equations by
Cramer’s Rule
= 2(a + b + c)(a + b + c)2 = 2(a + b + c)3
2x − y + 3z
= 9; x + y +=
z 6; x − y +=
z 2

Sol: By defining ∆ , ∆ x , ∆ y , ∆ z and by using Cramer’s Example 7: Using determinants, show that the points
Rule we will get the required result. (11, 7), (5, 5) and (–1, 3) are collinear.
M a them a ti cs | 17.23

Sol: If these points are collinear then the area of a JEE Advanced/Boards
triangle made by joining these points will be zero.
The area of the triangle formed by the given points Example 1: Without expanding, evaluate the determinant
11 7 1 sin α cos α sin(α + δ)
1
= 5 5 1 sin β cos β sin(β + δ)
2
−1 3 1 sin γ cos γ sin( γ + δ)
Operate: R1 → R1 − R 2 ; R 2 → R 2 − R 3
Sol: By using the formula sin(A+ B) = sin A cos B + cos
6 2 0 A sin B and invariance property of determinants we can
1 1
= 6 2 0 = ⋅0 = 0 expand the given determinant.
2 2
−1 3 1 sin α cos α sin(α + δ)
( R1 and R 2 are identical) Let =
∆ sin β cos β sin(β + δ)
sin γ cos γ sin( γ + δ)
Hence, the given points are collinear.
sin α cos α sin α cos δ + cos α sin δ
Example 8: If A and B are two matrices such that AB = ⇒=
∆ sin β cos β sin β cos δ + cos β sin δ
B and BA = A, then A2 + B2. sin γ cos γ sin γ cos δ + cos γ sin δ

Sol: By using the multiplication property of matrices we sin α cos α 0


can solve given problem.
⇒ =
∆ sin β cos β 0
A2 + B2 = AA + BB sin γ cos γ 0
= A(BA) + B(AB) [Given AB = B and BA = A]
[Applying C3 → C3 − cos δ . C1 − sin δ . C2 ]
= (AB)A + (BA)B
⇒ ∆ =0 [ C3 consists of all zeroes]
[Matrix multiplication is associatively]=BA + AB
[Given AB = B and BA = A] =A+B Example 2: By using properties of determinants prove
[Given AB = B and BA = A] that

1 x x2
12 22 32 42
22 32 42 52
x2 1 x = (1 − x3 )2
Example 9: Find the value of
32 42 52 62 x x2 1

42 52 62 72
Here in this problem by using invariance and
scalar multiple properties we will expand the given
Sol: By applying the invariance property we can find determinant and we will prove it.
the value of the given determinant.
1 x x2 1 + x + x2 x x2
12 22 32 42 1 4 9 16 1 4 9 16
Sol: L.H.S. = x2 1 x = 1 + x + x2 1 x
22 32 42 52
4 9 16 25 3 5 7 9
= = x x 2
1 1+x+x 2
x 2
1
32 42 52 6 2 9 16 25 36 5 7 9 11
42 52 62 7 2 16 25 36 49 7 9 11 13
[Applying C1 → C1 + C2 + C3 ]
[Applying R 4 → R 4 − R 3 , R3 → R3 − R 2 , R 2 → R 2 − R1 ]
1 x x2
1 4 9 16 2
= (1 + x + x ) 1 1 x
3 5 7 9 1 x2 1
=
2 2 2 2
2 2 2 2 1 x x2

[Applying R 4 → R 4 − R 3 , R 3 → R 3 − R 2 ] = (1 + x + x2 ) 0 1−x x − x2

=0 0 x2 − x 1 − x2
1 7 . 2 4 | Determinants

[Applying R 2 → R 2 − R1 and R 3 → R 3 − R1 ] a b 1
1
= (1 + x + x2 )(1){(1 − x)(1 − x2 ) − (x2 − x)(x − x2 )} c d 1 where (a, b) (c, d) (e, f) are the vertices of
2
e f 1
= (1 + x + x2 )(1 − x)2 {1 + x + x2 }
triangle. Therefore by substituting the value of vertices
= {(1 − x)(1 + x + x2 )}2 = (1 − x3 )2 = R.H.S. we will get required result.
Let the vertices of triangle be A(2, –6), B(5, 4) and C(k, 4).
Example 3: Show that x =−(a + b + c) is one root of
Since the area of the triangle ABC is 35 sq. units, we
x+a b c
2 −6 1 2 −6 1
the equation: b x+c a =0 and solve the 1 1
have, 5 4 1 = ±35 ⇒ 3 10 0 = ±35
c a x +b 2 2
k 4 1 k − 2 10 0
equation completely.
[Applying R 2 → R 2 − R1 and R 3 → R 3 − R1 ]
Sol: We can expand given determinant using the
invariance and scalar multiple properties and by solving 1 3 10
we will find out required result. ⇒ = ±35 [Expanding along C3]
2 k − 2 10
By C1 → C1 + C2 + C3 , we get
1
⇒ {30 − 10(k − 2)} =
±35
x +a+b+c b c 2
x +a+b+c x +c a =0 ⇒ 30 − 10k + 20 =
±70 ⇒ 10k = 50  70
x +a+b+c a x +b ⇒ k = +12 or k = –2
1 b c
Example 5: Solve the following system of equations by
⇒ (x + a + b + c) 1 x + c a =0
using determinants: x + y + z =,
1
1 a x +b
k ; a2 x + b2 y + c2 z =
ax + by + cz = k2
1 b c
Sol: Here in this problem first define D, D1, D2 and D3.
⇒ (x + a + b + c) 0 x − b + c a−c =0
then by using Cramer’s rule we can solve it.
0 a−b x +b−c
1 1 1
R 2 → R 2 − R1 ; R 3 → R 3 − R1 We have, D = a b c

On expanding by first column, we get a2 b2 c2

(x + a + b + c)[(x − b + c)(x + b − c) − (a − b)(a − c)] =0 1 0 0


= a b−a c−a
⇒ (x + a + b + c)[x2 − (b − c)2 − (a2 − ac − ab + bc)] =0
a2 b2 − a2 c2 − a2
⇒ (x + a + b + c)(x2 − b2 − c2 + 2bc − a2 + ac + ab − bc] =0
[Applying C2 → C2 − C1 and C3 → C3 − C1 ]
⇒ (x + a + b + c)(x2 − a2 − b2 − c2 + ab + bc + ca) =0
1 0 0
Either x + a + b + c = 0 ⇒ x = –(a + b + c)
= (b − a)(c − a) a 1 1
or x2 − a2 − b2 − c2 + ab + bc + ca =
0 a2 b + a c + a

⇒ x=± a2 + b2 + c2 − ab − bc − ca 1 1
= (b − a)(c − a).1.
b+a c+a
Example 4: If the area of a triangle is 35 sq. units with [Expanding along R1 ]
vertices (2, –6), (5, 4) and (k, 4), then find k.
= (b − a)(c − a)(c + a − b − a)
Sol: As we know that the area of the triangle =
= (b − a)(c − a)(a − b)  … (i)
M a them a ti cs | 17.25

1 1 1 [R1 → R1 − R 2 , R 2 → R 2 − R3 ]
D1 =k b c =(b − c)(c − k)(k − b)
2 −1 α
k 2 b2 c2
= −1 2 β −1
[Replacing a by k in (i)] u v − 1 1 + a3 + b3
1 1 1 [ C1 → C1 − C2 , C2 → C2 − C3 ]
D 2 =a k c =(k − c)(c − a)(a − k)
a2 k 2 c2 0 −1 α
= 3 2 β −1 [ C1 → C1 + 2C2 ]
[Replacing b by k in (i)]
u + 2v − 2 v − 1 1 + a3 + b3
1 1 1
and D3 =a b k =(a − b)(b − k)(k − a) = [3(1 + a3 + b3 ) − (u + 2v − 2)(β − 1)]
2 2 2
a b k
+ a 2 [3(v − 1) − 2(u + 2v − 2)]
[Replacing c by k in (i)]
= 3 + 3(a3 + β3 ) − uβ − 2vβ + 2β + u + 2 v
D1 (b − c)(c − k)(k − b) (c − k)(k − b)
x =
∴= = , −2 + α( − v + 1 − 2u)
D (b − c)(c − a)(a − b) (c − a)(a − b)
= 1 + 3(a3 + b3 ) + 2β + u + 2v + α − uβ −2vβ − αv − 2uα
D2 (k − c)(c − a)(a − k) (k − c)(a − k)
y =
= = = 1 + 3(a3 + β3 ) + 2(a2 − a3 ) + b1 − b2 + 2(β2 − b3 )
D (b − c)(c − a)(a − b) (b − c)(a − b)
+a − a − 2αu − 2β v − uβ − vα
D3 (a− b)(b− k)(k − a) (k − a)(b− k)
z =
= = = 1 + (a1 + b1 + a2 + b2 + a3 + b 3 ) − 2αu −2βv − uβ − vα
D (b − c)(c − a)(a − b) (c− a)(b− c)
= RHS [From (i)]
Example 6: Show that
Example 7: Find values of c for which the equations
1 + a1 + b1 a1 + b2 a1 + b3
2x + 3y =
3 ; (c + 2)x + (c + 4)y =c + 6
a2 + b1 1 + a2 + b2 a2 + b3
a3 + b1 a3 + b2 1 + a3 + b3 (c + 2)2 x + (c + 4)2 y = (c + 6)2 are consistent and hence
solve the equation.
3
=1 + ∑ (ai + bi ) + ∑ (ai − a j ) (b j − bi )
=i 1 1 ≤ i< j≤ 3 Sol: Here in this problem first define given equations as
∆ and solve it as ∆ =0 by using the invariance method.
Sol: By putting α = a1 ‒ a2, β = a2 ‒ a3 , then α + β = a1 ‒ a3 The equation will be consistent, if
u = b1 ‒ b2, v = b2 ‒ b3, then u + v = b1 ‒ b3. Using the
2 3 3
invariance property expand the given determinant, and
then comparing it to the R.H.S. of the given problem c+2 c+4 c+6 0
=
2 2 2
we can prove it. (c + 2) (c + 4) (c + 6)
Let Now R.H.S. Applying C3 → C3 − C2 , we get
3
=1 + ∑ (ai + bi ) + ∑ (ai − a j ) (b j − bi ) 2 3 0
=i 1 1 ≤ i< j≤ 3 c+2 c+4 2 0
=
=1 + (a1 + b1 + a2 + b2 + a3 + b3 ) + (a1 − a2 )(b2 − b1 ) (c + 2) 2
(c + 4)2
4(c + 5)
+ (a2 − a3 )(b3 + b2 ) + (a1 − a3 )(b3 − b1 )
Solving, we get c2 + 10c =
0
= 1 + (a1 + b1 + a2 + b2 + a3 + b3 ) − αu − βv − (α + β)(u + v)
or c = 0, –10  … (i)
= 1 + (a1 + b1 + a2 + b2 + a3 + b3 ) − 2αu − 2βv − βu − αv .
If c = 0, the system of equations becomes
 .... (i)
1+α α −1 α  2x + 3y =
3
 ⇒ x = – 3 , y = 3 .… (ii)
Now L.H.S.= β 1+β β −1 2x + 4y 6
=
a3 + b1 a3 + b2 1 + a3 + b3
1 7 . 2 6 | Determinants

If c = –10, then system of equations becomes Example 9: Let λ and α be real. Find the set of all
values of λ and α for which the system of linear
 2x + 3y = 3 1 4 equations λx + (sin α )y + (cos α )z =0
 ⇒ x = − , y =  .… (iii)
 −8x − 6y =− 4 2 3
x + (cos α )y + (sin α )z =
0 −x + (sin α )y − (cos α )z =0 has
16x + 9y = 4 a non-trivial solution. If λ =1 , find all values of α .

Hence the solutions are given by (ii) and (iii). Sol: Here in this problem first define the given equations
as ∆ and as we know that for non-trivial solution ∆ =0 .
Example 8: If (ar, br), r = 1, 2, 3 be the vertices of a For non-trivial solution, condition is ∆ =0 .
triangle, prove that
λ sin α cos α
a2 − a3 b2 − b3 a1 (a2 − a3 ) + b1 (b2 − b3 ) =∆ 1 cos α = sin α 0
∆= a3 − a1 b3 − b1 ) 0 … (i)
a2 (a3 − a1 ) + b2 (b3 − b1 =
−1 sin α − cos α
a1 − a2 b1 − b2 a3 (a1 − a2 ) + b3 (b1 − b2 )
or λ[ − cos2 α − sin2 α] − sin α [ − cos α + sin α]
and hence show that the altitudes of a triangle are
concurrent.
+ cos α [sin α + cos α] = 0
Sol: Using the invariance method we can expand the λ sin2α + cos2α ∴
or = α ∈R ; | λ | ≤ 2
given determinant and using the equations of altitude
If λ =1 , then=
1 sin2α + cos2α
we can prove it
 π 1 π
a2 − a3 b2 − b3 a1 (a2 − a3 ) + b1 (b2 − b3 ) cos  2α − = = cos
 4 2 4
∆= a3 − a1 b3 − b1 a2 (a3 − a1 ) + b2 (b3 − b1 =
) 0
π π π π
a1 − a2 b1 − b2 a3 (a1 − a2 ) + b3 (b1 − b2 ) ⇒ 2α − = 2nπ ± : n ∈ I ⇒ α = nπ ± + : n ∈ I
4 4 8 8
[Applying R1 → R1 + R 2 + R 3 ]

0 0 0 Example 10: For a fixed positive integer n, if


∆= a3 − a1 b3 − b1 a2 (a3 − a1 ) + b2 (b3 − b1 ) n! (n + 1)! (n + 2)!
a1 − a2 b1 − b2 a3 (a1 − a2 ) + b3 (b1 − b2 ) ∆= (n + 1)! (n + 2!) (n + 3)!
(n + 2)! (n + 3)! (n + 4)!
A(a1, b1)
 ∆ 
then show that  − 4  is divisible by n.
3
 (n!) 
F E
Sol: By using the scalar multiple property of
determinants we can take (n!)3 ,(n+1) and (n + 2)
B(a2, b2) D C(a3, b3) common and using the invariance property we can
solve the given problem.
∴ Equation of altitude AD is:
a − a3 1 n+1 (n + 2)(n + 1)
− 2
y − b1 = (x − a1 ) =∆ (n!)3 n+1 (n + 2)(n + 1) (n + 3)(n + 2)(n + 1)
b2 − b3
(n + 2)(n + 1) (n + 3)(n + 2)(n + 1) (n + 4)(n + 3)(n + 2)(n + 1)
or x(a2 − a3 ) + y(b2 − b3 )= a1 (a2 − a3 ) + b1 (b2 − b3 ) … (ii)
Taking (n+1) and (n + 2) (n + 1) common from C2 and
Similarly equation of altitudes BE and CF are C3 respectively, we get
x(a3 − a1 ) + y(b3 − b1 )= a2 (a3 − a1 ) + b2 (b3 − b1 ) .… (iii) =∆ (n!)3 (n + 2)(n + 1)

x(a1 − a2 ) + y(b1 − b2 )= a3 (a1 − a2 ) + b3 (b1 − b2 )  .… (iv) 1 1 1
n+1 n+2 n+3
Altitudes (ii), (iii), (iv) are concurrent, since the
determinant given by L.H.S. of (i) is zero. (n + 2)(n + 1) (n + 3)(n + 2) (n + 4)(n + 3)

[Apply C3 → C3 − C1 and C2 → C2 − C1 then


M a them a ti cs | 17.27

=∆ (n!)3 (n + 1)2 (n + 2) = (n!)3 (n + 1)2 (n + 2) ⋅ 2

1 0 0 =∆ (n!)3 (n2 + 2n + 1)(2n +=


4) (n!)3 (2n3 + 8n2 + 10n + 4)
n+1 1 2  ∆ 
∴  − 4  = 2n3 + 8n2 + 10n ,
(n + 2)(n + 1) 2(n + 2) 4n + 10 3
 (n!) 
= (n!)3 (n + 1)2 (n + 2)[4n + 10 − 4(n + 2)] 2n(n2 + 4n + 5) , which is divisible by n.

JEE Main/Boards

 3 1
Exercise 1 Q.11 Find the adjoint of matrix A =  .
 −5 4 
−1 2 2 x
Q.1 Find x, if = .  1 3
4 8 x −4 Q.12 Find the inverse of matrix   , if possible.
 −6 −18 
1 2 3 1 8
Q.2 It matrix A =   , find [A].
0 1  Q.13 Without expanding, find the value of −4 2 16 .
4 −1 0 −5 3 24
x 0 1
Q.3 Given 2 1 4 , find (i) M23 (ii) C32 .  
Q.14 If a =  2 −1 4  is a singular matrix, find x.
1 0 3 1 2 0
 

Q.4 Area of a triangle with vertices (k, 0), (1, 1) and Q.15 Find the area of the triangle whose vertices are
(0, 3) is 5 sq. units. Find the value(s) of k. (3, 1) (4, 3) and (–5, 4).

Q.5 Find the area of a triangle, whose vertices are (0, 3) Q.16 Find the value of x, if area of triangle
(–1, 4), (2, 6). is 35 square cms with vertices (x, 4), (2, –6) and (5, 4).

a11 a12 a13


Q.17 Show that the following determinant vanishes:
Q.6 Given determinant a21 a22 a23 .
a31 a32 a33 5 15 −25
7 21 30 .
Find the value of a11C21 + a12C22 + a13C33 . 8 24 42

 −1 24  Q.18 Using properties of determinants, prove that :


Q.7 Find the value of p, such that the matrix  
is singular.  4 p
1 a b+c
0.
1 b c+a =
Q.8 Given I2 . Find I2 . Also find 3I2 .
1 c a+b

Q.9 Find the value of x, such that the points (0,2), (1, x),
(3, 1) are collinear. Q.19 If points (2, 0), (0, 5) and (x, y) are collinear, then
x y
show that + = 1.
Q.10 For two given square matrices A and 2 5
B of the same order, such that |A| = 20 and |B| = –20,
find |AB|. Q.20 If for matrix A, |A| = 3 find |5A|, where matrix A is
of order 2 × 2.
1 7 . 2 8 | Determinants

 a11 a12   2 2 1 1 3 2 
Q.21 Given A = a 
a22  , such that |A| = –10. Find
 
Q.32 If A =  −2 1 2 and B =
 
 21 1 1 1  ,
a11C11 + a12C12 .  1 −2 2  2 −3 −1
   
verify that (AB)–1 = B−1 A –1 .
Q.22 Without expanding prove that, the value of
determinant (33-38) Using properties of determinant, prove that
1 a b+c a b c
1 b c + a is zero. Q.33 b c a =(a + b + c) (ab + bc + ca – a2 – b2 –c2)
1 c a+b c a b

= 3abc – a3 – b3 – c3.
Q.23 A is a non-singular matrix of order 3 and |A| = –4.
Find |adj.A|. y+z x y
Q.34 z + x y x =(x + y + z) (x – z)2.
Q.24 Is it possible to find the inverse of a matrix x+y z z

 2 1 5 −bc b2 + bc c2 + bc
  ? Given reasons.
 −1 0 3 Q.35 a2 + ac −ac c2 + ac = (ab+bc+ca)3.
a2 + ab b2 + ab −ab
Q.25 Given a square matrix A of order 3 × 3, such that
|A| = 12, find the value of |A.adj. A|.
(b + c)2 a2 bc
2 2
 2 3 Q.36 (c + a) b ac = (a – b) (b – c) (c – a) (a+ b +c)
Q.26 Compute A −1 for the matrix   and show 2 2
 5 −2  (a
(a + b) c ab 2
+ b2 + c2).
1
that A −1 = A.
19
a b−c c+b
Q.37 a + c b c − a = (a+b+c) (a2+b2+c2).
 1 −2 1
  a−b a+b c
Q.27 Let A=  −2 3 1
 1 1 5
 
a b ax + by
Verify that (i) (adj A)-1 (ii) (A−1)−1=A. Q.38 b c bx + cy =(b2– ac)(ax2+2bxy+cy2)
ax + by bx + cy 0
Q.28 Using matrix method, examine the system of
equations: 2x + 5y = 7, 6x + 15y = 13 for consistency. Q.39 Write the minors and cofactors of the elements of
second row of the following determinant:
a b  1 2 3
Q.29 Find the inverse of matrix A=  1 + bc  and
c  −4 3 6.
show that aA −1 = (a2 + bc + 1)I − aA .  a  2 −7 9

Q.40 Find the quadratic function defined by the equation


 1 tanx 
Q.30 If A =  , f(x) = ax2 + bx + c, if f(0) = 6(2) = 11 and f(–3) = 6, using
 − tanx 1  determinants.
cos2x − sin2x 
show that A′A −1 =  . Q.41 Examine whether the system of equations:
 sin2x cos2x  2x – y = 5, 4x – 2y = 10 is consistent or inconsistent.

 2 0 −1  Q.42 Verify, whether the system of equations: 3x – y – 2z


 
Q.31 If A =  5 1 0  , prove that A −1 = A2 − 6A + 11 I . = 2, 2y – z = –1, 3x – 5y = 3 is consistent or inconsistent.
 0 1 3
 
M a them a ti cs | 17.29

Without expanding the determinants, show that 2 −3


Q.52 Show that A =   satisfies the equation
2 3 4 
1 a bc 1 a a x2 – 6x + 17 = 0.
Q.43 1 b ca = 1 b b2 .
Hence find A–1.
1 c ab 1 c c2

a a2 bc 1 a2 a3 2 4  0 2  1 6 
2 2 3 Q.53 Find matrix A if,  A = .
Q.44 b b ca = 1 b b 1 3 1 3 3 −1
2 2
c c ab 1 c c3
5 0 4  1 3 3 
  −1  
0 p −q p −r Q.54 Given A 2 3 3  and B 1 4 3  . Compute
Q.45 q − p 0 q−r =0 (AB) .
–1
1 2 1  1 3 4 
   
r −p r −q 0
 2 −1 1
 
Q.55 For the matrix A =
 −1 2 −1 , verify that
x2 0 3
 1 −1 2
Q.46 Solve for x, x 1 −4 =
11 .  
1 2 0 A3 − 6A2 + 9A − 4I =
O , hence find A −1 .

3 2 2 1 0
Q.47 If   , verify that A − 4A − I =O =  
2 1 0 1 Exercise 2
0 0
and O =  −1
 and hence find A . Single Correct Choice Type
 0 0 
Q.1 If a, b, c are all different from zero and
1 / a a2 bc
1+a 1 1
Q.48 Evaluate : 1 / b b2 ca .
1 1+b 1 = 0, then the value of a−1 + b−1 + c−1
1/c c2 ab 1 1 1+c

is:
Q.49 Show that {–(a + b + c)} is root of the following
equation: (A) abc (B) a–1 b–1 c–1

x+a b c (C) –a – b – c (D) –1


b x+c a =0.
a a3 a4 − 1
c a x +b
Q.2 If a, b, c are all different and b b3 b 4 − 1 =
0,
then 3 4
c c c −1
Q.50 Using properties of determinants, prove
(A) abc (ab + bc + ca) = a + b + c
x+4 x x
that : x x+4 x =16(3x + 4) . (B) (a + b + c) (ab + bc + ca) = abc
x x x+4 (C) abc (a + b + c) = ab + bc + ca
(D) None of these
Q.51 Using properties of determinants, prove that:
b + c a−b a Q.3 If (sin–1 x + sin–1 w) (sin–1 y + sin–1 z)=p2, then
N1 N2
c + a b − c b= 3abc − a3 − b3 − c3 . x y
a+b c−a c N3 N4
; (N1 ,N2 ,N3 ,N4 ∈ N)
z w
1 7 . 3 0 | Determinants

(A) Has a maximum value 2. Q.9 The following system of equations 3x – 7y + 5z = 3;


3x + y + 5z = 7 and 2x + 3y + 5z = 5 are
(B) Has a minimum value 2.
(A) Consistent with trivial solution
(C) In independent of N1 ,N2 ,N3 ,N4
(B) Consistent with unique non-trivial solution
(D) None of these
(C) Consistent with infinite solution
2 n 2 2n
Q.4 If (1 + x + x ) = a0 + a1 x + a2 x + ...... + a2n x then (D) Inconsistent with no solution
an−3 an−1 an+1
an−6 an−3 an+3 is Q.10 The system of equations (sin θ)x + 2z =0,
an−14 an−7 an+ 7 (cos θ)x + (sin θ)y =,
0 (cos θ)y + 2z =a has

(A) Non unique solution


(A) 1 (B) 2 (C) 0 (D) –1
(B) A unique solution which is a function of a and θ
Q.5 The absolute value of the determinant (C) A unique solution which is independent of a and θ
−1 2 1 (D) A unique solution which is independent of θ only
3 + 2 2 2 + 2 2 1 is
3−2 2 2−2 2 1 Q.11 The equation

(A) 16 2 (B) 8 2 (C) 8 (D) None of these (1 + x)2 (1 − x)2 −(2 + x2 )


2x + 1 3x 1 − 5x +
a b c bc − a2 ac − b2 ab − c2 x +1 2x 2 − 3x
ac b2
Q.6 D1 = b c a , D2 =− ab − c2 bc − a2 ,
c a b (1 + x)2 2x + 1 x +1
ab − c2 bc − a2 ac − b2
3
(1 − x) 3x 2x = 0
a2 + b2 + c2 ab + bc + ca ab + bc + ca 1 − 2x 3x − 2 2x − 3
D3 = ab + bc + ca a2 + b2 + c2 ab + bc + ca then
ab + bc + ca ab + bc + ca a2 + b2 + c2 (A) Has no real solution
(B) Has 4 real solutions
(A) D1 ≤ 0 , if a+b+c>0 (B) D22 = D3
(C) Has two real and two non-real solutions
2
(C) D=
1 D=
2 D12
D3 (D) D2 ≠ D3 =
(D) Has infinite number of solutions, real or non-real

1 1 1 1 1 1 Q.12 The system of equation :


Q.7 a b c = a b c , where a, b, c are distinct
2x cos2 θ + y sin2θ − 2sin θ = 0 ;
bc ca ab a3 b3 c3
x sin2θ + 2y sin2 θ = −2cos θ ;
positive reals, then abc is always less than
x sin θ − y cos θ = 0 , for all values of θ , can
1 1 1 1
(A) (B) (C) (D)
243 729 27 81 (A) Have a unique nontrivial solution
(B) Not have a solution
Q.8 The value of ‘a’ for which the system of equations,
(C) Have infinite solutions
(a + 1)3 x + (a + 2)3 y = (a + 3)3 ,
(D) Have a trivial solution
logx xyz logx y logx z
logy xyz 1 logy z and x + y = 1 are consistent is Q.13 If x, y, z are not all simultaneously equal to zero,
satisfying the system of equations (sin3θ)x − y + z = 0;
logz xyz logz y 1
(cos2θ)x + 4y + 3z = 0; 2x + 7y + 7z = 0, then the
(A) –2 (B) 1 (C) 0 (D) None number of principal values of θ is
(A) 2 (B) 4 (C) 5 (D) 6
M a them a ti cs | 17.31

Q.14 For a non-zero, real a, b and c Q.6 The number of values of k for which the system of
2 2 equations (k + 1)x + 8y = 4k, kx+ (k + 3)y = 3k – 1 has
a +b
c c infinite solutions.
c
b2 + c2
a a = α abc
a Assertion Reasoning Type
2 2
c +a (A) Both assertion and reason are true and reason is the
b b
b
correct explanation of Assertion.
then the value of α is
(B) Both assertion and reason are true and reason is not
(A) –4 (B) 0 (C) 2 (D) 4
the correct explanation of assertion.
Q.15 Number of value of ‘a’ for which the system of (C) Assertion is true but reason is false
equations, a2 x + (2 − a)y = 4 + a2 ; ax + (2a − 1)y =a5 − 2
(D) Assertion is false but reason is true.
possess no solution is
(A) 0 (B) 1 (C) 2 (D) Infinite Q.7 Consider the system of equations x – 2y + 3z = –1,
x – 3y + 4z = 1 and –x + y – 2z = k

Previous Years’ Questions Statement-I: The system of equation has no solution


for k ≠ 3 . and
Q.1 The determinant
1 3 −1
xp + y x y Statement-II: The determinant −1 −2 k ≠ 0 , for
yp + z y z 0 
= (1997) 1 4 1
0 xp + y yp + z k ≠ 0.  (2008)
(A) x, y, z are in AP (B) x, y, z are in GP
Q.8 Given, x = cy + bz, y = az + cx, z = bx + ay, where
(C) x, y, z are in HP (D) xy, yz, zx are in AP
x, y, z are not all zero, prove that a2 + b2 + c2 + 2ab =1
 (1978)
1 x x +1
Q.2 If f(x)
= 2x x(x − 1) (x + 1)x , Q.9 If α be a repeated root of a quadratic equation
3x(x − 1) x(x − 1)(x − 2) (x + 1)x(x − 1) f(x) = 0 and A(x), B(x) and C(x) be polynomials of
degree 3, 4 and 5 respectively, then show that
then f(100) is equal to  (1999)
(A) 0 (B) 1 (C) 100 (D) –100 A(x) B(x) C(x)
A(α ) B(α ) C(α ) is divisible by f(x), where prime
Q.3 If the system of equations x – ky – z = 0, A ′(α ) B′(α ) C′(α )
kx – y – z = 0, x + y – z = 0 has a non-zero solution, then denotes the derivatives.  (1984)
possible values of k are  (2000)
(A) –1, 2 (B) 1, 2 (C) 0, 1 (D) –1, 1 a b c 
 
Q.10 If matrix A = b c a  , where a, b, c are real
Q.4 The number of distinct real roots of  (2001)  c a b
 
sinx cos x cos x positive number, abc = 1 and A T A = I , then find the
π π
cos x sinx cos x = 0 in the interval − ≤ x ≤ is value of a3 + b3 + c3 .  (2003)
4 4
cos x cos x sinx

(A) 0 (B)2 (C) 1 (D) 3 Q.11 The number of values of k , for which the system
of equations: (k + 1 ) x + 8y =
4k kx + (k + 3 ) = 3k − 1 Has
Q.5 If the system of equations x + ay = 0, az + y = 0 and no solution, is:  (2013)
ax + z = 0 has infinite solutions, then the value of a is
(A) Infinite (B) 1 (C) 2 (D) 3
 (2002)
(A) –1 (B) 1
Q.12 If α , β ≠ 0 , and f (n) = α + β
n n
(C) 0 (D) No real values
1 7 . 3 2 | Determinants

(B) Is a singleton
3 1 + f (1 ) 1 + f ( 2 )
1 + g (1 ) 1 + f ( 2 ) 1 + f ( 3= (C) Contains two elements
) K (1 − α ) (1 − β )( α − β ) ,
2 2

1 + f ( 2 ) 1 + g (3) 1 + f ( 4 ) (D) Contains more than two elements

then K is equal to:  (2014) Q.14 The system of linear equations  (2016)
α x + λy − z = 0
(A) αβ (B) (C) 1 (D) -1
β
λx − y − z =0
Q.13 The set of all values of λ for which the system of x + y − λz = 0
linear equations:
has a non-trivial solution for:
2x1 − 2x2 + x3 =
λx1
(A) Infinitely many values of λ
2x1 − 3x2 + 2x3 =
λx 2
(B) Exactly one value of λ
− x1 + 2x2 =λx3
(C) Exactly two values of λ
Has a non − trivial solution.  (2015)
(D) Exactly three values of λ
(A) Is an empty set

JEE Advanced/Boards

Exercise 1 Q.5 Given x = cy + bz; y = az + cx; z = bx + ay where x,


y, z are not all zero, prove that a2 + b2 + c2 + 2abc =
1.
Q.1 Solve the following using Cramer’s rule and state
whether consistent or not. x y z
Q.6 Given a = ; b= ; c= where x, y, z
(a) x + 2y + z =1 (b) x + y + z − 6 =0 y−z z−x x−y
3x + y + z =6 2x + y − z − 1 =0 are not all zero, prove that 1 + ab + bc + ca = 0.
x + 2y = 0 x + y − 2z + 3 =0
Q.7 If sinq ≠ cosq and x, y, z satisfy the equations
(c) 7x − 7y + 5z = 3 x cosp − y sinp +=
z cosq + 1
3x + y + 5z =7
x sinp + y cosp + z = 1 − sinq
2x + 3y + 3z =5
x cos(p + q) − y sin(p + q) + z =
2
Then find the value of x2 + y 2 + z 2 .

Q.2 For what value of K do the following system of Q.8 Investigate for what values of λ , µ the simultaneous
equations possess a non-trivial (i.e. not all zero) solution equations x + y + z = 6; x+ 2y + 3z = 10 and
over the set of rational Q? x + Ky + 3z = 0, 3x + Ky – 2z x + 2y + λz = µ have;
= 0, 2x + 3y – 4z = 0. For that value of K, find all the
(a) A unique solution
solutions of the system.
(b) An infinite number of solutions
Q.3 The system of equations αx + y + z = α − 1 (c) No solution
x + αy + z = α − 1 ; x + y + αz = α − 1 has no solution.
Find α . Q.9 For what values of p, the equations: x + y + z = 1;
x + 2y + 4z = p and x + 4y + 10z = p2 have a solution?
Q.4 If the equations a(y + z) = x, b(z + x) = y, c(x + y) Solve them completely in each case.
= z have non-trivial solutions, then find the value of
1 1 1 Q.10 Solve the equations : Kx + 2y – 2z = 1; 4x + 2Ky
+ + .
1+a 1+b 1+c – z = 2; 6x + 6y + Kz = 3 considering specially the case
M a them a ti cs | 17.33

when K = 2. x+y+z = 4
2x + y + 3z = 6
Q.11 (a) Let a, b, c, d are distinct numbers to be chosen x + 2y + az = b
from the set {1, 2, 3, 4, 5}. If the least possible positive
Let L: denotes number of ordered pairs (a, b) so that the
ax + by =
1
solution for x to the system of equation  system of equations has unique solution,
cx + dy =
2
p
can be expressed in the form where p and q are M: denotes number of ordered pairs (a, b) so that the
q
system of equations has no solution and
relatively prime, then find the value of (p + q).
N: denotes number of ordered pairs (a, b) so that the
(b) Find the sum of all positive integral values of a for system of equations has infinite solutions. Find (L + M – N).
which every solution to the system of equations x + ay
= 3 and ax + 4y = 6 satisfy the inequalities x > 1, y > 0. Q.16 (a) Prove that the value of the determinant
2
Q.12 If the following system of equations (a – t)x + by + −7 5 + 3i − 4i
3
cz = 0, bx + (c – t)y + az = 0 and cx + ay + (b – t)z = 5 − 3i 8 4 + 5i is real.
0 has non-trivial solutions for different values of t, then
2
show that we can express product of these values of + 4i 4 − 5i 9
3
t in the form of determinant.
(b) On which one of the parameter out of a, p, d or x value
Q.13 Show that the system of equations3x – y + 4z = 3,
x + 2y – 3z = –2 and 6x + 5y + λz = –3 has atleast one 1 a a2
solution for any real number λ . Find the set of solutions of the determinant cos(p − d)x cospx cos(p + d)x
of λ = −5 . sin(p − d)x sinpx sin(p + d)x
does not depend.
Q.14 Solve the system of equations:
x3 + 1 x 2 x
0
z + ay + a2 x + a3 =
 3
(c) If y + 1 y 2
y =
0 and x, y, z are all different then,
z + by + b2 x + b3 =
0 3 2
 z +1 z z
z + cy + c2 x + c3 =
0

prove that xyz = –1.
Q.15 (a) Consider the system of equations
αx − y + z =α a2 + 2a 2a + 1 1
x − αy + z =1 Q.17 Prove that (a) 2a + 1 a + 2 1 = (a − 1)3
x − y + αz =1 3 3 1

If L, M and N denotes the number of integral values 1 1 1


of α in interval [–10, 10] for which the system of the (b) x y z =[(x − y)(y − z)(z − x)(x + y + z)]
equations has unique solution, no solution and infinite
x3 y3 z3
solutions respectively, then find the value of (L – M + N).
(b) If the system of equations is −3
x 1
2x + 3y − z = 0 2
3x + 2y + kz = 0 Q.18 (a) Let f(x) = 2 2 1 Find the minimum
4x + y + z = 0
1 1
0
x −1 2
have a set of non-zero integral solutions then, find the
smallest positive value of z. value of f(x) (given x > 1).

(c) Given a, b ∈ {0, 1, 2, 3, 4,……, 9, 10}.


(b) If a2 + b2 + c2 + ab + bc + ca ≤ 0 ∀ a, b, c ∈ R, then
Consider the system of equations find the value of the determinant
1 7 . 3 4 | Determinants

(a + b + 2)2 a2 + b2 1 x − 2 2x − 3 3x − 4
1 (b + c + 2)2 b2 + c2 (b) x − 4 2x − 9 3x − 16 =0
x − 8 2x − 27 3x − 64
c2 + a2 1 (c + a + 2)2

a b c Q.25 If a + b + c = 0, solve for x :


Q.19 If D = c a b and a−x c b
b c a c b−x a =0
b+c c+a a+b b a c−x
D′ =+
a b b + c c + a then prove that D’ = 2D.
c+a a+b b+c Q.26 Let a, b, c are the solutions of the cubic
x3 − 5x2 + 3x − 1 =0 , then find the value of the
Q.20 Prove that a b c
determinant a − b b − c c − a
1 + a2 − b2 2ab −2b
2 2 b+c c+a a+b
2ab 1−a +b 2a = (1 + a2 + b2 )3
2b −2a 1 − a2 − b2
a2 + λ ab ac
Q.27 Show that, ab 2
b λ bc is divisible by λ2
Q.21 Let
ac bc c2 + λ
sinx sin(x + h) sin(x + 2h) and find the other factor.
f(x) =
sin(x + 2h) sinx sin(x + h) .
sin(x + h) sin(x + 2h) sinx Q.28 Prove that

f(x) a2 b2 c2 a2 b2 c2
If Lim has the value equation to k(sin3x + sin x) 3
h→0 h2 (a + 1)2 (b + 1)2 (c + 1)2 =
4 a b c
find k ∈ N. (a − 1)2 (b − 1)2 (c − 1)2 1 1 1

Q.22 Prove that Q.29 In a ∆ABC , determine condition under which


(β + γ − α − δ)4 (β + γ − α − δ)2 1 A B C
cos cot cot
2 2 2
( γ + α + β − δ )4 ( γ + α − β − δ)2 1
B C C A A B
( α + β − γ − δ )4 (α + β − γ − δ)2 1 tan + tan tan + tan tan + tan =0
2 2 2 2 2 2
1 1 1
= −64(α − β)(α − γ )(α − δ)(β − γ )(β − δ)( γ − δ)

Q.23 If a, b and c are the roots of the cubic x3 − 3x2 + 2 =0 Exercise 2


then find the value of the determinant.
Single Correct Choice Type
(b + c)2 a2 a2
b2 (c + a)2 b2 Q.1 Let m be a positive integer &
c2 c2 (a + b)2 m
2r − 1 Cr 1
2 m
Dr = m −1 2 m+1 (0 ≤ t ≤ m) ,
Q.24 Solve for x 2 2 2 2
sin (m ) sin (m) sin (m + 1)
x + 2 2x + 3 3x + 4 m

(a) 2x + 3 3x + 4 4x + 5 = 0
then the value of ∑ Dr is given by
r =0
3x + 5 5x + 8 10x + 17 (A) 0 2
(B) m − 1 (C) 2m (D) 2m sin2 (2m )
M a them a ti cs | 17.35

Q.2 If α,β and γ are real numbers, then then the maximum value of f(x) is equal to

1 cos(β − α ) cos( γ − α ) (A) 2 (B) 4 (C) 6 (D) 8


D cos(α − β) cos(β − γ ) cos( γ −=
= β)
cos(α − γ ) 1 1 Q.8 If px 4 + qx3 + rx 2 + sx + t

(A) –1 (B) cos α cos β cos γ x2 + 3x x − 1 x + 3


x + 1 2 − x x − 3 then t is equal to
(C) cos α + cos β + cos γ (D) Zero
x − 3 x + 4 3x

Q.3 If a, b and c are non-zero real numbers, then (A) 33 (B) 0 (C) 21 (D) None
2 2
b c bc b + c
=D c a 2 2
ca
= c+a 0 a2 + 1 ab ac
2
ab 2 2
ab a + b Q.9 If D = ba b +1 bc , then D is equal to
2
ca cb c +1
(A) abc (B) a2b2c2
(C) bc + ca + ab (D) Zero (A) 1 + a2 + b2 + c2 (B) a2 + b2 + c2

(C) (a + b + c)2 (D) None of these


mx mx − p mx + p
Q.4 If f ′(x)
= n n+p n−p , Q.10 If α + β + γ = π , then the value of
mx + 2n mx + 2n + p mx + 2n − p
sin(α + β + γ ) sin β cos γ
then y=f(x) represents − sin β 0 tan α is
cos(α + β) − tan α 0
(A) A straight line parallel to x-axis
(B) A straight line parallel to y-axis (A) 0 (B) 1
(C) Parabola (C) 2 (D) 2.sin β. cos γ. tan α
(D) A straight line with negative slope
Q.11 If the entries of 3 × 3 determinant are zero or one
then the value of the determinant
x − 1 (x − 1)2 x3
(A) Cannot be 3 (B) Cannot be 2
Q.5 If D(x) =
x −1 x2 (x + 1)3 ,
(C) Can be –2 (D) Is essentially zero
x (x + 1)2 (x + 1)3

then the coefficient of x in D(x) is Q.12 In a third order determinant, each element of
the first column consists of sum of two terms, each
(A) 5 (B) –2 (C) 6 (D) 0 element of the second column consists of sum of three
terms and each element of the third column consists of
Q.6 The number of integral solutions of |D|=8, where sum of four terms. Then it can be decomposed into n
determinants, where n has the value
y+z z y
=D z z+x x is (A) 1 (B) 9 (C) 16 (D) 24
y x x+y
Q.13 If the system of equations x + 2y + 3z = 4, x +
(A) 3 (B) 8 (C) 16 (D) 24 py + 2z =3, µx + 4 y + z = 3 has an infinite number of
solutions, then
1 + sin2 x cos2 x 4 sin2x (A) p = 2, µ =3 (B) p = 2, µ =4
2 2
Q.7
= Let f(x) sin x 1 + cos x 4 sin2x , (C) 3p = 2µ (D) None of these
2 2
sin x cos x 1 + 4 sin2x
1 7 . 3 6 | Determinants

Q.14 Number of triplets of a, b and c for which the Q.20 The set of equations λx − y + (cos θ)z =0 ;
system of equations, ax – by = 2a – b and (c + 1)x + cy 3x + y + 2z =;0 (cos θ)x + y + 2z =,
0 0 ≤ θ < 2π , has
= 10 – a + 3b has infinitely many solutions and x = 1, nontrivial solution(s)
y = 3 is one of the solutions, is
(A) For no value of λ and θ
(A) Exactly one (B) Exactly two
(B) For all values of λ and θ
(C) Exactly three (D) Infinitely many
(C) For all values of λ and only two values of θ

Q.15 If the system of equations ax + y + z = 0, (D) For only one value of λ and all values of θ
x + by + z = 0 & x + y + cz = 0 (a , b, c ≠ 1) has a non-
1 1 1 Multiple Correct Choice Type
trival solution , then the value of + + is
1−a 1−b 1−c
(A) –1 (B) 0 (C) 1 (D) None of these Q.21 The determinant
cos(x − y) cos(y − z) cos(z − x)
Q.16 The determinant cos(x + y) cos(y + z) cos(z + x) =
cos(θ + φ) − sin(θ + φ) cos2φ sin(x + y) sin(y + z) sin(z + x)
sin θ cos θ sin φ is (A) 2sin(x − y)sin(y − z)sin(z − x)
− cos θ sin θ cos φ
(B) −2sin(x − y)sin(y − z)sin(z − x)
(A) 0 (C) 2cos(x − y)cos(y − z)cos(z − x)
(B) Independent of θ (D) −2cos(x − y)cos(y − z)cos(z − x)
(C) Independent of φ
π π
(D) Independent of θ and φ both Q.22 The value of θ lying between − and and
4 2
π
−0 ≤ A ≤ and satisfying the equation
Q.17 The values θ , λ for which the following equations 2
x sin θ − y cos θ + (λ + 1)z = 0 ; x cos θ + y sin θ − λz = 0 ; 1 + sin2 A cos2 A 2sin 4θ
λx + (λ + 1)y + z cos θ =0 are consistent with infinite 2
sin A 1 + cos A 2
2sin 4θ are
solution, are 2 2
sin A cos A 1 + 2sin 4θ
(A) θ = nπ, λ ∈, λ ∈ R − {0}
π π 3π
(B) θ= 2nπ, λ is any rational number (A) A = , θ = − (B) A = = θ
4 8 8
θ (2n + 1)π, λ ∈ R + , n ∈ I
(C) = π π π 3π
(C) A = 9 θ = − (D) A = = θ =
π 5 8 6 8
(D) =
θ (2n + 1) , λ ∈ R, n ∈ I
2
1 a a2
Q.18 If the system of equations, a2 x − ay =1 − a and Q.23 If 1 x x2 = 0
bx + (3 − 2b)y =3 + a possess a unique solution x = 1, b2 ab a2
y = 1 then 1 a
(A) x = a (B) x = b (C) x = (D) x =
(A) a = 1; b = –1 (B) a = –1, b = 1 a b
(C) a = 0, b = 0 (D) None of these
a b aα + b
Q.24 The determinant b c bα + c
n+ 2 n+ 3 n+ 4
Cn Cn+1 Cn+ 2 aα + b bα + c 0
n+ 3 n+ 4 n+5
Q.19 Let D = Cn+1 Cn+ 2 Cn+3 and n ∈ N is equal to zero, if
n+ 4 n+5 n+ 6
Cn+ 2 Cn+3 Cn+ 6 (A) a, b, c are in AP
then the value of D is equal to (B) a, b, c are in GP
(A) –1 (B) 0 (C) 1 (C) α is a root of the equation ax2 + bx + c =0
(D) (n + 2) (n + 3) (n + 4) (n + 5) (n + 6) (D) (x − α ) is a factor of ax2 + 2bx + c
M a them a ti cs | 17.37

Q.25 The set of equations x − y + 3z =,


2 2x − y + z =,
4 (A) D is independent of x (B) D is independent of y
x − 2y + αz =3 has (C) D is independent of z (D) D is dependent of x, y, z
(A) Unique solution only for α =0
(B) Unique solution for α ≠ 8 Previous Years’ Questions
(C) Infinite number of solution of α =8
(D) No solution for α =8 Q.1 The parameter, on which the value of the
determinant
Q.26 Which of the following determinant(s) vanish(es)? 1 a a2
cos(p − d)x cospx cos(p + d)x does not depend
1 1
+ 1 ab sin(p − d)x sinpx sin(p + d)x
1 bc bc(b + c) a b
1 1 upon, is  (1997)
(A) 1 ca ca(c + a) (B) 1 bc +
b c (A) a (B) p (C) d (D) x
1 ab ab(a + b)
1 1
1 ca +
c a
Q.2 Let λ and α be real. Find the set of all values
0 a−b a−c logx xyz logx y logx z of λ for which the system of linear equations
λx + (sin α )y + (cos α )z =0
(C) b − a 0 b−c (D) logy xyz 1 logy z
c −a c −b 0 logz xyz logz y 1 x + (cos α )y + (sin α )z =and
0

−x + (sin α )y − (cos α )z =0 has a non-trivial solution.
Q.27 If the system of equation a2 x − by = a2 − b For λ =1 , find all value of α .  (1993)
and bx − b2 y =+2 4b possess an infinite number of
solutions then the possible values of ‘a’ and ‘b’ are Q.3 Let a, b, c be real numbers with a2 + b2 + c2 =
1.
(A) a = 1, b = –1 (B) a = 1, b = –2 Show that the equation
(C) a = –1, b = –1 (D) a = –1, b = –2 ax − by − c bx + ay cx + a
bx + ay −ax + by − c cy + b 0
=
Q.28 If p, q, r, s are in A.P, and cx + a cy + b −ax − by + c

p + sinx q + sinx p − r + sinx represents a straight line.


f(x) = q + sinx r + sinx −1 + sinx such that
r + sinx s + sinx s − q + sinx Q.4 For what value of k does the following system of
2
equations possess a non-trivial solution over the set of
rationals x + y – 2z = 0; 2x – 3y + z = 0 and x – 5y +
∫ f(x)dx = −4 , then the common difference of the A.P.
4z = k. Find all the solution.  (1979)
0
can be
Q.5 For what value of m does the system of equations
(A) –1 (B) ½ (C) 1 (D) None of these
3x + my = m and 2x – 5y = 20 has a solution satisfying
the conditions x > 0, y > 0. (1979)
Q.29 If the system of equations x+ y – 3 =0, (1 + K)x +
(2 + K)y – 8 = 0 and x – (1 + K) y + (2 + K) are consistent
Q.6 Prove that for all values of θ
then the value of K is
(A) 1 (B) 3/5 (C) –5/3 (D) 2
sin θ cos θ sin2θ

1 1 (x + y)
 2π   2π   4π 
− sin  θ +  cos  θ +  sin  2θ +  = 0  (2000)
z z  3   3   3 
z2
(y + z) 1 1  2π   2π   4π 
Q.30 If D = − then sin  θ −  cos  θ −  sin  2θ − 
x 2 x x  3   3   3 
y(y + z) x + 2y + z y(x + y)
− − −
2
x z xz xz 2
1 7 . 3 8 | Determinants

Q.7 The total number of ways in which 5 balls of Q.9 The total number of distinct x ∈ R for
different colours can be distributed among 5 persons
x x2 1 + x3
so that each person gets at least one ball is  (2012)
which 2x 4x2 1 + 8x3 =
10 is  (2016)
(A) 75 (B) 150 (C) 120 (D) 243 2 3
3x 9x 1 = 27x

Q.8 Which of the following values of α satisfy the


equation

(1 + α ) (1 + 2α ) (1 + 3α )
2 2 2

( 2 + α ) ( 2 + 2α ) ( 2 + 3α )
2 2 2
−648α ?
= (2015)

(3 + α ) (3 + 2α ) (3 + 3α )
2 2 2


(A) −4 (B) 9 (C) −9 (D) 4

PlancEssential Questions
JEE Main/Boards JEE Advanced/Boards
Exercise 1 Exercise 1
Q.15 Q.29 Q.38 Q.1 Q.3 Q.8
Q.48 Q.52 Q.54 Q.11 Q.15 Q.23
Q.55
Exercise 2
Exercise 2 Q.1 Q.11 Q.14
Q.1 Q.6 Q.13 Q.20 Q.25 Q.30

Previous Years’ Questions Previous Years’ Questions


Q.2 Q.6 Q.7
Q.2 Q.5 Q.7
Q.10
Q.10
M a them a ti cs | 17.39

Answer Key

JEE Main/Boards
Exercise 1

Single Correct Choice Type

7 13 5
Q.1 x = ±2 2 Q.2 1 Q.3 (i) 1 (ii) –16 Q.4 − , Q.5 sq. units
2 2 2
5
Q.6 0 Q.7 –96 Q.8 1; 9 Q.9 Q.10 –400
3
 4 −1 5 19
Q.11   Q.12 Not possible Q.13 0 Q.14 x = Q.15 sq. units
5 3  8 2

Q.16 -2 Q.20 75 Q.21 –10 Q.23 16 Q.24 No

−1 1 2 3 
Q.25 1728 Q.26 A =   Q.28 Inconsistent
19 5 −2 
 1 + bc 
−b 
Q.29  a Q.39 M21=39, M22=3, M23=−11, C21=−39, C22=3, C23=11
 
 −c a 

1 2 3
Q.40 f(x) = x + x+6 Q.41 Consistent Q.42 Inconsistent
2 2

7  −1 2 
Q.46 x = − or 1 Q.47 A −1 =   Q.48 0
4  2 −3

 3 −19 12 
1  4 3 1  49 −18  −1  
Q.52 A −1 Q.53  Q.54 4 −18 5 
=   
4  −23 10  11 
17  −3 2   4 −29 27 
 
 3 1 −1
1 
Q. 55  1 3 1 
4
 −1 1 3
 

Exercise 2

Single Correct Choice Type

Q.1 D Q.2 A Q.3 A Q.4 C Q.5 A Q.6 C

Q.7 C Q.8 A Q.9 B Q.10 B Q.11 D Q.12 B

Q.13 C Q.14 D Q.15 C


1 7 . 4 0 | Determinants

Previous Years’ Questions


Q.1 B Q.2 A Q.3 D Q.4 C Q.5 A Q.6 1

Q.7 A Q.10 4 Q.11 B Q.12 C Q.13 C Q.14 D

JEE Advanced/Boards

Exercise 1

Q.1 (a) x = 1, y = 2, z = 3; consistent (b) x = 2, y = –1, z = 1; consistent (c) Inconsistent

33 15
Q.2 K = ,x:y:z=
− : 1 : −3 Q.3 –2 Q.4 2 Q.7 2
2 2

Q.8 (a) λ ≠ 3 (b) λ =3 , µ =10 (c) λ= 3, µ ≠ 10

Q.9 x = 1 + 2K, y = –3K, z = K, when p = 1; x = 2K, y = 1 – 3K, z = K when p = 2; where K ∈ R

x y z 1 1 − 2λ
Q.10 If K ≠ 2, = = = , If K = 2, then x =λ, y = and z = 0 where λ ∈ R
2(K + 6) 2K + 3 6(K − 2) 2(K 2 + 2K + 15) 2

a b c
Q.11 (a) 19 (b) 4 Q.12 b c a
c a b

4 9 4 − 5K 13K − 9
Q.13 If λ ≠ −5 , then x = ; y = − and z = 0; If λ =5=
then x = ;y and z = K where K ∈ R .
7 7 7 7
Q.14 x = –(a + b + c), y = ab + bc + ca, z = – abc

Q.15 (a) 21 (b) 5 (c) 119 Q.16 (b) p Q.18 (a) 4 (b) 65

Q.21 3 Q.23 –108 Q.24 (a) x = –1 or x = –2; (b) x = 4

3 2
Q.25 X = 0 or x ± (a + b2 + c2 ) Q.26 80
2

Q.27 λ2 (a2 + b2 + c2 + λ ) Q.29 Triangle ABC is isosceles.

Exercise 2

Single Correct Choice Type

Q.1 A Q.2 D Q.3 A Q.4 A Q.5 A Q.6 D

Q.7 C Q.8 C Q.9 A Q.10 A Q.11 C Q.12 D

Q.13 D Q.14 B Q.15 C Q.16 B Q.17 D Q.18 A

Q.19 A Q.20 A
M a them a ti cs | 17.41

Multiple Correct Choice Type

Q.21 A,D Q.22 A, B, C, D Q.23 A, D Q.24 B, D Q.25 B, D Q.26 A, B, C, D

Q.27 A, B, C, D Q.28 A, C Q.29 A, C Q.30 A, B, C

Previous Years’ Questions


Q.1 B Q.2 Zero Q.4 k = 0, the given system has infinitely many solutions
15
Q.5 m < − or m > 30 Q.7 B Q.8 B C Q.9 2
2

Solutions

JEE Main/Boards ⇒ |–2k + 3| = 10

⇒ –2k + 3 = 10 or 2k – 3 = 10
Exercise 1 7 13
⇒k= – or k =
2 2
–1 2 2 x
Sol 1: =
4 8 x –4 Sol 5: Vertices of triangle (0, 3) (–1, 4) (2, 6)

–8 – 8 = 2(–4) –x2 = – 8 – x2 1 0 3
1 1
Area = 1 −1 4 =  –1(6) − 8  + 3[2 + 1]
⇒ x = ± 8 = ±2 2 2 2
1 2 6
1 2 1 1 5
Sol 2: A =   , |A| = 1[1] – 2(0) = 1 = –14 + 9 = 5 = Sq. Unit
0 1  2 2 2

4 −1 0 a11 a12 a13


Sol 3: 2 1 4 , Sol 6: D ⇒ a21 a22 a23
1 0 3 a31 a32 a33

4 −1 a11C21 + a12C22 + a13C23


(i) M23 = = 0 – (–1) = 1
1 0
a13 a12 a11 a13 a a
= a11 + a12 + a13 11 12 (–1)2+3
C23 = (–1) 2+3
= –1 a33 a32 a31 a33 a31 a32

4 0 = a11[a13a32 – a33a12] + a12[a11a33 – a31a13] – a13[a11a32 – a31a12]


(ii) C32 = (–1)2+3 = = –16
2 4
=0
It can be directly said as it is a property
Sol 4: Area of triangle, [(k, 0), (1, 1), (0, 3)] = 5 unit2

1 k 0 –1 24
1 1 Sol 7: = A (assume)
⇒ 1 1 1 = 1[3 – 0] + k(1 − 3) = 5 4 P
2 2
1 0 3
1 7 . 4 2 | Determinants

–1 24 So |a| = 0
|A| = = –P –4(24) = – (96 + P) = 0
4 P ⇒ x[–8] + 1{2(2) –(–1)(1)}] = –8x + 4 + 1 = 0

⇒ P = –96 5
⇒ 8x = 5 → x =
8

1 0  1 0  Sol 15: Vertices → (3, 1) (4, 3) and (–5, 4)


Sol 8: I2 =   , |I2| =   =1
0 1  0 1 
1 3 1
1
1 0  3 0  Area = 1 4 3
3I2 = 3   =   , 2
0 1  0 3  1 −5 4
1
3 0  = [16 + 15 + 3 [3 – 4] + 1 [–5 – 4]]
|3I2| =   =9 2
0 3  1 19
= [31 – 3 – 9] = sq. unit
2 2
Sol 9: (0, 2), (1, x) and (3, 1) points are collinear
Sol 16: Vertices (x, y) (2, –6), (5, 4)
1 0 2
So 1 1 x = 0 1 x 4
1 1
1 3 1 Area = 1 2 −6 = [8 + 30 + x[–6 – 4] + 4[5 – 2]]
2 2
1 5 4
⇒ [1 – 3x] + 2[3 – 1] = 0 ⇒ 1 – 3x + 4 = 0
5 38 – 10x + 12 = 70 ⇒ 10x = 50 – 70 = –20
⇒ 3x = 5 ⇒ x =
3 –20
⇒x= = –2
10
Sol 10: |A| = 20, |B| = – 20

|AB| = |A| |B| = 20(–20) = –400 5 15 −25 5 5 −25


Sol 17: 7 21 30 = (3) 7 7 30
 3 1 8 24 42 8 8 42
Sol 11: A =  
 –5 4  Two column are same so Determinants is 0
C11 = 4, C12 = 5, C21 = –1, C22 = 3 1 a b+c
T
C C12   4 –1 Sol18: 1 b c + a C2 → C2 + C3
adjA =  11  =   1 c a+b
C21 C22  5 3 
1 a+b+c b+c
1 3  ⇒ 1 a+b+c c+a
Sol 12: A =   1 a+b+c a+b
 –6 –18 
1 1 b+c
|A| = – 18 – [3] [–6] = –18 + 18 = 0
= (a+b+c) 1 1 c + a C1 → C2 – C1
So, A–1 does not exist
1 1 a+b
0 1 b+c
3 1 8 3 1 1
= (a + b + c) 0 1 a + c =0
Sol 13: −4 2 16 = (8) −4 2 2 0 1 a+b
−5 3 24 −5 3 3

Two columns are same, so determinant is 0 Sol 19: (2, 0), (0, 5) and (x, y) are collinear

1 2 0
x 0 1
  ⇒ 1 0 5 =0
Sol 14: a =  2 −1 4  is singular 1 x y
1 2 0 
 
M a them a ti cs | 17.43

1[–5x] + 2[5 – y] = 0 C11 = 15 – 1 = 14


–5x + 10 –2y = 0 C12 = 1 + 10 = 11
x C13 = –2 – 3 = –5,
5x + 2y = 10 → + y - 5= 1
2
C21 = 1 + 10 = 11
Sol 20: |A| = 3, A’s order → 2 × 2 C22 = 5 –1 = 4
|5A| = (5)2 |A| = 25 × 3 = 75 C23 = –2 – 1 = –3
C31 = –2 – 3 = –5,
a a12  C32 = –2 – 1 = –3
Sol 21: A =  11  , |A| = –10
a21 a22 
C33 = 3 – 4 = –1
|A| = a11C11 + a12C12 (along first row) = |A| = –10
|A| = 1[14] – 2[11] + 1[–5]
= 14 – 5 – 22 = –13
1 a b+c 1 a b+c
R 2 −R1 , R3 −R1
Sol 22: 1 b a + c  → 0 b −a a−b 14 11 –5 
 
1 c a+b 0 c−a a−c AdjA = 11 4 –3 ,
 –5 –3 –1 
 
1 a b+c 1
A–1 = adjA
=( a − b )( a − c ) 0 −1 1 |A|
0 −1 1
14 11 −5
1  
A =
–1
 11 4 −3 ,
Since the columns are linearly dependent, hence the –13
 −5 −3 −1 
value of determinant is zero.  
1 1
|A–1| = =
Sol 23: |A| = –4 |A| –13

Order of A = 3 For adjA,


|adjA| = |A| 3–1
= (–4) = 16
2
4 −3
C11 = = –4 – 9 = –13,
−3 −1
 2 1 5
Sol 24:   C12 = 15 + 11 = 26
 −1 0 3
C13 = –33 + 20 = –13,
It is not a square matrix, so inverse not exist
C21 = 15 + 11 = 26,
Sol 25: |A| = 12, A’s order 3 × 3 C22 = –14 – 25 = –39
|A. adjA| = |A| |A| 3–1
= |12| = 1728
3
C21 = C12, C31 = C13,
C23 = –55 + 42 = –13,
2 3  –1 1
Sol 26: A =  ,A = A C38 = 56 – 121 = –65
5 –2 19
 −13 26 −13
C11 = –2, C12 = –5, C21 = –3, C22 = 2 1  
So |A | =
–1 –1
 26 −39 −13
adjA 1  −2 −3 1 2 3  1 | A –1 | 
A–1 = =   =   = A  −13 −13 −65 
|A| (–4 – 15)  −5 2  19 5 –2 19
 −13 26 −13  1 –2 1
1    
 1 −2 1 =  26 −39 −13 =  –2 3 1 = A
  –13
Sol 27: A =  −2 3 1   −13 −13 −65  1 1 5
   
 1 1 5
 
1 7 . 4 4 | Determinants

Sol 28: 2x + 5y = 7, 6x + 15y = 13


 1 – tanx   cos2 x – sinx cos x 
AA = 
1 –1
 
2 5 tanx 1  sinx cos x cos2 x 
D= = 30 – 30 = 0
6 15
 cos2 x − sin2 x – sinx cos x − sinx cos x 
D = 0. So system is inconsistent =  
sinx cos x + sinx cos x – sin2 x + cos2 x 

a b cos2x – sin2x 
Sol 29: A = =   = R.H.S.
c (1 + bc) / a  sin2x cos2x 

 1 + bc 
|A| = a   – bc = 1 + bc – bc = 1 2 0 –1
 a   
Sol 31: A = 5 1 0  ,
(1 + bc) / a –b  0 1 3 
adjA =    
 –c a
Assume A – xI = 0  ….. (i)
adjA (1 + bc) / a –b  |A1 – xI| = 0
A–1 = = 
|A|  –c a
2 − x 0 –1 
(a2 + bc + 1)I – aA  
So  5 1–x 0  =0
a2 + bc + 1 0  a b   0 1 3 − x 

=  – a 
c (1 + bc) / a
2
 0 a + bc + 1 ⇒ (2 – x) [x2 + 3 – 4x] –1[5] = 0

a2 + bc + 1 − a2 –ab  ⇒ –x3 – 3x + 4x2 + 2x2 + 6 – 8x = 5


=  
 –ac 2
a + bc + 1 – (1 + bc) ⇒ x3 – 6x2 + 11x = 1
1
⇒ x2 – 6x + 11 = = x–1
(1 + bc) / a –b  x
= a  = aA
–1
(A – xI) = 0
 –c a 
⇒ A2 – 6A + 11I = A–1
R.H.L. = L.H.S.
 2 2 1 1 3 2 
 1 tanx     
Sol 30: A =  Sol 32: A =  −2 1 2  = 1 1 1 
 ,
 – tanx 1   1 −2 2 
 
2 −3 −1
 
 1 – tanx   2 + 2 + 2 6 + 2 −3 4 + 2 −1 
A1 =    
tanx 1  AB =  −2 + 1 + 4 –6 + 1 − 6 –4 + 1 – 2
 1−2+ 4 3 − 2 − 6 2 − 2 − 2 
1 
|A| = 1 + tan2x = 6 5 5
cos2 x  
= 3 −11 −5
 1 – tanx  3 −5 −2 
adjA =    
 tanx 1 
For A
 sinx  C11 = 2 + 4 = 6,
adjA  1 – 
A–1 = cos2 x
= cos x 
C12 = 2 + 4 = 6,
|A|  sinx
 cos x 1  C13 = 4 – 1 = 3,

 cos2 x – sinx cos x  C21 = –2 – 4 = –6,
=  
sinx cos x cos2 x  C22 = 4 – 1 = 3,
C23 = 2 + 4 = 6,
M a them a ti cs | 17.45

C31 = 4 – 1 = 3, C22 = –12 – 15 = –27


C32 = –2 – 4 = –6, C23 = 30 + 15 = 45
C33 = 2 + 4 = 6 C31 = –25 + 55 = 30
|A| = 2(6) + 2(6) + 1(3) = 27 C32 = –30 + 15 = 15,

6 −6 3  C33 = –66 – 15 = –81


adjA 1  
A
= –1
= 6 3 −6   –3 –15 30 
| A | 27   
3 6 6 
  So (AB) =  –9 −27 15 
–1

2 −2 1  18 45 –81
 
1 
= 2 1 −2  −1 −5 10 
9 1 
1 2 2 
  =  −3 −9 15  = B–1A–1
9
For B  −30 3 9 
 
C11 = –1 + 3 = 2 ,
a b c
C12 = 2 + 1 = 3, Sol 33: b c a
C13 = –3 –2 = –5, c a b
C21 = –6 + 3 = –3, = (a+b+c) (ab + bc + ca – a2 – b2 – c2)
C22 = –1 – 4 = –5, = 3abc – a3 – b3 – c3
C23 = –3 – 6 = –9, C1 → C1 + C2 + C3
C31 = 3 – 2 = 1,
a+b+c b c 1 b c
C32 = 2 – 1 = 1, = a + b + c c a = (a+b+c) 1 c a
C33 = 1 – 3 = –2 a+b+c a b 1 a b
|B| = 1[2] + 3[3] + 2[–5] = 11 – 10 = 1 R1 → R1 – R3, R2 → R2 – R3
 2 −3 1  0 b −c c −b
adjB  
B = =  3 −5 1 
–1
(a+b+c) 0 c − a a − b
|B |
 −5 −9 −2
  1 a b
 4 −6 +1 –4 − 3 + 2 2 + 6 + 2  = (a+b+c)[(b–a) (a–b) – (c–a) (c–b)]
1  
B A =
–1 –1
 6 − 10 + 11 –6 − 5 + 2 3 + 10 + 2  = (a+b+c) (ab + bc + ca – a2 – b2 – c2)
9
 –10 − 18 − 2 16 − 3 − 4 –5 + 18 − 4 
  = a2b – a2b + ....... + 3abc – a3 – b3 – c3
 −1 −5 10  = 3abc – a3 – b3 – c3
1 
=  −3 −9 15 
9
 −30 3 9  y+z x y
 
6 5 5 Sol 34: z + x z x = (x + y + z) (x – z)2
  x+y y z
AB = 3 −11 −5 ,
3 −5 −2  R → R1 + R2 + R3
 
|AB| = 6(–3) + 5(–9) + 5(18) = –18 – 45 + 90 = +27 2(x + y + z) x + y + z x + y + z
C11 = 22 – 25 = –3 z+x z x
x+y y z
C12 = –9
C13 = –15 + 33 = 18 2 1 1
= (x+y+z) z + x z x
C21 = –25 + 10 = –15
x+y y z
1 7 . 4 6 | Determinants

C1 → C1 – C2 – C3 R1 → R1 + R2

2 −1 −1 1 1 = (c – a)(b – c)
= (x+y+z) z + x − z − x z x a + 2b + ( −2a − b + c) −a − c + b + c b − a
x+y−y−z y z −(2a + b + c) b+c −a
0 1 1 (a + b)2 c2 ab
= (x+y+z) 0 z x
b–a b−a b−a
x−z y z
= (c–a)(b–c) −2a − b − c b + c −a
= (x + y + z) (x – z) (x – z)
(a + b)2 c2 ab

–bc b2 + bc c2 + bc 0 1 1
Sol 35: a2 + ac –ac c2 + ac = (ab + b + ca)3 0 b + c −a
a2 + ab b2 + ab −ab a2 + b2 + c2 c2 ab

C1 → C1 + C3, C2 → C2 + C3 C1 → C1 + C2 – 2C3
= (c–a)(b–c)(b–c)
c2 (b + c)2 c2 + bc
(a + c)2 c2 c2 + ac z−z 1 1
a2
b 2
−ab –2a − b − c + b + c + 2a b + c –a
a2 + b2 + 2ab + c2 − 2ab c2 ab
= c2[–abc2 – b2(c2 + ac)] +
= –(a–b)(b–c)(c–a)
(b+c)2 [a2(c2+ac) + ab(a+c)2]
z−z 1 1
(c2 + b) [b2(a + c)2 – c2a2]
–2a − b − c + b + c + 2a b + c –a
= [–abc4 – c4b2 – c3b2a + (b+c)2 (a2c2 + a3c + a3b + abc2 a2 + b2 + 2ab + c2 − 2ab c2 ab
+ 2a2bc] + (c2 + bc)[b2a2 + b2c2 + 2acb2 – c2a2]
This on simplification comes out to be equal to = –(a–b)(b–c) (c–a) (a2+b2+c2)(–a–b–c)

(ab + bc + ca)3 = (a–b)(b–c)(c–a)(a2+b2+c2)(a+b+c)

(b + c)2 a2 bc a b−c c+b


2 2
Sol 36: (c + a) b ca = (a–b) (b–c) (c–a) Sol 37: a + c b c−a
2 2
(a + b) c ab a−b a+b c

R1 → R1 – R3, R2 → R2 – R3 = (a b + c) (a2 + b2 + c2)

(b − c)2 − (a + b)2 a2 − c2 bc − ab a b−c c+b 0 b−c c+b


= a b c−a + c b c−a
= (c + a)2 − (a + b)2 b2 − c2 ac − ab
2 2 a a+b c –b a + b c
(a + b) c ab
1 b−c c+b 0 1 c+b
(c − a)[c + a + 2b] (c − a)(–a − c) b(c − a)
=a 1 b c−a + c 1 c−a
= (b − c)[–c − b − 2a] (b − c)(b + c) –a(b + c)
1 a+b c –b b c
(a + b)2 c2 ab
0 –c c + b
a + 2b + c −a − c b + c 0 c−a
= (c – a) (b – c) −(2a + b + c) b + c −a –b a c
(a + b)2 c2 ab
M a them a ti cs | 17.47

1 b−c c+b 0 1 c+b 1 3


M22 = =9–6=3
=a 1 b c − a +b c 1 c − a 2 9
1 a+b c –b 1 c
1 2
M23 = = –7 – 4 = –11
0 –c b 0 –c c 2 –7
+ c 0 –a + c 0 c C21 = –39, C22 = 3, C23 = 11
–b a 0 –b a c
Sol 40: f(x) = ax2 + bx + c, f(0) = 6
Using, C2 → C2 – C1 and C3 → C3-C1
f(2) = 11, f(–3) = 6
in (i), (ii), (iii)
 02 0 1  a  6
    
1 b−c c+b 0 1 c+b 4 2 1 b  = 11 
a 0 c −a − b + c 0 −a − b 9 –3 1  c  6
   
0 a+c −b –b 0 −b
D = –12 – 18 = –30
0 –c 1  6 0 1
+c c c 0  
Da= 11 2 1
–b a + c 0  6 −3 1
 
= a2[a + b + c] + b2[a + b + c] + c2[a + b + c] = 6[2+3]+1[–33 – 12] = 30 – 33 – 12 = –15
= (a2 + b2 + c2) (a + b + c) Da –15 1
a= = =
D –30 2
a b ax + by 0 6 1
Sol 38: b c bx + cy Db= 4 11 1
ax + by bx + cy 0 9 6 1
R3 → R3 – xR1 – yR2 = 6[9 – 4] + 1[24 – 99] = 30 + 24 – 99 = –45

a b ax + by –45 3
b= =
0= b c bx + cy –30 2

0 0 (ax2 + byx + byx + cy 2 ) 0 0 6


Dc= 4 2 17 = 6[–12 – 18] = –30 × 6
0 = –(ax2 + 2bxy + cy2) (ac – b2)
9 −3 6
0 = (b2 – ac) (ax2 + 2bxy + cy2)
DC –30 × 6
1 b−c c+b 0 1 c+b C=
= = 6
D –30
a 0 c −a − b + b c 0 −a − b x2 3
Equation → ax2+bx+c = + x +6
0 a+c −b −b 0 −b 2 2

0 –c 1 Sol 41: 2x – y = 5
+c c c 0
4x – 2y = 10
−b a + c 0
2 −1
D= =0
1 2 3 4 −2
Sol 39: –4 3 6 5 −1 2 5
2 –7 9 Dx = = 0, Dy = =0
10 −2 4 10
2 3
⇒M21 = = 18 + 21 = 39 So system has infinite solution (consistent).
−7 9
1 7 . 4 8 | Determinants

Sol 42: 3x – y – 2z = 2
1 a2 a3
2y – z = – 1 = 1 b2 b3 = R.H.S.
3x – 5y = 3 1 c 2
c 3

3 −1 −2 C2 ↔ C3 and then C1 ↔ C2
D = 0 2 −1
3 −5 0 0 p −q p −r
= 3[–5] –1[–3] –2[–6] = –15 + 3 + 12 = 0 Sol 45: q − p 0 q−r
r −p r −q 0
2 −1 −2
Dx = –1 2 −1 q−p q−r q−p 0
= –(p – q) + (p – r)
3 −5 0 r −p 0 r −p r −q

= 3[1 + 4] – 5[2 + 2] = 15 – 20 = –5 ≠ 0 =+(p – q (q – r) (r – p) – (p – q) (q – r) (r – p)=0


So system is inconsistent.
x2 0 3
1 a bc 1 a a 2 Sol 46: x 1 −4 = 11
Sol 43: 1 b ca = 1 b b2 1 2 0
1 c ab 1 c c2 ⇒ x2[8] + 3[2x – 1] = 11

1 a bc a a2 abc ⇒ 4x2 + 3x – 7 = 0
1 ⇒ (x – 1) (4x + 7) = 0
L.H.S. = 1 b ca = b b2 abc
abc
1 c ab c c2 abc ⇒ (x – 1)(4x + 7) = 0
7
a a2 1 ∴ x = 1 or –
abc 4
= b b2 1 C2 ↔ C3
abc
c c2 1 3 2  2
Sol 47: A =   , A – 4A – I = 0
a 1 a 2 2 1 
= – b 1 b2 C1 ↔ C2 Assume A – xI = 0 →
2
c 1 c 3 − x 2 
  =0
 2 1 − x
2
1 a a
2
= (–1)2 1 b b = R.H.S. (3 – x)(1 – x) – 4 = 0
1 c c2 3 + x2 – x – 3x – 4 = 0
⇒ x2 – 4x – 1 = 0
2 2 3
a a bc 1 a a A – xI = 0
2 2
Sol 44: b b ca = 1 b b3
⇒ A2 – 4A – I = 0 Hence proved.
c c2 ab 1 c2 c3
⇒ A–1[A2 – 4A – I] = 0
a a2 bc A – 4I – A–1 = 0
2 abc
L.H.S. = b b ca ×
2
abc 3 2  1 0 
c c ab ⇒ A – 4I =   –4  
2 1  0 1 
a2 a3 abc a2 1 a3
1 abc 2 3 − 4 2   –1 2 
= b2 b3 abc =
3
b b 1 A–1 = 
abc 2 abc 2  =  
c c3 abc c c3 1  2 1 − 4  2 –3
M a them a ti cs | 17.49

1 / a a2 bc a/a a3 abc 0 a + c − 2b a − b
2 1 = (a + b + c) 0 b − 2c + a b − c
Sol 48: 1 / b b ac = b / b b3 abc
(abc) 1 c−a c
1 / c c2 ab c / c c3 abc
= (a + b + c) [(b – c)(a + c – 2b) – (a – b)(b + a – 2c)]
1 a3 1
abc = a2b + b2a – b2a – a2b + ….. + 3abc – a3 – b3 – c3
= 1 b3 1 = 0
(abc) = 3abc – a3 – b3 – c3
1 c3 1

2 −3
x+a b c Sol 52: A =  
Sol 49: b x+c a =0 3 4 
c a x +b Assume |A – xI| = 0
Have to show that x = –(a + b + c) 2 −3 1 0 
⇒   –x   =0
R1 → R1 + R2 + R3 3 4  0 1 
x +a+b+c x +a+b+c x +a+b+c 2 − x − −3 
b x+c a =0 ⇒   = |0| = 0
 3 4 − x
c a x +b
⇒ (2 – x) (4 – x) + 9 = 0
1 1 1
⇒ 8 + x2 – 4x – 2x + 9 = 0
(x+a+b+c) b x + c a =0
⇒ x2 – 6x + 17 = 0 and |A – xI| = 0
c a x +b
So, A satisfied this equation
x + a + b + c = 0 ⇒ x = –(a + b + c)
⇒ A2 – 6A + 17 I = 0
A–1[A2 – 6A + 17I] = 0
x+4 x 2
Sol 50 : 2 x+4 x ⇒ A – 6I + 17A–1 = 0
x x x+4 –17A–1 = (A – 6I)

C1 → C1 − C3 ,C2 → C2 − C3 –1 1  2 −3 1 0  
A–1 = (A – 6I) =   −6 
17 17  3 4  0 1  
4 0 x
1 2 − 6 −3  1  −4 −3
0 4 x A–1 = –   =–  
17  3 4 − 6 17  3 –2 
−4 −4 x + 4

= ( 4x + 16 + 4x ) + 16x = 48x+64 1  4 3
A–1 =  
17  −3 2

b + c a−b a
2 4  0 2  1 6 
Sol 51: c + a b − c b = 3abc – a3 – b3 – c3 Sol 53:   A 1 3 = 3 –1
1 3     
a+b c−a c
Assume BAC = D
C1 → C1 + C3
2 4
a+b + c a−b a 1 a−b a |B| = =6–4=2
1 3
= a + b + c b − c b = (a+b+c) 1 b − c b
a+b+c c−a c 1 c−a c 3 –4
Adj B =
R1 → R1 – R3, R2 → R2 – R3 –1 2

1  3 −4 
B–1 =  
2  −1 2 
1 7 . 5 0 | Determinants

B–1BAC = B–1D  −3 + 3 + 3 8 + 3 − 30 −12 − 21 + 45   3 −19 12 


–1   –1  
1 3 −4 1 6 −3 + 4 + 3 8 + 4 − 30 −12 − 28 + 45 =  4 −18 5 
AC = B–1D = 11  11
 −3 + 3 + 4 8 + 3 − 40 −12 − 21 + 60   4 −29 27 
2 −1 2 3 –1    

1  3 − 12 18 + 4  1  −9 22   2 −1 1 
AC =   =  
2  –1 + 6 –6 − 2  2  5 –8  Sol 55: A =  −1 2 −1 ,
 1 −1 2 
0 2   
|C| =   = –2 Assume (A –XI) = 0
1 3
2 − x −1 1 
 3 –2 1  –3 2   
adjC =   ,C = 
–1
 ⇒  −1 2 − x −1  = 0
 –1 0  2  1 0  1
 −1 2 − x 
ACC–1 = B–1DC–1 (2 – x) [(2 – x)2 – 1[–1 + 2 – x] + 1[1 – 2 + x] = 0
1 1  –9 22   −3 2  ⇒ (2 – x)[4 + x2 – 4x –1] –1 + x –1 + x = 0
A= ×   
2 2  5 −8   1 0  ⇒ 6 – x3 + 2x2 – 8x – 3x + 4x2 – 2 + 2x =0

1 27 + 22 −18  1  49 −18  ⇒ –x3 + 6x2 – 9x + 4 = 0


A=   =+  
4  −15 − 8 10  4  −23 10  ⇒ x3 – 6x2 + 9x – 4 = 0
|A – xI| = 0, so this equation satisfied A

5 0 4  1 3 3  ⇒ A3 – 6A2 + 9A – 4I = 0 
⇒ A–1[A3 – 6A2 + 9A – 4I] =


    A–10 = 0
Sol 54: A = 2 3 3  , B–1 = 1 4 3 
1 2 1  1 3 4  ⇒ A2 – 6A + 9I – 4A–1 = 0
   
(AB)–1 = B–1A–1  2 −1 1   2 −1 1 
   
C11 = 3 – 6 = –3, A =  −1 2 –1  −1 2 –1
2

 1 −1 2   1 −1 2 
C12 = 3 – 2 = 1,    

C13 = 4 – 3 = 1,  4 + 1 + 1 −2 – 2 – 1 2 + 1 + 2 
 
C21 = 8, =  −2 − 2 − 1 +1 + 4 + 1 –1 − 2 − 2 
 2 + 1 + 2 −1 – 2 – 2 1 + 1 + 4 
C22 = 5 – 4 = 1,  

C23 = –10,  6 −5 5 
 
C31 = –12, A =  −5 6 −5 ,
2

 5 −5 6 
C32 = 8 – 15 = –7,  

C33 = 15 4A–1 = A2 – 6A + 9I

|A| = 5(–3) + 4(1) = –15 + 4 = –11  6 −5 5   2 −1 1  1 0 0 


     
4A =  −5 6 −5 – 6  −1 2 −1 + 9 0 1 0 
–1

 −3 8 −12  5 −5 6   1 −1 2  0 0 1 
1 –1        
A =
–1
adjA =  1 1 −7 
|A| 11 6 − 12 + 9
 1 −10 15  −5 + 6 5−6 
   
4A =  −5 + 6
–1
6 − 12 + 9 −5 + 6 
1 3 3   –3 8 12   5−6 −5 + 6 6 − 12 + 9 

1    
B A =–
–1 –1
1 4 3  1 1 –7 
11   3 1 −1
1 3 4   1 –10 15 
    1 
A–1 = 1 3 1
4
 −1 1 3 
 
M a them a ti cs | 17.51

Exercise 2 1 a2 a3 a a3 1
2 3 3
⇒ abc 1 b b = b b 1
Single Correct Choice Type
2 3 3
1 c c c c 1

1+a 1 1 R1 → R1 – R3, R2 → R2 – R3
Sol 1: (D) 1 1+b 1 =0
1 1 1+c 0 a2 – c2 a3 – c3 a – c a3 – c3 0
2 2 3 3 3 3
⇒ abc 0 b – c b –c = b–c b –c 0
1 1 1 2 3 3
+1 1 c c c c 1
a a a
1 1 1 ⇒ abc [(a2–c2) (b3 – c3) – (b2 – c2) (a3 – c3)
abc 1+ ` =0
b b b
1 1 1 = [(a – c) (b3 – c3) – (b – c) (a3 – c3)
+1
c c c ⇒ abc (a – c) (b – c)[(a + c) (b2 + c2 + bc)

R1 → R1 + R2 + R3 – (b + c) (a2 + c2 + ac)
= (a – c) (b – c) [b2 + c2 + bc – (a2 + c2 + ac)]
1 1 1 1 1 1 1 1 1
1+ + + 1+ + + 1+ + +
a b c a b c a b c abc [ab2 + ac2 + abc + cb2 + c3 + bc2
1 1 1 – ba2 – bc2 – abc – ca2 – c3 – ac2]
abc 1+ =0
b b b
1 1 1 = b2 + c2 + bc – a2 – c2 – ac
1+
c c c = (b – a) (b + a + c)
⇒ abc (b – a) [ab + c(b + a)]
1 1 1 = (b – a) (a + b + c)
 1 1 1 1 1 1
1 + + +  1+ =0 ⇒ abc [ab + bc + ca] = [a + b + c]
 a b c b b b
1 1 1
1+ Sol 3: (A) (sin–1x + sin–1w) (sin–1y + sin–1z) = p2
c c c
N1 N2
C2 → c2 – c1, C3 → c3 – c1 x y
D→
N3 N4
z w

1 0 0 –1 ≤ (x, y, w, z) ≤ 1
 1 1 1 1 N N4 N N4
1 + + +  1 0 =0 x 1w – z 3y
 a b c b
1 If x = y = 7 = w = – 1
0 1
c
N1 N2
x y N2 +N4 N2 +N3
 1 1 1 → (–1) – (–1)
 1 + + +  = 0 ⇒a + b + c = – 1 N N
–1 –1 –1
a b c z 3 w 4
 
For max value
3 4 N1 + N4 = 2n. N2 + N3 = 2m + 1
a a a –1
Sol 2: (A) b b3 b 4 – 1 = 0 ⇒ n,m ∈ N
3 4
c c c –1 Value (–1)2n – (–1)2n – (–1)2m+1
⇒ 1 – (–1) = 2
a a3 a4 a a3 –1
Min value → – 1 – 1 = – 2
⇒ b b3 b 4 + b b3 –1 = 0
c c3 c4 c c3 –1 Dependent of N1, N2, N3, N4
1 7 . 5 2 | Determinants

Sol 4: (C) (1+ x + x2)n = a0 + a1x + a2x2 + …+… a2n x2n = D3 (given)

an–3 an–1 an+1 in D3 → C1 → C1 – C3, C2 → C2 – C3


an–6 an–3 an+3 and assume T = a2+b2 + c2 – ab – bc – ca
an–14 an–7 an+ 7
T 0 ab + bc + ca
(1 + x + x2)n = (x2 + x + 1)n +0 T ab + bc + ca
an–1 = an+1 –T –T a2 + b2 + c2
a0 = an 1 0 ab + bc + ca
an – r = an + rO ≤ r ≤ n = T2 0 1 ab + bc + ca
an–3 an–1 an–1 –1 –1 a2 + b2 + c2
So determinate → an–6 an–3 an–3 T2[a2+b2+c2+ab+bc+ca+ab+bc+ca(1)]
an–14 an–7 an–7
= T2[a2 + b2 + c2 + 2(ab + bc + ca)]
C2 → C2 – C3
bc – a2 ac – b2 ab – c2
an–3 0 an–1
D2 = ac – b2 ab – c2 bc – a2
an–6 0 an–3 = 0
ab – c2 bc – a2 ac – b2
an–14 0 an–7
= C1 → C1 + C2 + C3
–1 2 1
Sol 5: (A) 3 + 2 2 2 + 2 2 1 C1 → C1 – C2 – C3 –T ac – b2 ab – c2
3–2 2 2–2 2 1 D2 = –T ab – c2 bc – a2
–T bc – a2 ac – b2
–1 – 2 – 1 2 1
–1 ac – b2 ab – c2
(– 1) 3 + 2 2 – 2 – 2 2 – 1 2 + 2 2 1
3

D2 = T –1 ab – c2 bc – a2
3–2 2 –2+2 2 –1 2–2 2 1
–1 bc – a2 ac – b2
–4 2 1
R1 → R1 – R3, R2 → R2 – R3
=– 0 2 + 2 2 1 = + 4 [2 + 2 2 –(2 – 2 2 )]
0 2–2 2 1 0 ac – b2 + a2 – bc ab – c2 + b2 – ac
2 2 2 2
D2 = T 0 ab – c + a – bc bc – a + b – ac
= + 4 [ 4 2 ] = 16 2
–1 bc – a2 ac – b2
a b c D2 = – T [(ac – b2 + a2 – bc) (bc – a2 + b2
Sol 6: (C) D1 = b c a ,
– ac) – (ab – c2 + b2 – ac) (ab – c2 + a2 – bc)]
c a b
D2 = (– T) (– T) [T + 3 (ab + bc + ca)]
2 2 2
bc – a ac – b ab – c
D2 = T2[a2 + b2 + c2 + 2(ab + bc + ca)]
2 2 2
D2 = ac – b ab – c bc – a
2 2 ∴ D12 = D2 = D3
ab – c bc – a ac – b2

a b c a b c
1 1 1 1 1 1
D12 = b c a b c a
Sol 7: (C) a b c = a b c
c a b c a b
bc ca ab a3 b3 c3
2 2 2
a +b +c ab + bc + ca ab + bc + ca
L.H. S. = (a – c) (b – c) (b – a)
= ab + bc + ca a2 + b2 + c2 ab + bc + ca
in L.H.S. C1 → C1 – C3, C2 → C2 – C3
ab + bc + ca ab + bc + ca a2 + b2 + c2
M a them a ti cs | 17.53

= (a + 1) (a + 3) [a2 + 1 + 2a – a2 – 9 – 6a]
0 0 1
a–c b–c c = (a + 1) (a + 3) ( – 4a – 8)
a3 – c3 b3 – c3 c3 = – 4 (a + 2) (a + 1) (a + 3)
D = D x = Dy = 0
0 0 1
⇒ a = – 2 (common solution in all)
= (a – c) (b – c) 1 1 c
a2 + c2 + ac b2 + c2 + bc c3
Sol 9: (B) 3x – 7y + 5z = 3, 3x + y + 5z = 7
⇒ (a – c) (b – c)[b2 + c2 + bc – a2 – c2 – ac]
2x + 3y + 5z = 5
⇒ (a – c) (b – c) (b – a) (b + a + c)
3 –7 5
→a+b+c=1
D= 3 1 5
= (a – c) (b – c) (b – a) 2 3 5
abc = a + b +c = 3 [5 – 15] – 7 [10 – 15] + 5 [9 – 2]
A.M. ≥ G.M = – 30 + 35 + 35 = 40 ≠ 0
a+b+c 1
≥ (abc)1/3 ; ≥ (abc)1/3 So system is consistent with unique non trivial solution.
3 3
1 Sol 10: (B) (sin θ)x + 27 = 0
≥ abc → abc is always less than 1/27
27
(cos θ)x + sin θ y = 0
Sol 8: (A) (a + 1)3x + (a + 2)3y = (a + 3)3 (cos θ)y + 2z = 0
(4 + 1)x + (a + 2)y = (a + 3) sin θ 0 2
x+y=1 D = cos θ sin θ 0
Here for two variable thus equation 0 cos θ 2

So D = Dx = Dy=0 for consistent D = sin θ (sin θ.2) + 2(cos2θ)

(a + 1)3 (a + 2)3 = 2 (sin2θ + cos2θ) = 2 Constant


D=
(a + 1) (a + 2) 0 0 0 2
C = 0 , Dx = 0 sin θ 0
(a + 1)2 (a + 2)2 0 a cos θ 2
= (a + 1) (a + 2)
1 1
So system has a unique solution which is a function of
= (a + 1) (a + 2) [a2 + 1 + 2a – a2 – 4 – 4a] = (a + 1) a and θ
(a + 2) (–2a – 3) … (i)
(1 + x)2 (1 – x)2 –(2 + x2 )
3 2
(a + 3) (a + 2) Sol 11: (D) 2x + 1 3x 1 – 5x
Dx =
(a + 3) (a + 2) x +1 2x 2 – 3x
(a + 3)2 (a + 2)2 (1 + x)2 2x + 1 x +1
= (a + 2) (a + 3)
1 1 2
+ (1 – x) 3x 2x = 0
= (a + 2) (a + 3) (a + 9 + 6a – a – 4 – 4a)
2 2
1 – 2x 3x – 2 2x – 3
= (a + 2) (a + 3) (5 + 2a) … (ii) In 2nd determinate R3 → + R3 + R1
(a + 1)3 (a + 2)3 (1 + x)2 2x + 1 x +1
Dy =
(a + 1) (a + 3) (1 – x)2
3x 2x =- B  1 – 2x + (1 + x)2
(a + 1)2 (a + 3)2 +(2 + x2 ) –(1 – 5x) 3x – 2
= (a + 1) (a + 3)
1 1 = 2 + x2 + 2x – 2x = 2 + x2
1 7 . 5 4 | Determinants

R3 → – R3 So Dx = Dy = Dz = 0

(1 + x)2 (2x + 1) x +1 sin3θ –1 1


2
⇒ |A| – (1 – x) 3x 2x = B D = cos2θ 4 3
–(2 + x2 ) 1 – 5x 2 – 3x 2 7 7

x, y, z are not all simultaneously equal to zero so for


Now all rows of A is equal to columns of B
solution (not-trivial), D = 0
⇒ |B| = – |A|
sin 3θ [28 – 21] – 1[6 – 7cos 2q]
|A| – |A| = 0 (always)
+ 1 [7 cos2θ – 8] = 0
For every valued of x
⇒ 7 sin 3θ – 6 + 7 cos2θ + 7 cos2θ – 8 = 0
|A| + |B| is zero
⇒ 7 sin 3θ + 14 cos 2θ = 14
Therefore infinite solutions
⇒ Sin 3θ + 2 cos 2θ = 2

Sol 12: (B) 2x cos2 θ + y sin 2θ – 2sin θ = 0  … (i) ⇒ Sin 3θ + 2 cos 2θ = 2

x sin 2θ + 2y sin2 θ = – 2 cos θ …(ii) ⇒ 3 sin θ – 4 sin3θ + 2[1 – 2 sin2q] = 2

x sin θ – y cos θ = 0 …(iii) ⇒ 3 sin θ – 4 sin3θ + 2 – 4 sin2θ = 2

for (i) & (ii) assume sin θ = x


⇒ 4x3 + 4x2 – 3x = 0
2cos2 θ sin2θ
D= ⇒ x[4x2 + 4x – 3] = 0
sin2θ 2sin2 θ
⇒ x[4x2 + 6x – 2x – 3] = 0
= 4 sin2θ cos2θ – 4 sin2 θ cos2 θ = 0
x = 0 or 2x (2x + 3) – 1 (2x + 3) = 0
= (sin2 θ = 2 sin θ cos θ)
(2x + 3) (2x – 1) = 0 ⇒ x = 1/2 or –3/2
2sin θ sin2θ
Dx = x = 0, ½, –3/2 but – 1 ≤ sin θ ≤ 1
–2cos θ 2sin2 θ
sin θ ≠ –3/2
= 4 sin θ + 4 sin θ cos θ
3 2
sin θ ∈ {0, ½}, x = 0, π/6, 5π/6, π, 12π
= 4 sin θ(sin2 θ + cos2 θ) = 4 sin x
between [0, 2p]
for consistent Dx = 0 → 4 sin θ = 0
No. of principle value = 5
θ ∈ n π, n ∈ I
a2 + b2
2sin2 θ 2sin θ c c
Dy = = 4 cos3θ – 4 sin2θ cos θ c
sin2θ – sin θ
b2 + c2
Sol 14: (D) a a = α abc
= – 4 cos θ [sin θ + cos θ) = – 4 cos θ
2 2
a
Dy = 0 θ = (2n + 1)π/2, n ∈ I a2 + c2
b b
b
Sin θ and cos θ both are not zero for same θ, so for
every value of θ system has not a solution
a2 + b2
1 1
c2
Sol 13: (C) (sin 3θ)x – y + z = 0
b2 + c2
abc 1 1
(cos 2θ)x + 4y + 3z = 0 a2
2x + 7y + 7z = 0 a2 + c2
1 1
0 b2
( c = 0  a2 + b2   (b2 + c2 )(a2 + c2 ) 
0 ⇒ abc    – 1
 c2   a2b2 

M a them a ti cs | 17.55

 a2 + c2    b2 + c2    0 x y
– – 1 + 1 1 –   
 b
2
   a2    ⇒ 0 y z =0
2
−(xp + yp + yp + z) xp + y yp + z
 (a b + a c + b + b c )(a + c ) – b a – a b
2 2 2 2 4 2 2 2 2 4 2 4 2
⇒ abc  ⇒ −(xp2 + 2yp + z)(xz − y 2 ) =
0
 a2b2c2
∴ Either xp2 + 2yp + z =0 or y 2 = xz
a +c –b
2 2 2  a –b –c
2 2 2 
– +  ⇒ x, y, z are in GP.
 b2  a2 
 
a4b2 + a4 c2 + a2b 4 + a2b2c2 + a2b2c2 + a2c 4  Sol 2: (A) Given
abc  4 2 
1 x x +1
⇒  +b c + b2c 4 – a2b2c2 – a4 c2 – a2c 4 + a2c2b2 
a2b2c2  2 2 2 4 2 2 4 4 2 2 4  f(x)
= 2x x(x − 1) (x + 1)x
 +b a c – b c – b c – a b – a b 
3x(x − 1) x(x − 1)(x − 2) (x + 1)x(x − 1)
1  2 2 2
= 4a b c + a4b2 + a2b 4  = 4abc =2abc
abc   Applying C3 → C3 − (C1 + C2 )

⇒α=4 1 x 0
= 2x x(x − 1) = 0 0
Sol 15: (C) a2x + (2 – a)y = 4 + a2 3x(x − 1) x(x − 1)(x − 2) 0
ax + (2a – 1)y = a5 – 2
∴ f(x) = 0 ⇒ f(100) = 0
2
a 2–a
D= = a2(2a – 1) + (a – 2) a
a 2a – 1 Sol 3: (D) Since, the given system has non-zero solution.

= 2a3 – a2 + a2 – 2a 1 −k −1
∴ k −1 −1 =0
For D = 0 = 2a (a2 – 1) → +1, – 1, 0
1 1 −1
4 + a2 2–a
Dx = Applying C1 → C1 − C2 , C2 → C2 + C3
5
a – 2 2a – 1
1 + k −k − 1 −1
= (4 + a2) (2a – 1) + (a – 2) (a5 – 2)
⇒ 1+k −2 −1 =0
= 8a – 4 + 2a3 – a2 + a6 – 2a5 – 2a + 4 0 0 −1
at a = 0 Dx = 0 ⇒ 2(k + 1) − (k + 1)2 =
0
a2 4 + a2 0 4+0 ⇒ (k + 1)(2 − k − 1) =0
So Dy = =
a a3 – 2 0 0–2 ⇒k= ± 1
So at a = 0, system has infinite solution Note: There is a golden rule in determinant that n one’s
⇒ (n – 1) zero’s or n(constant) ⇒ (n – 1) zero’s for all
At a = – 1, + 1, D = 0, and Dx, Dy ≠ 0
constant should be in a single row or a single column.
⇒ No solution, no. of values = 2
sinx cos x cos x
Sol 4: (C) Given cos x sinx cos x = 0
Previous Years Questions
cos x cos x sinx

Applying C1 → C1 + C2 + C3
xp + y x y
Sol 1: (B) Given yp + z y z 0
= sinx + 2cos x cos x cos x
0 xp + y yp + z = sinx + 2cos x sinx cos x
Applying C1 → C1 − (pC2 + C3 ) sinx + 2cos x cos x sinx
1 7 . 5 6 | Determinants

1 cos x cos x When k ≠ 3 , the given system of equation has no


solution.
(2cos x + sinx) 1 sinx cos x =
= 0
1 cos x sinx ⇒ Statement I is true. Clearly, Statement II is also true
as it is rearrangement of rows and columns of
Applying R 2 → C2 − R1 , R 3 → C3 − R1  1 −2 3
 
1 cos x cos x  1 −3 4 
 −1 1 −2
⇒ (2cos x + sinx) 0 sinx − cos x 0 0
=  
0 0 sinx − cos x
Sol 8: Given systems of equation can be rewritten as
⇒ (2cos x + sinx)(sinx − cos x)2 =
0
–x + cy + bz = 0
0 or sinx − cos x =
⇒ 2cos x + sinx = 0
cx – y + az = 0 and bx + ay – z = 0
⇒ 2cos x = − sinx or sinx = cos x
Above system of equations are homogeneous equation.
1 π π
⇒ cot x = − gives no solution in − ≤ x ≤ and sin Since, x, y and z are not all zero, so it has non-trivial
2 4 4 solution.
x = cos x ⇒ tan x = 1
Therefore, the coefficient of determinant must be zero
π
⇒ x= −1 c b
4
∴ c −1 a =
0
Sol 5: (A) Given equations b a −1

x + ay = 0, az + y = 0 and ax + z = 0 ⇒ −1(1 − a2 ) − c( −c − ab) + b(ca + b) =0


has infinite solutions. ⇒ a2 + b2 + c2 + 2abc − 1 =0
⇒ a2 + b2 + c2 + 2abc =
1
1 a 0
∴ 0 1 a = 0 ⇒ 1 + a3 =
0 or a = –1
Sol 9: Since α is repeated root of f(x) = 0.
a 0 1
= a(x − α )2 , a ∈ constant ( ≠ 0)
∴ f(x)
Sol 6: (1) For infinitely many solution, we must have A(x) B(x) C(x)
k +1 8 4k Let φ(x) = A(α ) B(α ) C(α )
= = ⇒k=1
k k + 3 3k − 1 A ′(α ) B′(α ) C′(α )

[To show φ(x) is divisible by (x − α )2 , it is sufficient to


Sol 7: (A) The given system of equation can be show that φ(α ) and φ′(α ) =0 ].
expressed as
A(α ) B(α ) C(α )
 1 −2 3  x   −1 ∴ φ(α=
) A(α ) B(α ) C(α ) = 0
     
 1 −3 4  y  = 1 A′(α ) B′(α ) C′(α )
 −1 1 −2 z   k
     
[ R1 and R 2 are identical]
Applying R 2 → R 2 − R1 , R 3 → R 3 + R1
A ′(x) B′(x) C′(x)
1 −2 3  x   −1 Again, φ′(x) = A(α ) B(α ) C(α )
     A ′(α ) B′(α ) C′(α )
⇒ ~ 0 −1 1   y  =
 2
0 −1 1   z  k − 1 A ′(α ) B′(α ) C′(α )
    
φ′(α )= A(α ) B(α ) C(α ) =0
1 −2 3   x   −1
     A ′(α ) B′(α ) C′(α )
⇒ ~ 0 −1 1   y  =
 2
0 0 0   z  k − 3 [ R1 and R3 are identical]
    
Thus, α is repeated root of φ(x) =
0
⇒ R3 → R3 − R 2
Hence, φ(x) is divisible by f(x).
M a them a ti cs | 17.57

a b c  1+1+1 1+α+β 1 + α2 + β21 1 1 1 1


  Sol 12: (C) 1+α+β 2
1+α +β 2 3 3
1 + α + β= 1 α β ×1 α
Sol 10: (4) Given A = b c a  , abc = 1
 c a b 1 + α2 + β2 1 + α3 + β3 1 + α 4 + β3 1 α2 β2 1 β
 
T
and A A = 1  … (i)
1 1 1 1 1 1 1 1 1
Now, A T A = 1
= 1 α α × 1 α α = 1 α α2
2 2

 a b c   a b c  1 0 0  1 α2 β 1 β β2 1 β β2
    
⇒ b c a  b c a  = 0 1 0 
= (1 − α ) ( α − β ) ( β − 1 )
2 2 2
 c a b   c a b  0 0 1 
    
K =1
 a2 + b2 + c2 ab + bc + ca ab + bc + ca
 
⇒ ab + bc + ca a2 + b2 + c2 ab + bc + ca
 Sol 13: (C) ( 2 − λ ) x1 − 2x1 + x3 = 0
2 2 2
ab + bc + ca ab + bc + ca a + b + c 
2x1 − ( 3 + λ ) x2 + 2x3 = 0
1 0 0  −x1 + 2x2 − λx3 = 0
 
= 0 1 0 
Non-trivial solution
0 0 1 
  ∆ =0
2 2 2
1 and ab + bc + ca = 0 
⇒ a +b +c = … (ii) 2−λ −2 1
2 −3 − λ 2 = 0
We know, a3 + b3 + c3 − 3abc
−1 2 −λ
= (a + b + c)(a2 + b2 + c2 − ab − bc − ca)

⇒ a3 + b3 + c3 = (a + b + c)(1 − 0) + 3
(1 − λ ) {3λ + λ2 − 4} + 2. {−2λ + 2} + ( 4 − 3 − λ ) = 0
[from equation (i) and (ii)] (
⇒ 6λ + 2λ2 − 8 − 3λ2 − λ3 + 4λ − 4λ + 4 + 1 − λ = 0 )
3 3 3
∴ a + b + c = (a + b + c) + 3  … (iii) ⇒ −λ2 − λ2 − 5λ + 3 =0

Now, (a + b + c)2 =a2 + b2 + c2 + 2(ab + bc + ca) =1… (iv) x3 − λ 2 + 2λ 2 − 2λ − 3λ + 3 =0


∴ From equation (iii), a3 + b3 + c3 =1 + 3
3 3 3
( )
⇒ λ2 λ2 − 1 + 2λ ( λ − 1 ) − 3 ( λ − 1 ) =0
⇒ a +b +c =
4
(
⇒ ( λ − 1 ) λ2 + 2λ − 3 =0 )
k +1 8
Sol 11: (B) ∆= = k 2 + 4k + 3 − 8k ⇒ ( λ − 1 )( λ + 3)( λ − 1 ) =0
k k +3
λ 1,1, −3
=
= k 2 − 4k + 3
=(k − 3)(k − 1) Sol 14: (D) x + λy − z = 0
4k 8 λx − y − z =0
∆1= = 4k 2 + 12k − 24k + 8
3k − 1 k + 3 x + y − λz = 0
2
( 2
)
= 4k − 12k + 8 = 4 k − 3k + 2 = 4 (k − 2 )(k − 1 ) For non-trivial solution ⇒ ∆ =0
1 λ −1
k +1 4k
∆2= = 3k 2 + 2k − 1 − 4k 2 ⇒ λ −1 −1 =0
k 3k − 1
1 1 −λ
− (k − 1 )
2
=−k 2 + 2k − 1 =
As given no solution ⇒ ∆1 & ∆2 ≠ 0
{ }
⇒ λ + 1 − λ −λ2 + 1 − ( λ + 1 ) =0

⇒ λ ( λ − 1 ) =0
2
∆ =0
⇒k =
3 λ= 0, ±1
1 7 . 5 8 | Determinants

JEE Advanced/Boards D z 10
y= = = 2
D 5
Exercise 1 1 1 1
Dy = 3 6 1 = 1[1 – 6] = –5,
Sol 1: (a) x + y + z = 6 1 0 0
2x + y – z = 1 D y –5
y = = = –1
x + y – 2z = –3 D 5

1 1 +1 1 2 1
D = 2 1 –1 C1 → C1 – C2 Dz = 3 1 6 = 1[6 – 1] + 6[0] = 5m,
1 1 −2 1 2 0
Dz 5
0 1 +1 Z= = =1
D 5
⇒ 1 1 –1 = 1[+2 + 1] = 3
(c) 7x – 7y + 5z = 3
0 1 −2
3x + y + 5z = 7
6 1 +1
2x + 3y + 5z = 5
Dx = 1 1 –1 C3 → C3 + C2
–3 1 −2 7 −7 5
D= 3 1 5 R1 → R1 – R3; R2 → R2 – R3
6 1 2
2 3 5
⇒ 1 1 0 = 1[6 – 1] + 2(1 + 3) = –5 + 8 = 3
–3 1 −1 5 −10 0
1 –2 0 = 5[–10 + 10] = 0
1 6 1
2 3 5
Dy = 2 1 –1
1 –3 –2 3 −7 5
Dx = 7 1 5 R1 → R1 – R3; R2 → R2 – R3
= 1[–2 – 3] + 6[–1 + 4] + 1[–6 – 1] = –5 + 18 – 7 = 6
5 3 5
1 1 6
−2 −10 0
Dz = 2 1 1
2 –2 0 = 5[4 + 20] = 120 ≠ 0
1 1 –3
5 3 5
= 1[–3 – 1] + 1[1 + 6] + 6[2 – 1] = –4 + 7 + 6 = 9
D = 0 but Dx ≠ 0, so, system is inconsistent
Dx 3 Dy 6 Dz 9
x= = = 1 ,y= = = 2 , z= = = 3
D 3 D 3 D 3
Sol 2: x + ky + 3z = 0  … (i)
Here, it is consistent 3x + ky – 2z = 0 … (ii)

(b) x + 2y + z = 1 2x + 3y – 4z = 0  … (iii)

3x + y + z = 6 Equation has non-trivial solution.

x + 2y = 0 So, D = Dx = Dy = Dz = 0

1 2 1 1 k 3
D = 3 1 1 = 1[6 – 1] + 1[0] = 5 D = 3 k −2
1 2 0 2 3 –4

1 2 1 = 1 [–4k + 6] + k[–4 + 12] + 3[9 – 2k]


Dx = 6 1 1 = 2[6 – 1] = 10, = –4k + 6 + 8k + 27 – 6k = 33 – 2k = 0
0 2 0
M a them a ti cs | 17.59

33 Sol 4: a(y + z) = x → x – ay – az = 0
K= , assuming x = t
2 b(z + x) = y → bx – y + bz = 0
From equation (ii) – (i) c(x + y) = z → cx + cy – z = 0
2x – 5z = 0 0 
2x 2t  
z= = , (x = t) c = 0  , so Dx = Dy = Dz = 0
5 5 0 
 
In (iii)
So for non-trivial solution, D = 0
⇒ 2t + 3y – 4z = 0 → 3z = 4z – 2t
1 −a –a
 2t  8t − 10t –2t
⇒ 3y = 4   – 2t = = D= b –1 b C1 → C1 – C3; C2 → C2 – C3
5 5 5
c c –1
–2t
⇒y=
15 1+a 0 –a
 –2t 2t  D= 0 –(1 + b) b
(x, y , z) ⇒  t, ,  t∈R
 15 5  1+c 1+c –1

 1 1 1 
Sol 3: ax + y + z = α –1 →  + +  =0
 1 + a 1 + b 1 +c
x + ay + z = α – 1
or from equation
x + y + az = α – 1
x y z
a= ,b= ,c=
α 1 1 y+z x+z x+y
D= 1 α 1 x+y+z x+y+z
1+a= ; 1 + b = ;
1 1 α y+z x+z
D = a[a2 –1] + 1[1 – a] + 1[1 – a] x+y+z
1+c=
= a – α + 2 – 2α = a – 3α + 2
3 3 x+y

a3 – 3α + 2, 1 1 1 x+y+y+z+z+x
+ + =
1+a a+b 1+c x+y+z
At α = 1 ⇒ 1 – 3 + 2 = 0
2(x + y + z)
So (α – 1) is a factor of a3 – 3α + 2 = =2
(x + y + z)
Now, a3 – 3α + 2 can be witten as
⇒ a3 – a2 + a2 – α – 2α + 2 Sol 5: x = cy + bz → x – cy – bz = 0
⇒ a (α – 1) + α(α – 1) – 2(α – 1)
2
y = az + cx → cx – y + az = 0
⇒ (α – 1) (a + α – 2)
2
z = bx + ay → bx + ay – z = 0
D = (α – 1) (a + α – 2)
2
0 
D = (α – 1) (a2 + 2α – α – 2) (α – 1)  
c = 0  →
D = (α – 1)[α(α + 2) – 1(α + 2)] 0 
 
D = (α – 1) (α + 2) (α – 1) Dx = Dy = Dz = 0,
For D = 0, α = 1 or –2 But system has solution. So D = 0
For α = 1,
1 −c −b
0 1 1 D = c −1 a = 1[1–a2] + c[–c – ab) – b[ac +b] = 0
Dx = 0 1 1 = 0, so consistent b a −1
0 1 1
1 – a2 – c2 – abc – abc – b2 = 0
So on Dy and Dt = 0 \α ≠ 1 ⇒ α = –2 a2 + b2 + c2 + 2ab = 1
1 7 . 6 0 | Determinants

x Sol 8: x + y + z = 6
Sol 6: a = → x – ay + az = 0
y−z x + 2y + 3z = 10
y
b= → bx + y – bz = 0 x + 2y + lz = µ
z−x
z (a) A unique solution, D ≠ 0
c= → cx – cy – z = 0
x−y
1 1 1
0  D= 1 2 3
 
c = 0  , so Dx = Dy = Dz = 0, 1 2 λ
0 
  = 1[2λ – 6] + 1[–λ + 3] + 0
For solution → D = 0
= 2λ – 6 + 3 – λ = λ – 3 ≠ 0
1 −a a
λ≠3
D = b 1 −b
(b) Infinite solution
c −c −1
So D = 0, Dx = Dy = Dz = 0
= 1[–1 – bc] – a[–bc + b] + a[–bc–c]
D=0→λ=3
= –1 – bc + abc – ab – abc – ac
6 1 1 6 1 1
= –1(ab + bc + ca + 1) = 0
Dx = 10 2 3 = 10 2 3
= ab + bc + ca + 1 = 0 µ 2 λ µ 2 3

Sol 7: sinq ≠ cosq ⇒ 6[0] + 1[3µ – 30] + [20 – 2µ]

Xcosp – ysinp + z = cosq + 1  ... (i) ⇒ (µ−10)5 = 0

xsinp + ycosp + z = 1 – sinq ... (ii) µ = 10

xcos(p + q) – ysin(p + q) + z = 2  ... (iii) (c) No solution → D = 0, Dx ≠ 0

cos(A + B) = cosA cosB – sinAsinB λ = 3, µ ≠ 10

sin(p + q) = sinpcosq + cosp sinq


Sol 9: x + y + z = 1
equation (i)2 + equation (ii)2
x + 2y + 4z = p
⇒ x2(sin2p+cos2p) + y2(cos2p+sin2p)
x + 4y + 10z = p2
–2xy cospsinp + 2xcospz – 2yzsinp + 2xysinpcosp
1 1 1
+ 2xzsinp + 2z2 + 2yzcosp
D= 1 2 4
= z + 1 – 2sinq + 2cosq
1 4 10
⇒ x2 + y2 + z2 + 2xycosp – 2yzsinp
D = 1[20 – 16] + 1[4 – 10] + 1[4 – 2] = 4 – 6 + 2 = 0
+ 2xzsinp + z2 + 2yzcosp
So for solution, Dx = Dy = Dz = 0
= 2 + 1 – 2sinq + 2cosq
1 1 1
From equation (iii) and (i)
Dx = p 2 4
= x2 + y2 + z2 + 2z (1 + cosq – z) + 2q(1 – sinq – z) z2
p2 4 10
= 3 – 2(sinq – cosq)
= 1[20 – 16] + 1[4p2 – 10p] + 1[4p – 2p2] = 0
= x2+y2+z2 + 2z(2 + cosq – sinq – 2z)
= 4 + 4p2 – 10p + 4p – 2p2 = 0
= 3 + 2(cosq – sinq)
2p2 – 6p + 4 = 0
For equation (iii)
p2 – 3p + 2 = 0
⇒ 2z(2 + cosq – sinq – 2z) = 1 + 2 |cosq – sinq|
p2 – 2p – p + 2 = 0
\x + y + z = 2
2 2 2
M a them a ti cs | 17.61

(p – 2) (p – 1) = 0 ⇒ p = 1 or 2 4x + 4y – z = 2 … (ii)
For p = 1 6x + 6y + 2z = 3 … (iii)
⇒ x + y + z = 1 ... (i) Assume x = l
x + 2y + 4z = 1  … (ii) Equation (iii), (ii) – (iii).(ii)
x + 4y + 10z = 1 … (iii) 7z = 0 → z = 0
Assume that x = k 2y = 1 + 2z – 2x = 1 – 2λ
Equation (ii) – ii(i) (x, y, z) = (λ, 1–2λ, λ)
–x + 2z = –1 If K ≠ 2
k −1
⇒ 2z = x – 1 ⇒ z = K 2 −2 
2  
D =  4 2K −1
(k − 1)
So y = 1 – z – x = 1 – k – 6 6 K 
2  
2 − 2k − k + 1 3 − 3k = K[2K2 + 6] + 2[–6 – 4K] – 2[24 – 12K]
y= =
2 2 = 2K3 + 6K – 12 – 8K – 48 + 24K
 3 − 3k k − 1  = 2K3 + 22K – 60 = 2(K3 + 11K – 30)
(x,y,z) =  k, , 
 2 2 
At K = 2
At p = 2 ⇒ 2(8 + 11(2) – 30) = 0
x + y + z =1  … (1) So (K – 2) is a factor
x + 2y + 4z = 2 … (2)
k 3 + 11k – 30
= K2 + 2K + 15
x + 4y + 10z = 4 … (3) k −2
Assume x = k D = 2(K – 2) (K2 + 2K + 15)
Equation (2) – 2(1)
1 2 −2
–x + 2z = 0  
Dx = 2 2K −1
k 3 6 K 
⇒ x = 2z = k ⇒ z =  
2
= 2K2 + 6 + 2[–3 – 2K] – 2[12 – 6K]
k
y=1–x–z=1–k–
2 = 2K2 + 6 – 6 – 4K – 24 + 12K
3k 2 – 3k = 2K2 + 8K – 24 = 2[K2 + 4K – 12]
=1– =
2 2 = 2[K2 + 6K – 2K – 12] = 2[K(K+6) – 2(K + 6)]

Sol 10: Kx + 2y – 2z = 1 = 2(K – 2) (K + 6)

4x + 2Ky – z = 2 Similarly, Dy = (K – 2) (2K + 3) and Dz = 6(K – 2)2


6x + 6y + Kz = 3 if K ≠ 2,

K 2 −2  x y z 1
= = =
  2(K + 6) 2K + 3 6(K − 2) 2(K + 2K + 15)
2
D =  4 2K −1 at K = 2 (given)
6 6 K 
 
Sol 11: (a) a, b, c, d are distinct no.
2 2 −2
a, b, c, d ∈ {1, 2, 3, 4, 5}
= 4 4 −1 = 0
ax + by = 1
6 6 2
cx + dy = 2
⇒ 2x + 2y – 2z = 1 … (i)
1 7 . 6 2 | Determinants

a b Sol 12: (a – t)x + by + cz = 0


D= = ad – bc … (i)
c d bx + (c – t)y + az = 0

1 b cx + ay + (b – t)z = 0
Dx = = d – 2b,
2 d Has non-trivial solution,
Dx d − 2b So D = 0
x= =
D ad − bc a−t b c
for least possible +ve value of x D= b c−t a =0
d – 2b = 1 (least natural number) c a b−t

(d, b) → (3, 1) or (5, 2) Assume D = a0t3 + b0t2 + c0t + d0 = 0

ad – bc should be maximum for least x –d0


So t1t2t3 =
a0
(a, b) → (3, 1) (ad – bc) → (3a – c)
At t = 0, D = d0
a,c ∈ {7, 4, 5}
a b c
Max. → 3(5) – 2 = 15 – 2
So d0 = b c a
1
x= c a b
13
If a, b → (5, 2), And a0 is coefficient of t3 = (–1)(–1)(–1) = –1

ad – bc → 5a – 2c, a b c
–d0
t1t2t3 = = d0 = b c a
a, c ∈ {1, 3, 4} –1
c a b
Max. 5a – 2c → 5(4) – 2(1) = 18
1 p
→x= = (min.) Sol 13: 3x – y + 4z = 3
18 q
X + 2y – 3z = –2
p + q = 1 + 18 = 19
6x + 5y + lz = –3,
(b) x + ay = 3 and ax + 4y = 6 → x > 1, y > 0
3 −1 4
1 a
D= =4–a , 2 D = 1 2 –3
a 4
6 5 λ
3 a
Dx = = 12 – 6a ⇒ 3(2λ + 15) –1[–18 – λ) + 4[5 – 12]
6 4
⇒ 6λ + 45 + 18 + λ – 28 = 7λ + 35 = 7(λ + 5)
1 3
Dy = = 6 – 3a, D = 7(λ + 5)
a 6
3 −1 4
Dx 6(2 − a)
x > 0, > 101→ > 101 Dx = –2 2 –3
D (2 − a)(2 + a)
–3 5 λ
6
> 1,
01 = 3[2λ + 15] + 1[–2λ – 9] + 4[–10 + 6]
(2 − a)
= 6λ + 45 – 2λ – 9 – 16
2 + a < 6 → a = 1, 3
= 4λ + 20 = 4(λ + 5);
Dy3(2 − a) 3 1
y= = = = + ve D 4(λ + 5) 4
D 6(2 − a) 6 2 = x
x= =
D 7(λ + 5) 7
So a is 1 and 3 3 3 4
1+3=4 Dy = 1 −2 −3
6 −3 λ
M a them a ti cs | 17.63

= 3[–2λ – 9] + 3[–18 – l] + 4[–3 + 12] 1 a –a3


= –6λ – 27 – 54 – 3λ + 36 Dx = 1 b –b3
= –9λ – 45 = –(λ + 5) 1 c −c3
Dy –9(λ + 5) –9 = –(a + b + c) (a – b) (b – c) (c – a);
⇒y= = =
D 7(λ + 5) 7
\x = –(a + b + c)
3 −1 3
1 –a3 a2
Dz = 1 2 −2
Dy = 1 –b3 b2
6 5 −3
1 –c3 c2
= 3[–6 + 10] + 1[–3 + 12] + 3[5 – 12]
= (ab + bc + ca) (a – b)(b – c) (c – a)
= 12 + 9 – 21 = 0,
Dz \y = [ab + bc + ca]
z= =0
D –a3 a a2
So x, y, z is not dependent on λ Dz = −b3 b b2
(if λ ≠ –5) −c3 c c2
At λ = –5 = –abc(a – b) (b – c) (c – a)
3x – y + 4z = 3  … (i) \ z = –abc
x + 2y – 3z = –2 … (ii)
6x + 5y – 5z = –3  … (iii) Sol 15: (a) ax – y + z = α

Assume z = k, (iii) – (ii)(i) x – ay + z = 1

7y – 13z = –9 x – y + az = 1

13k − 9 D = a[–a2 + 1] –1 [1 – a] + [–1+a]


⇒y=
7 = –a3 + α – 2 + 2a
2 4 − 5k = (–a3 + 3α – 2) = –(a3 – 3α + 2)
So, x = 3z – 2y – z = 3k – (13k – 9) – z =
7 7
At α = 1
 4 − 5k 13k − 9 
(x, y, z)  , ,k  D = –(1 – 3+ 2) = 0
 7 7 
So (α – 1) is a factor
Sol 14: z + ay + a2x + a3 = 0
α3 − 3α + 2
= a2 + α – 2
z + by + b x + b = 0
2 3
α −1
z + cy + c2x + c3 = 0 So D = – (α – 1) (a2 + α – 2)
 a3  = –(α –1)(a2 + 2α – α – 2)
 
Now, c = b3  = –(α – 1)[α(α + 2) – 1(α + 2)]
 3
 c  = –(α – 1) (α – 1) (α + 2)
α ∈ [–10, 10]
1 a a2
So, α has an integral value
D = 1 b b2 = (a – b) (b – c) (c – a)
1 c c2 α −1 1
Dx = 1 −α 1
Dx Dy Dz 1 −1 α
x= ,y= ,z=
D D D
So x = 1,
Dx = –(α – 1)2(α + 2)
1 7 . 6 4 | Determinants

α α 1 (c) a, b ∈ {0, 1, 2, ……., 10}


Dy = 1 1 1 = 0, x+y+z=4
1 1 α 2x + y + 3z = 6
α –1 α x + 2y + az = b
Dz = 1 – α 1 =0
1 1 1
1 –1 1
D= 2 1 3
(a) Unique solution, 1 2 a
So D≠0 →α ≠1, –2 = 1(a – 6) + 1(3 – 2a) + 1(4 – 1)
Number of values for α in = a – 6 + 3 – 2a + 3 = – 9
[–10, 10] = 21 – 2 = 19 = L
4 1 1
(b) Number solution is not possible for every value of α,
Dx= 6 1 3
system has atleast one solution. So M = 0
b 2 a
(c) Infinite solution → D = 0
= 4(a–6) + 1(3b–6a) + 1(12–b)
α = 1, –2 → N = 2
= 4a – 24 + 3b – 6a + 12 – b
L – M + N = 19 + 2 = 21
= –2a + 2b – 12
(b) 2x + 3y – z = 0
1 4 1
3x + 2y + kz = 0
Dy= 2 6 3
4x + y + z = 0
1 b a
Has non-trivial solution
= 6a – 3b + 4[3 – 2a] + b – 6
2 3 −1 = 6a – 3b + 12 – 8a + b – 6
So, D = 0 ⇒ 3 2 k
= –2a – 2b + 6
4 1 1
1 1 4
= 2[2 – k] + 3[4k – 3] – 1[3 – 8] = 0
Dz= 2 1 6
4 – 2k + 12k – 9 + 5 = 10k = 0
1 2 b
⇒k=0
= b – 12 + 6 – 2b + 4[4 – 1]
⇒ 2x + 3y – z = 0 … (i)
= b – 12 + 6 – 2b + 12
3x + 7y = 0  … (ii)
= –b + 6
4x + y + z = 0  … (iii)
(i) Unique solution so D ≠ 0
(iii) – (ii) (i) →
→a≠0
– 5y + 32 = 0 → 3z = 5y
∴ a ∈ {1, 2, ………. , 10},
–2y
3x = – 2y → x = , b ∈ {0, 1, ………. , 10}
3
5 L = 10 × 11 = 110
z= y,y=y
3 (ii) Number solution D = 0, a = 0
x, y, z are integer , so at for x and z to be integer x = n Dx ≠ 0 → 2b ≠ 12 → b ≠ 6,
2
= – y and Dy ≠ 0 → b ≠ 3
3
3n M = 1(11 – 2) = 9
→y= (also an integer)
–2
(iii) Infinite solution D=0 → a=0,
So at n = –2, –7y = 3, z = 5 (minimum +ve value)
D x = Dy = Dz = 0
M a them a ti cs | 17.65

But Dx and Dz can’t be zero at same times, so no possible = [sin x (p+d – p) + a [sinx (p – d – p – d)]
common solution N = 0
+ a2[sin x (p – p + d)]
L + M – N = 110 + 9 – 0 = 119
= sin x d + a sin (– 2d) + a2 sin dx
2 It dose not depend upon p
–7 5 + 3i – 4i
3
Sol 16: 5 – 3i 8 4 + 5i x3 + 1 x 2 x x3 x2 x 1 x2 x
2 3
(c) y + 1 y 2
y = y 3
y 2
y + 1 y 2
y
+ 4i 4 – 5i 9
3 3
z +1 z 2
z z 3
z 2
z 1 z 2
z
2
(a) Assume z1 = 5 + 3i, z2 = + 4i
3
z3 = 4 + 5i x2 x 1 1 x2 x
2 2
= xyz y y 1 + 1 y y
(z3)2 = 42 + 52 = 41
2 2
z z 1 1 z z
–7 z1 z2
= (xyz + 1) (x – y) (y – z) (z – x) = 0
⇒ z1 8 z3
z2 z3 9 (given) x, y, z are all different
So (xyz + 1) = 0 ⇒ xyz = – 1
= –7[72–z3 z ]+z1[z2z3 – 9 z 2 ]+ z 2 [ z 1 z 3 – 8z2]
3

= –7[72 – 41] + (5 + 3i) a2 + 2a 2a + 1 1


Sol 17: (a) 2a + 1 a + 2 1 = (a – 1)3
 2  2 
 + 4i  ( 4 + 5i) – 9  – 4i   3 3 1
 3  3 
R1 → R1 – R3, R2 → R2 – R3
2   2 
+  – 4i  (5 – 3i) (4 – 5i) – 8  + 4i  
3   3  a2 + 2a – 3 2a + 1 0
= – 7 (31) + (5 + 3i) 2a – 2 a–2 0
3 3 1
8 10 
 + 16i + i – 20 – 6 + 36i = (a – 1) (a2 + 2a – 3) – 4(a – 1)2
3 3 
= (a – 1) [(a2 + 3a – a – 3) – 4 (a – 1)]
2   16 
+  – 4i  20 – 15 – 12i – 25i – – 32i
3   3  (a – 1) [(a – 1) (a + 3) – 4(a – 1)]

 –70 160i  2   1  = (a – 1)2 [a + 3 – 4] = (a – 1)3


= –217 + (5 + 3i)  +  +  – 4i   – – 69i
 3 3  3   3 
1 1 1 0 0 1
Coefficient of i (b) x y z ⇒ x–2 y–2 2
800 4 804 x3 y3 z2 x3 – z 3 y 3 – z3 z3
= – 70 + – 46 + = – 106 +
3 3 3
0 0 1
1 a a2 = (x – z) (y – z) 1 1 2
(b) cos(p – d)x cospx cos(p + d)x x2 + y 2 + xz y 2 + z 2 + yz z3
sin(p – d)x sinpx sin(p + d)x
∴ a2 – b3 = (a – b) (a2 + b2 + ab)
= 1[cos px sin (p+d)x – cos(p+d)x sin px] = (x – z) (y – z) (y2 + z2 + yz – x2 – z2 – xz)
+ a[cos(p+d)x sin (p – d)x – ws(p – d)x sin (p + d)x] = (x – z) (y – z) [ z (y – x) + (y2 – x2)
+ a2[cos (p –d)x sin px – cos px sin (p –d)x] = (x – y) (y – z) (z – x) (x + y + z)
1 7 . 6 6 | Determinants

22 0 1
x 1 –3 / 2 2
1 2 0 = 4[16] + [1] = 65
Sol 18: (a) f(x) = 2 2 1 x>1
1 0 1 22
0 1/2
x –1
a b c b+c c+a a+b
 1  3 2  Sol 19: D = c a b , D’ = a + b b + c c + a
f(x) = x [1 – 0] + 1  – 1 +  
x – 1  2 x – 1 b c a c+a a+b b+c
1 3
=x+ –1+ b c+a a+b c c+a a+b
x –1 x –1
D’= a b + c c + a + b b + c c + a
3 (x – 1)2 + 3 c a+b b+c a a+b b+c
= (x – 1) + =
x –1 x –1
C2 → C2 + C1 – C3, C2 → C2 – C1,
2 2
x + 1 + 3 – 2x x – 2x + 4 C3 → C3 – C1C3 → C3 – C2
= =
x –1 x –1
b c a c a b
3 3
F’(x) = 1 – ⇒ 01 = D’ = a b c + b c a
(x – 1)2 (x – 1)2
c a b a b c
⇒ (x – 1)2 = 3 ⇒ x = 1 ± 3 After swapping rows according to D

But x . so x = 1 ± 3 a b c a b c
D’ = c a b + c a b = 2D
–6
f’’(x) = , at x = 1 + 3 b c a b c a
(x – z)3
–6
f”(x) = > 0 so minima 1 + a2 – b2 2ab –2b
3 3 2 2
Sol 20: 2ab 1–a +b 2a
3
f(1 + 3)= 3 + = 2 3 2b –2a 1 – a2 – b2
3
But if x is integer for min. volue of f(x) C1 → C1 – BC3, C2 → C1 + AC3

⇒ x = [1 + 3]=2 1 + a2 + b2 0 –2b
3 0 2
1–a +b 2
2a
F(x) = f(2) = 1 + =4
1 2 3 3 2
b+b+a b+b –a + a – ab 1 – a2 – b2
(b) a2 + b2 + c2 + ab + bc + ca ≤ O ∀ a,b,c ∈ R
R3 → R3 + aR2 – bR1
(a + b + 2)2 a2 + b2 1
1 (b + c + 2)2 b2 + c2 1 + a2 + b2 0 –2b
c2 + b2 1 (c + a + 2)2 = 0 1 + a2 + b2 2a
0 0 1 + a2 + b2
(a + b) + (b + 1) + (c + a) ≥ 0
2 2 2

= (1 + a2 + b2)3
(always & square is +ve)
= 2 (a2 + b2 + c2 + bc + ca + ab)
sinx sin(x + h) sain(x + 2h)
Its given that a2+b2+c2 + bc + ca + ab ≤ 0
Sol 21: f(x) = sin(x + 2h) sinx sin(x + h)
So 0 ≤ a2 + b2 + c2 + ab + ca + ab ≤ 0 sin(x + h) sin(x + 2h) sinx
⇒ (a + b)2 + (b + 1)2 + (c + a)2 = 0
sin (A + B) = sin A cos B + cos A sin B
⇒a=b=c=0
M a them a ti cs | 17.67

sin (x + nh) = sin x cos (nh) + sin nh cos x ((γ + α – β – δ)2 – (α + β – γ – δ)2)

⇒ lim sin (x + nh) = (sin x ) 1+ (nh) cos x [(β + γ – α – δ)2 + (d2 + b2 – γ – δ)2
h→0
(γ + α – β – δ)2 – (α + β – ρ – δ)2]
sinx sinx + ncos x sinx + 2hcos x
= – 2 (α – β)2 (α – γ)2 (α – γ)2 (β – γ)2
⇒f(x) sinx + 2hcos x sinx sinx + hcos x
sinx + hcos x sinx + 2hcos x sinx (β – δ)2 (γ – δ) (– 1)6

C1 → C1 → C3, C2 → C2 – C3 = –64(α – β)(α – γ)(α – δ)(β – γ)(β – γ)(γ – δ)

–2hcos x –hcos2 x sinx + 2hcos x Sol 23: x3 – 3x2 + 2 = 0


f(x) = hcos x –hcos x sinx + hcos x
At x = 1 ⇒1 – 3 + 2 = 0.
cos x 2hcos x sinx
So (x – 1) is a factor of x3 – 3x2 + 2 = 0
–2cos x – cos x sinx + 2hcos x
(b + c)2 a2 a2
f(x) = h cos x
2 – cos x sinx + hcos x
b2 (c + a)2 b2
cos x 2cos x sinx
c2 c2 (a + b)2
–2cos x – cos x sinx
f(x) ⇒ x3 – 3x2 + 2 = (x – 1) (x2 – 2x – 2)
lim = cos x – cos x sinx
h→0 h2 a = 1, and bc = – 2, b + c = 2
cos x 2cos x sinx

R1 → R1, R3, R2 → R2 – R3 ⇒ bc = 1 ± 3 , c2b2 = 4 ± 2 3

–3cos x –3cos x 0 22 1 1
= 0 –3cos x 0 = 4 + 2 3 (2 – 3) 2
4+2 3
cos x 2cos x sinx 4–2 3 4–2 3 (2 + 3)2
= sin x (9 cos2x) = sin x (9 – 9sin2x) 4 1 1
= 9 sin x – 9sin3x = 3 (3sin x – 3 sin3x) = 4+2 3 7– 4 3 4+2 3

= 3 [sin 3x + sin3x] = k(sin 3x + sin3x) 4–2 3 4–2 3 7+4 3

⇒K=3 = 4[49 – 48] – [16 – 12] – 1 [16 + 12) + 28

(β + γ – α – δ)4 (β + γ – α – δ)2 1 + 30 3 + 24]+[16 – 12– (28 + 24 – 3 (30) = – 108

Sol 22: ( γ + α – β – δ)4 ( γ + α – β – δ)2 1


x + 2 2x + 3 3x + 4
( α + β – γ – δ )4 (α + β – γ – δ)2 1
Sol 24: (a) 2x + 3 3x + 4 4x + 5 = 0
R1 → R1 – R3, R2 → R2 – R3 3x + 5 5x + 8 10x + 17

x+2 4x + 6 3x + 4
1
4
(β + γ – α – δ) – (α + β – γ – δ) 4 4
(β + γ – α – δ) – (α + β – γ – δ) 2
0 = 2x + 3 6x + 8 4x + 5
2
( γ + α – β – δ )4 – ( α + β – γ – δ )4 ( γ + α – β – δ)2 – (α + β – γ – δ)2 0 3x + 5 10x + 16 10x + 17
( α + β – γ – δ )4 (α + β – γ – δ)2 1
c2→ c2 – c1 – c3
= ((β + γ – α – δ)2 – (α + β – γ – δ)2
x+2 0 3x + 4
– ((γ + α – β – δ)2 – (α + β – γ – δ)2) 1
= 2x + 3 0 4x + 5
2
3x + 5 –3x – 6 10x + 17
(β + γ – α – δ)2 + (α2 + β – γ – δ)2 1 0
2 2
( γ + α – β – δ) + (α + β – γ – δ) 1 0 [3x + 6]
4 2 ⇒ [(3x + 4) (2x + 3)–(x + 2) (4x + 5)]= 0
(α + β – γ – δ) (α + β – γ – δ) 1 2
= ((β – γ – δ – δ)2 – (α + β – γ – δ)2)
1 7 . 6 8 | Determinants

⇒ (3x + 6)[6x2 + 17x + 12 – 4x2 – 13x – 10]=0 x2 = a2 + b2 + c2 – (ab + bc + ca)


⇒ (3x + 6) [2x2 + 4x + 2] = 0 \ a + b + c = 0 ⇒ (a + b + c)2 = 0
⇒ (x + 2) (x2 + 2x + 1) = 0 a2 + b2 + c2 + 2 (ab + bc + ca)
⇒ (x + 2) (x + 1)2 = 0 –(a2 + b2 + c2 )
⇒ ab + bc + ca =
⇒ x = – 2, – 1 2

x – 2 2x – 3 3x – 4 a2 + b2 + c2 3
x2 = a2 + b2 + c2 + = (a2 + b2 + c2)
(b) x – 4 2x – 9 3x – 16 = 0 2 2
x – 8 2x – 27 3x – 64 3 2
x=± (a + b2 + c2 )
2
R1 → R2 – R3, R2 → R2 – R3
a c b
6 24 60 x = 0 c b a = a3 + b3 + c3 − 3abc = 0
⇒ 4 18 48 =0 b c a
x – 8 2x – 27 3x – 64
∴a+b +c =0
1 4 10
⇒ 2 9 24 =0 a
x – 8 2x – 27 3x – 64 Sol 26: xX33−–5x
5x2++3x
2
3x−–11==00 b

= 9(3x – 64) – 24(2x – 27) + 4 [24 (x – 8) c


– 2 (3x – 64)] + 10 [2(2x – 27) – 9(x – 8)] a b c
= (6 – 48 + 96 – 24 + 40 – 90) (x – 4) = 0 a – b b – c c – a C1 → C1 + C2 + C3
b+c c+a a+b
⇒x=4
a+b+c b c
Sol 25: a + b + c = 0 = 0 b–c c–a
a–x c b 2(a + b + c) c + a a + b
c b–x a =0
1 b c 1 b c
b a c–x
=5 0 b–c c–a =5 0 b–c c–a
c1 → c1 + c2 + c3 2 c+a a+b 2 a+c a+b
a+b+c – x c b
= 5[(b – c) (a + b) + (a – c) (a + c) + 2 (bc – ab – bc + c2)]
a+b+c – x b – x a
a+b+c – x a c–x = 5[ab – ac + b2 – bc + a2 – c2 – 2ab + 2c2]
= 5[a2 + b2 + c2 – (ab + bc + ca)]
1 c b
a2 + b2 + c2 = (a + b + c)2 – 2(ab + bc +ca)
= (a + b + c – x) 1 b.x a =0
1 a c–x = 25 – 2 (3) = 19
= 5[19 – 3] = 5. 16 = 80
a + b + c = 0 (a + b + c – x) = – x = 0

1 c b a2 + λ ab ac
1 b–x a Sol 27: ab 2
b +λ bc
1 a c–x 2 2
ac b c +λ
= (b – x) (c – x) – a + c (a – c + x) + b (c – b + x] = 0
2

(a2 + λ ) ab2 ac2


bc – x (b + 1) + x2 – a2 + a (– c2 + cx + ba – b2 + bx = 0 1
= a2b b(b2 + λ ) bc2
x + x(b + c – b – c) = a + b + c – (ab + bc + ca)
2 2 2 2 abc
a2c b2 c c(c2 + λ )
M a them a ti cs | 17.69

a2 + λ b2 c2 C A B A B C
= tan cot – 1 – tan cot + tan cot
abc 2 2 2 2 2 2 2 2 2
= a b +λ c
abc 2
a b 2 2
c +λ B C A B A C
– cot tan + tan cot +1 – tan cot
2 2 2 2 2 2
R1 → R1 – R3, R2 → R2 – R3 A 1
We know that tan =
2 A
λ 0 –λ 1 0 –1 cot
2
2
= 0 λ –λ = λ 0 1 –1
C B C A
a2 b2 c2 + λ a2 b2 (c2 + λ ) tantan tan tan
= 2 – 2 – 2 + 2
A A B B
= l2 (c2 + λ + b2 – 1[– a2]) tan tan tan tan
2 2 2 2
= l2 (a2 + b2 + c2 + λ)
A B
tan tan
– 2 +1–1+ 2 =0
a2 b2 c2 C C
tan tan
Sol 28: = 4 a b c 2 2
1 1 1
1  C B 1  A C
⇒  tan – tan  +  tan – tan 
C2 → C2 − C1 ,C3 → C3 − C1 A 2 2 B 2 2
tan  tan 
2 2
a2 b2 − a2 c2 − a2
1  B A
4 a b−a c−a = =
 ( ) ( )
4  b2 − a2 ( c − a) − c2 − a2 (b − a+) 
 C tan – tan  = 0
 2 2
1 1 1 tan
2
a2 c2 − a2
a c−a = =
=
 ( ) (
4  b2 − a2 ( c − a) − c2 − a2 (b − a) 
 ) It can only happen when two angles are equal.

1 ⇒ ∆ABC is isosceles
=-4 ( c − a)(b − a)(b − c )
= 4 ( c − b )(b − c )( c − a)

Exercise 2
Sol 29:
Single Correct Choice Type
A B C
cot cot cot
2 2 2 m
B C C A A B 2r – 1 Cr 1
tan + tan tan + tan tan + tan =0 2 m
2 2 2 2 2 2 Sol 1: (A) Dr = m –1 2 1+m
1 1 1 sin (m ) sin m sin2 (m + 1)
2 2 2

m
C1 → C1 – C3, C2 → C2 – C3 ∑ Dr =
r =0
A C B C C
cot – cot cot – cot cot m m
2 2 2 2 2
C A C B A B
∑ (2r – 1) ∑ m Cr m+1
=r 0=r 0
tan – tan tan – tan tan + tan
2 2 2 2 2 2 2
(m + 1)(m – 1) (m + 1)2m (m + 1)2
0 0 1
(m + 1)sin2 (m2 ) (m + 1)sin2 m (m + 1)sin2 (m + 1)

 C B  A C (m + 1)(m – 1) 2m (m + 1)
=  tan – tan  cot – cot  2 m
 2 2  2 2 = (m + 1)(m – 1) (m + 1)2 (m + 1)2
(m + 1)sin2 m2 (m + 1)sin2 m (m + 1)sin2 (m + 1)
 B C  C A
–  cot – cot  tan – tan 
 2 2  2 2
1 7 . 7 0 | Determinants

Common (m + 1) from C1, C3 and R2 C3 → C3 + C1

m–1 2m 1 bc 1 ab + bc + ca
= abc ac 1 ab + bc + ca
= (m+1)3 m – 1 2m 1 =0
ab 1 ab + bc + ca
sin2 m2 2 2
(m + 1)sin m sin (m + 1)
bc 1 1
= (abc) (ab + bc + ca) ac 1 1 = 0
1 cos(β – α ) cos( γ – α )
ab 1 1
Sol 2: (D) D = cos(α – β) 1 cos( γ – β)
cos(α – γ ) cos(β – γ ) 1
Sol 4: (A)
D = 1 – cos (β – γ) cos (γ – β) + cos(β – α)
mx mx – p mx + p
[cos (γ – β) cos (α – γ) – cos (α – β)] f’(x) = n n+p n–p
+ cos (γ – α) [cos (α – β) cos (β – γ) mx + 2n mx + 2n + p mx + 2n – p
– cos (α – γ)] C2 → C2 + C3
D = 1 – cos2(β – γ) + cos (β – α) cos (γ – β)
mx 2mx mx + p
cos (α – γ) – cos2 (β – α) + cos (γ – α)
f’(x) = n 2n n–p
cos (α – β) cos (β – γ) – cos2 (γ – α) mx + 2n 2(mx + 2n) mx + 2n – p
cos(β – γ )
D=1+2 [cos (γ – β) + cos C2 → C2 – 2C1
2
(γ – β – 2α)] – cos2 (γ – α) – cos2(β – α) – cos2(β – γ) mx 0 mx + p
f’(x) = n 0 n–p =0
 cos2(β – α ) + cos2( γ – α ) 
D = 1+cos2(β–γ)+   mx + 2n 0 mx + 2n – p
 2 
y = f(x)
– cos2(β – γ) – cos2(γ – α) – cos2 (β – α)
y’ = 0
=1+
1
2
(
2cos2 (β – α ) – 1 + 2cos2 ( γ – α ) – 1 ) y=K
It is a straight line parallel to x–axis.
– cos2 (β – α) – cos2(γ – α)
2
= 1 –   = cos2(β – α) + cos2 (γ – α) x − 1 (x − 1)2 x3
2
Sol 5: (A) D(x) = x − 1 x2 (x + 1)3
– cos2 (β – α) – cos2 (γ – α) = 1 – 1 = 0
x (x + 1)2 (x + 1)3
Assume D(x) a0 + a1x + …
b2 c2 bc b + c
Sol 3: (A) D = c2a2 ca c+a D’(x) = a1 + 2a2x

a2b2 ab a + b At x = 0 D’(0) = a1

+1 (x – 1)2 x3 x – 1 2(x – 1) x3
ab2c2 abc a(b + c)
1 D’(x) = +1 x2 (x + 1)3 + x – 1 2x (x + 1)3
= bc2a2 abc b(a + c)
abc 2 2 1 (x + 1)2 (x + 1)3 x 2(x + 1) (x + 1)3
ca b abc c(a + b)
x – 1 (x – 1)2 3x2
bc 1 a(b + c)
abc.abc + x –1 x2 3(x + 1)2 at x = 0
= ac 1 b(a + c)
abc x (x + 1)2 3(x + 1)2
ab 1 c(a + b)
M a them a ti cs | 17.71

1 1 0 –1 –2 0 –1 1 0 = 1 + (sin2x + cos2x) + 4 sin2x = 2 + 4 sin2x

D’(0) = 1 0 1 + –1 0 1 + –1 0 3 For max value


1 1 1 0 2 1 0 1 3 Sin 2x = 1
⇒ 2 + 4 sin 2x = 2 + 4 = 6
= 1[–1] + 1 [1 – 1] – 1[–2] –2 (1) –1 (–3) + 1[3]
=–1+0+2–2+3+3=–1+6=5 x2 + 3x x – 1 x + 3
Sol 8: (C) x + 1 2 – x x – 3 = px4+qx3+rx2 +5x+t
y+z z y
x – 3 x + 4 3x
Sol 6: (D) D = z z+x x , |D| = 8
y x x+y At x = 0

R1 → R1 – R2 – R3 0 –1 3
R2 → R2 – R3 1 2 –3 = t
–3 4 0
0 –2x –2x
D= z–y z –y t = 1 [– 9] + 3 [4 + 6] = 30 – 9 = 21
y x x+y

0 1 1 a2 + 1 ab ac
D = (–2x) z – y z –y Sol 9: (A) D = ba 1+b 2
bc
y x x+y 2
ca cb c +1
C2 → c2 – c3
a3 + a a2b a2c
0 0 1 1
= b2 a b + b3 b2 c
D = (–2x) z – y z + y –y =(–2x) [– y(z – y) – y (z+y) abc
y –y x + y c2 a c2b c + c3

= – 2x [–yz + y2 – y2 – yz] = }4xyz| = 8 given |xyz| = 2 R1 → aR1, R2 → bR2, R3 → cR3

For → 2 → (2, 1, 1) (–2, 1, –1) (2, – 1, – 1) 1 + a2 a2 a2


abc
3! 3! D= b2 1 + b2 b2
⇒ + 3! + = 12 abc
2! 2! c2 c2 1 + c2
For → –2→ (2, 1,–1), (–2,–1,–1), (–2,1,1) = 12
C1 → C1 – C3, C2 → C2 – C3
3 + 6 + 3 = 12
1 0 a2
Total solution = 12 + 12 = 24
0 1 b2 = 1[1 + c2 + b2] + a2 [+1]
1 + sin2 x cos2 x 4 sin2x –1 –1 1 + c2
Sol 7: (C) f(x) = sin2 x 1 + cos2 x 4 sin2x
2 2 = 1 + a2 + b2 + c2
sin x cos x 1 + 4 sin2x

R1 → R1 – R3, R2 → R2 – R3 Sol 10: (A) α + β + γ = π

sin(α + β + γ ) sin β cos γ


1 0 –1
sin β 0 tan α
f(x) = 0 1 –1
cos(α + β) tan α 0
sin2 x cos2 x 1 + 4 sin2x
sin π = 0
= 1 + 4 sin 2x + cos2x – 1(– sin2x)
α + β = π – γ, cos (π – γ) = – cos γ
1 7 . 7 2 | Determinants

0 sin β cos γ For infinite solution D= 0, Dx = Dy = Dz = 0


– sin β 0 tan α P + 4µ – 3pµ + 2 = 0
– cos α – tan α 0 4 3 3
  sin α     sin α   Dx = 3 p 2 = 4[p–8] + 2[6–3] + 3 [12 – 3p]
= sin β  –   cos γ  + cos γ sin β  
  cos α     cos α   3 4 1

sin β sin α cos γ = 4p – 32 + 6 + 36 – 9p = 0


=
cos γ → p = 10/5 = 2

a11 a12 a13 1 4 3


Sol 11: (C) a21 a22 a23 , aij ∈ {0, 1} Dy = 1 3 2 = 1[3–6] + 4[2µ–1] +3[3–3m]=0
a31 a32 a33 µ 3 1

– 3 + 8µ – 4 + 9 – 9µ = 0
1 0 1
2=µ
0 1 1 ⇒ 1 [– 1] + 1 [– 1] = – 2
1 1 0 For equation (i) p + 4(2) – 3p (2) + 2 = 0
P + 8 – 6p + 2 = 0
0 1 1
1 0 1 = – 1 [– 1] + 1 [1] = 2 → p = 10/5 = 2
1 1 0 1 2 4
1 1 1 1 1 1 Dz = 1 p 3
1 1 1 = 0, 0 1 1 µ 4 3
1 1 1 1 1 0 = 3p – 12 + 2(3µ – 3) + 4 (4 – pµ)
= 1 [–1] – 1[–1] + 1 [– 1] = 3p – 12 + 6µ – 6 + 16 – 4pµ
Cannot be → 3 = 3p + 6µ – 4pµ – 2
D = a11 a22 a33 + a12 a23 a31 + a21 a32 a13 – a31 a22 a13 – a32 At p = 2, µ = 2
a23 a11 – a21 a12 a33 < 3 ⇒ 3(2) + 6(2) – 4(2) (2) – 2
As, for it to be 3, atleast one terms must be 0 but there ⇒ 6 + 12 – 16 – 2 = 0
sum would not be 3
At p = 2, µ = 2, system has infinite solutions.

Sol 12: (D) Order 3 × 3


Sol 14: (B) ax – by = 2a – b
First column consists of sum of 2 terms
(c + 1)x + cy = 10 – a + 3b
2nd ‘’3’’
For infinitely many solution
3rd”4”
Total no. of determinats = 2.3.4 = 24 a –b
D= = 0 ac + b (c + 1) = 0
c +1 c
Sol 13: (D) x + 2y + 3z = 4 ac + bc + b = 0 …(i)
x + py + 2z = 3
2a – b –b
mx + 4y + z = 3 Dx = 0
10 – a + 3b c
1 2 3 4
  = c(2a – b) + b (10 – a + 3b) = 0
D= 1 p 2 ,C= 3 
µ 4 1 3  2ac – bc + 10b – ba + 3b2 = 0 … (ii)
 
a 2a – b
D = p – 8 + 2 [2µ 4 – 1] + 3 [4 – pµ) Dy = 0 =
c + 1 10 – a + 3b
D = p – 8 + 4µ – 2 + 12 – 3pµ = p + 4µ – 3pµ + 2
M a them a ti cs | 17.73

⇒ a(10 – a + 3b) + (2a – b) (–1 – c) = 0 And a, b, c ≠ 1 , So


⇒ 10a – a2 + 3ba – 2a – 2ac + b + bc = 0 1
–1 0
At x = 1, y = 3 1–a
1
a – 3b = 2a – b 0 –1 =0
1–b
o = a + 2b ⇒ a = – 2b c
1 1
c + 1 + 3c = 10 – a + 3b( – a = 2b) 1–c
4c = 9 + 2b + 3b = 9 + 5b  –c 1  1
⇒ – 1 –  + 1 = 0
4c = 9 + 5b  1 – c 1 – b  1 –a 
In equation (i) ac + bc + b = 0
1 c +1 –1 1
(9 + 5b) (9 + 5b) ⇒ + + =0
(– 2b) +b +b=0 1–a 1–c 1–b
4 4
– 18b – 10b2 + 9b + 5b2 + 4b = 0 1 1 1 (1 – c)
⇒ + + – =0
1–a 1–b 1–c (1 – c)
– 5b2 – 5b = 0
1 1 1
b2 + b = 0 ⇒ + + =1
1–a 1–b 1–c
b = – 1 or o
a = 2 or 0 cos(θ + φ) – sin(θ + φ) cos2 φ
c = 1 or 9/4 Sol 16: (B) sin θ cos θ sin φ
(a, b, c) → exactly ⇒ (–1, 2, 1) or (0, 0, 9/4) – cos θ sin θ cos φ

⇒ cos (θ + φ) [cos θ cos φ – sin θ sin f] +


Sol 15: (C) ax + y + z = 0
sin (θ + φ) [ sin θ – cos f] + sin φ cos q]
x + by + z = 0
+ cos2φ (sin2θ + cos2θ)
x + y + cz = 0 a, b, c ≠ 1
⇒ cos2 (θ + θ) + sin2 (θ + φ) + cos 2φ
a 1 1 0
= 1 + cos 2φ
D= 1 b 1,C= 0
1 1 c 0 So determinant is only dependent of φ
So, Dx = Dy = Dz = 0
Sol 17: (D) x sin θ – y cos θ + (λ + 1)z = 0
But system has nontrival solution
x cos θ + y sin θ – λ . z = 0
So, D = 0 and a, b, c ≠ 1
λ x + (λ + 1)y + z cos θ = 0
a 1 1
D= 1 b 1 sin θ – cos θ λ + 1
1 1 c D = cos θ sin θ –λ
λ λ + 1 cos θ
C1 → C1 – C3, C2 → C2 – C3
D = sin θ [sin θ cos θ + l2 + l]
a–1 0 1
D= 0 b –1 1 + cos (cos2θ + l2] + (λ + 1) (λ cos θ + cos θ – λ sin θ)
1–c 1–c c D = (sin2θ + cos2θ) cos θ + sin θ (l2 + λ
1 – l2 – λ) + cosθ (l2 + l2 + λ + λ + 1)
–1 0
1–a D = cos θ (2l2 + 2λ + 2) = cos θ [l2 + 1 + (λ + 1)2]
1
= (1 – a) (1 – b) (1 – c) 0 –1 =0 So for D = 0 ( System has infinite solution)
1–b
c cos θ = 0, θ ∈ (2n + 1) π/2, λ ∈ R, n ∈ I
1 1
1–c
1 7 . 7 4 | Determinants

Sol 18: (A) a2x – ay = 1 – a


(n + 3)(n + 2)
–(n + 2) n+3
1 – a 2
bx + (3 – 2b)y = 3 + a C =  
3 + a –1 n+3 1 =–1
Unique solution x = 1, y = 1 0 1 0

a2 –a
D= , at (x, y) ⇒ (1, 1)
b 3 – 2b Sol 20: (A) lx – y + cos θ z = 0
a2 – a = 1 – a 3x + y + 2z = 0
a2 = 1 ⇒ a = ± 1 cos x + y + 2z = 0
And b + 3 – 2b = 3 + a 0 ≤ θ ≤ 2π
3–b=3+a 0 
 
a=–b C = 0 
0 
So (a, b) ⇒ (1, – 1) or (–1, 1)  

At (–1, 1) ⇒ x + y = 1 – (–1) = 2 So Dx = Dy = Dz = 0

x+y=2 λ –1 cos θ
And x + (3 – 2)y = 3 – 1 = 2 D= 3 1 2
cos θ 1 2
x+y=2
Both equations are same so, D =0 at (–1, 1) For non-trivial solution
So it is not unique solution D = 0 ∴Dx = Dy = Dz = 0
(a, b) ≠ (–1, 1) (a, b) = (1, – 1) l[2–2]+1[6 – 2 cos q]+cos θ [3 – cos q] = 0
⇒ 6 – 2 cos θ + 3 cos θ – cos2θ = 0
n+ 2 n+ 3 n+ 4
Cn Cn+1 Cn+ 2 ⇒ cos2θ – cos2θ – 6 = 0
n+ 3 n+ 4 n+5
Sol 19: (A) D = Cn+1 Cn+ 2 Cn+3
⇒ cos2θ – 3cos θ + 2 cos θ – 6 = 0
(n+ 4) n+5 n+ 6
Cn+ 2 Cn+3 Cn+ 6
⇒ cos θ (cos θ – 3) + 2 (cos θ – 3) = 0
⇒ (cos θ – 3) (cos θ + 2) = 0
(1 + n)(n + 2) (n + 3)(n + 2) (n + 4)(n + 3)
2 2 2 ⇒ cos θ = 3 or cos θ = – 2
(n + 3)(n + 2) (n + 4)(n + 3) (n + 5)(n + 4) But – 1 ≤ cos θ ≤ 1
D=
2 2 3
(n + 4)(n + 3) (n + 5)(n + 4) (n + 6)(n + 5) So cos θ ≠ 3, – 2
2 2 2 There is no solution for non-trivial solution

At n = 1
Multiple Correct Choice Type
3 6 10
D = 6 10 15 cos(x – y) cos(y – z) cos(z – x)
10 15 21 Sol 21: (A, D) cos(x + y) cos(y + z) cos(z + x)
sin(x + y) sin(y + z) sin(z + x)
= 3 [210 – 225) + 6 (150 – 126) +10(90–100)
= – 45 + 144 – 100 = – 1 cos (A – B) = cos A cos B + sin A sin B

There is only one option (A) sin (A + B) = cos B sin A + sin B cos A

Which satisfied the ans. cos (A + B) = cos A cos B – sin A sin B

Using, C1 → C1 – C2, C3 → C3 – C2, = cos (x – y) [cos (y + z) sin (z + x) – cos (z + x)


sin (y + z)] + cos (y – z) [cos (z + x) sin (x + y) – sin (x + z)
R3 → R3 – R2, we get
M a them a ti cs | 17.75

cos (x + y)] + cos (z – x) [cos (x + y) sin (y + z) – cos (y ⇒ x2ab (b – 1) + a2x (1 – b2) + a3 (b – 1) = 0


+ z) cos (x + y)]
⇒ x2ab – a2x (1 + b) + a3 (+1) = 0
= cos (x – y) [sin (z + x – y – z)]+ cos (y – z) [ sin (x +
⇒ x2ab – x (a2 + a2b) + a3 = 0
y – z – x)]+ cos (z – x) [sin (y + z – x – y)]
⇒ x2ab – a2x – a2 (bx – a) = 0
= cos (x – y) sin (x – y) + cos (y – z) sin (y – z) + cos
(z – x) sin (z – x) ⇒ (bx – a) (ax – a2) = 0

1 ⇒ bx – a = 0 or ax – a2 = 0
= [sin 2 (x –y) + sin (2(y – z) + sin 2(z – x)]
2 a a2
⇒x= or x = =a
1 b a
= [2 cos (x + z – zy) sin (x + z) – 2 sin (x – z) cos (x – z)]
2
a b aα + b
1
= sin (x – z) [cos (x + z – 2y) – cos (x – z)] Sol 24: (B, D) b c bα + c
2
aα + b bα + c 0
 (x – y) (y – z) 
= sin (x – z) 2sin sin 
 2 2  R3 → R3 – aR1 – R2
a b aα + b
= 2 sin (x – y) sin (y – z) sin (z – x)
b c bα + c
–π π 0 0 –(aα2 + bα + bα + c)
Sol 22: (A, B, C, D) < θ < , 0 ≤ A ≤ π/2
4 2
= (aa2 + 2bα + c) (b2 – ac) = 0
1 + sin2 A cos2 A 2sin 4θ
sin2 A 1 + cos2 A 2sin 4θ =0 So (b2 – ac) = 0

sin2 A cos2 A 1 + 2sin 4θ b2 = ac → b is GM of a, c → ab, c are in GP


or (aa2 + 2b α + c) = 0
R1 → R1 – R3 , R2 → R2 – R3
⇒ x = α → (x – α)
1 0 –1 ⇒ ax2 + 2bx + c, (x – α) is a factor of this)
0 1 –1 =0
2 2 2
sin A cos A 1 + sin θ Sol 25: (B, D) x – y + 3z = 2
2x – y + z = 4
⇒ 1 + 2 sin4θ + cos2A – 1 [– sin2 A]
x – 2y + az = 3
⇒ 1 + 2 sin 4θ + sin2θ + cos2 θ
1 –1 3
⇒ 2 + 2 sin 4θ = 0 (only depend on θ)
D = 2 –1 1
⇒ sin 4θ = – 1 ⇒ 4q∈ – π/2 + 2nπ, n ∈ I 1 –2 α
θ ∈ – π/8 + nπ/2, n ∈ I = 1 [– α + 2 ] – 1 [1 – 2a] + 3 [– 4 + 1]
(A) θ → – π/8 = – α + 2 – 1 + 2α – 9= α – 8
(B) θ → 3π/8 sin 4θ = – 1 D≠0→α≠8
(C) θ = – π/8 sin 4θ = – 1 If α = 8, D = 0
(D) θ = 3π/8 2 –1 3
Dx = 4 –1 1
1 a a2
3 –2 α
Sol 23: (A, D) 1 x x2 = 0
= 2 [– α + 2] – 1 [3 – 4a] + 3 [– 8 + 3]
b2 ab a2
= – 2α + 4 – 3 + 4α – 15 = 2α – 14 = 2(α – 7)
⇒ xa2 – x2ab + a [x2b2 – a2] + a2 [ab – xb2]=0
= At α = 8 (0 = 0), Dx ≠ 0
⇒ x [a – a b ] + x [ab – ab] – a + a b = 0
2 2 2 2 2 3 3
So, at α = 8, system has no solution.
1 7 . 7 6 | Determinants

1 bc bc(b + c) logx xyz logx y logx z


Sol 26: (A,B,C,D) 1 ca ca(c + a) (D) logy xyz 1 logx z
1 ab ab(a + b) logx xyz logz y 1

a abc abc(b + c) a 1 b+c logxyz log y logx


1 (abc)2
= b abc abc(c + a) = b 1 c+a logx logx logx
abc abc
c abc abc(a + b) c 1 a+b logxyz log y logz
log y log y log y
C3 → C3 + C1 logxyz log y logz
a 1 a+b+c a 1 1 logz logz logz
= (abc) b 1 a + b + c = (abc) (a + b + c) b 1 1
1 1 1
c 1 a+b+c c 1 1
=
(logxyz )(log y )(logz ) 1 1 1 =0
C2 → c2 – c3
(logx )(log y )(logz ) 1 1 1
a 0 1 ( C1 = c2 = c3)
= (a b c) (a + b + c) b 0 1 = 0
c 0 1 Sol 27: (A, B, C, D) a2x – by = a2 – b2posses an infinite
no. of solution
1 1 1 1 
1 ab + c abc c  +  bx – b2y = 2 + 4b
a b a b
1 1 1 1 1 a2 –b
(B) 1 bc + = a abc a  +  So D = 0 ⇒ = – b2a2 + b2 = 0 Þ
b c abc b c b –b2
1 1 1 1
1 ca + b abc b  +  ⇒ b2(1 – a2) = 0
c a
c a
→ b = 0 or a = ± 1 … (i)
c c c
c +
c a b a2 – b –b
Dx = 0 ⇒
abc a a a 2 + 4b –b2
a + C → c3 + c2
abc a b c 3
b b b = – b2 (a2 – b) + b (2 + 4b) = 0
b +
b a c – a2b2 + b3 + 2b + 4b2 = 0

1 1 1 b(b2 + 4b + 2 – a2b) = 0
c 1 c + + 
a b c c 1 C
⇒ b = 0 or b2 + 4b + 2 = a2b … (ii)
1 1 1 1 1 1
a 1 a + +  =  + +  a 1 a a2 a2 – b
a b c a b c b 1 b Dx = 0 ⇒ = a2(2 + 4b) + b(b – a2) = 0
b 2 + 4b
1 1 1
b 1 b + +  2a2 + 4a2b + b2 – ba2 = 0 …(iii)
a b c
All option are satisfied equation (i, ii, iii)
C1 → c1 – c3

0 1 c Sol 28: (A, C) p, q, r, s are in AP


1 1 1
=  + +  0 1 a =0 P = p q = p + d r = p + 2d, s = p + 3d
a b c 0 1 b
D is common difference of the A.P.
0 a–b a–c 0 (a – b) (a – c) p + sinx q + sinx p – r + sinx
(C) b – a 0 b – c = –(a – b) 0 (b – c) f(x) = q + sinx r + sinx –1 + sinx
c–a c–b 0 –(a – c) –(b – c) 0 r + sinx s + sinx s – q + sinx
This is stew symmetric matrix so value of determinate C2 → c2 – c1, C3 → c3 – c1
is zero.
M a them a ti cs | 17.77

p + sinx q – p –p + q – r Dx 3k – 2
x= =
D 1
f(x) = q + sinx r – q –1 – q
r + sinx s – r s–q–r 1 3
Dy = = 8 – 3 (1 + k) = 5 – 3k
1+k 8
p + sinx d –p – 2d
= q + d + sinx d –1 – p – d y = 5 – 3k
p + 2d + sinx d –p Which will satisfied other equation of (x, y)

R3 → R3 – R1, R2 → R2 – R1 ⇒ (3k – 2) – (1 + k) (5 – 3k) + 2 + k = 0


⇒ 3k – 2 – (5 – 3k2 + 2k) + 2 + k = 0
p + sinx d –p – 2d
f(x) = d 0 –1 – p – d + p + 2d ⇒ 3k – 2 – 5 + 3k2 – 2k + 2 + k = 0
2d 0 –p + p + 2d ⇒ 3k2 + 2k – 5 = 0
⇒ 3k2 + 5k – 3k – 5 = 0
p + sinx d –p – 2d
= k (3k + 5) – 1(3k + 5) = 0
d 0 d–1
2d 0 2d ⇒ (k – 1) (3k + 5) = 0
⇒ k – 1 = 0 or 3k + 5 = 0
= d[2d(d – 1) – 2d2]
k =1 or k = –5/3
= d(2d2 – 2d – 2d2) = – 2d2
2 2 2
2
 –2d2  Sol 30: (A, B, C)
∫ f(x)dx = ∫ –2d dx = [ 
 d
x  = – 2d2 (2) = – 4
 0
0 0
1 1 (x + y)
⇒ d2 = 1 ⇒ d = ± 1 z z z2
–(y + z) 1 1
D=
Sol 29: (A, C) x + y = 3 … (i) x2 x x
(1 + k) x + (2 + k)y = 8 … (ii) –y(y + 2) –(x + 2y + z) –y(x + y)
x2 z xz xz 2
x – (1 + k)y = – (2 + k) … (iii)
1 1 1
1 1 D= . .
2 2 2 2
D12 = = 2 + k – (1 + k) = 1 z x x z
1+K 2+K
z z –(x + y)
D12 ≠ 0 , So
–(y + z) x x
1 1 –zy(y + z) –(x + 2y + z)(xz) –yx(x + y)
D13 = = – 1 – k – 1 = – 2 – k = – (2 + k)
1 –(1 + k) C1 → C2 – C3 and

3 1 –zy (y + z) + yx (x + y) = – zy2 – z2y + yx2 + y2x


So Dx = = – 3 (1 + k) + 2 + k
–(2 + k) –(1 + k) = y [y(x –z) + x2 – z2] = y(x – z) [y + x + z]

= – 3 – 3k + 2 + k = –2k – 1 1
D=
4 4
x z
1+k (2 + k)
∴ D23 = 0
1 –(1 + k) (x + y + z) (x + y + z) –(x + y)
= – 1(1 + k)2 – (2 + k) = – (k2 + 3k + 3) –(x + y + z) 0 x
y(x – z)(x + y + z) x[(y – z)(x + y + z) – 2yz] –xy(x + y)
From (ii) & (iii)
(x + y + z)
3 1 D=
Dx = = 3(2 + k) – 8 = 6 + 3k – 8 = 3k – 2 x4 z4
8 2+k
1 7 . 7 8 | Determinants

1 x+y+z –(x + y) variables and that could have only unique, no solution
or infinitely many solution.
–1 0 x
y(x – z) x[(y – z)(x + y + z) – 2yz] –xy(x + y) ∴ It is not possible to have two solutions.
Hence, number of matrices A is zero.
R1 → R1 + R2

(x + y + z) ax − by − c bx + ay cx + a
D=
4 4
x z Sol 3: Given bx + ay −ax + by − c cy + b 0
=
cx + a cy + b −ax − by + c
0 x+y+z –y
–1 0 x a2 x − aby − ac bx + ay cx + a
y(x – z) x[(y – z)(x + y + z)(–xyz) –xy(x + y) 1 2
⇒ abx + a y −ax + by − c cy + b 0
=
a 2
x+y+2 acx + a cy + b −ax − by + c
x +4 y + 2
x +4
x4 + 4 Applying C1 → C1 + bC2 + cC3
( x + y + z )( xy + y ) − xy ( x − z ) + yx ( −xyz )( x + y + z )( y − z ) 
( x + y + z )( xy + y ) − xy ( x − z ) + yx ( −xyz )( x + y + z )( y − z )  
 
(a2 + b2 + c2 )x ay + bx cx + a
1 2 2 2
⇒ (a + b + c )y by − c − ax b + cy = 0
Previous Years Questions a 2 2
a +b +c 2
b + cy c − ax − by

1 a a2 x ay + bx cx + a
1
Sol 1: (B) Let
= ∆ cos(p − d)x cospx cos(p + d)x ⇒ y by − c − ax b + cy = 0
a
sin(p − d)x sinpx sin(p + d)x 1 b + cy c − ax − by

Applying C1 → C1 + C3 ( a2 + b2 + c2 =
1)

Applying C2 → C2 − bC1 and C3 → C3 − cC1


1 + a2 a a2
=
⇒ ∆ cos(p − d)x + cos(p + d)x cospx cos(p + d)x x ay a
1
sin(p − d)x + sin(p + d)x sinpx sin(p + d)x ⇒ y −c − ax b =0
a
1 cy −ax − by
1 + a2 a a2
⇒ ∆ 2cospx cosdx cospx cos(p + d)x x2 axy ax
1
2sinpx cosdx sinpx sin(p + d)x ⇒ y −c − ax b =0
ax
1 cy −ax − by
Applying C1 → C1 − 2cosdx C2
Applying R1 → R1 + yR 2 + R 3
1 + a2 − 2acosdx a a2
⇒ ∆ 0 cospx cos(p + d)x x2 + y 2 + 1 0 0
1
0 sinpx sin(p + d)x ⇒ y −c − ax b =0
ax
1 cy −ax − by
⇒ ∆= (1 + a2 − 2acosdx)
1 2
[sin(p + d)x cospx − sinpx cos(p + d)x] ⇒ [(x + y 2 + 1){( −c − ax)( −ax − by) − b(cy)}] =0
ax
⇒ ∆= (1 + a2 − 2acos dx)sindx
1 2
Which is independent of p. ⇒ [(x + y 2 + 1)(acx + bcy + a2 x2 + abxy − bcy)] =
0
ax
1 2
 x  1  ⇒ [(x + y 2 + 1)(acx + a2 x2 + abxy)] =
0
    ax
Sol 2: Since,  y  = 0  is linear equation in three
A 1
 z  0  ⇒ [ax(x2 + y 2 + 1)(c + ax + by)] =
0
    ax
M a them a ti cs | 17.79

Applying R 2 → R 2 + R 3
⇒ (x2 + y 2 + 1)(ax + by + c) =
0 ⇒ ax + by + c =0
Which represents a straight line.
sin θ cos θ sin2θ
Sol 4: Since, the given system of equations posses non-  2π   2π   4π 
sin  θ +  cos  θ +  sin  2θ + 
trivial solution, if  3   3   3 
=
0 1 −2  2π   2π   4π 
+ sin  θ −  + cos  θ −  + sin  2θ − 
0 k=0
0 −3 1 =  3   3   3 
k −5 4  2π   2π   4π 
sin  θ −  cos  θ −  sin  2θ − 
 3   3   3 
On solving the equations x = y = z = λ (say)
∴ For k = 0, the system has infinite solutions for λ ∈ R .  2π   2π 
Now, sin  θ +  + sin  θ − 
 3   3 
Sol 5: Given system of equations are
 2π 2π   2π 2π 
3x + my = m and 2x – 5y = 20 θ+ +θ−  θ+ −θ+ 
= 2sin  3 3  cos  3 3 
3 m  2   2 
Here, ∆ = = −15 − 2m    
2 −5    
2π  π
m m 3 m = 2sin θ cos = 2sin θ cos  π − 
and ∆ x = =−25m ; ∆y = = 60 − 2m 3  3
20 −5 2 20
π =
− sin θ
=−2sin θ cos
If ∆ =0 , then system inconsistent i.e. it has no solution. 3
15  2π   2π 
If ∆ ≠ 0 i.e. m ≠ , then system has a unique solution and cos  θ +
2  + cos  θ − 
 3   3 
for any fixed value of m.
 2π 2π   2π 2π 
∆x −25m 25m θ+ +θ−  θ+ −θ+ 
x
We have,= = = 3 3 3 3 
∆ −15 − 2m 15 + 2m = 2cos   cos 
 2   2 
   
∆y 60 − 2m 2m − 60    
y
and= = =
∆ −15 − 2m 15 + 2m  2π   1
= 2cos θ cos  =  2cos θ  −  =
− cos θ
25m  3   2
For x > 0, >0
15 + 2m
 4π   4π 
15 and sin  2θ +  + sin  2θ − 
⇒ m > 0 or m < − … (i)  3   3 
2 
2m − 60  4π 4π   4π 4π 
and y > 0, >0  2θ + + 2θ −   2θ + − 2θ + 
2m + 15 = 2sin  3 3  cos  3 3 
 2   2 
15    
⇒ m > 30 or m < − … (ii)    
2 
4π  π
From equation (i) and (ii) we get = 2sin2θ cos= 2sin2θ cos  π + 
3  3
15 π
m<− or m > 30 =−2sin2θ cos = − sin2θ
2 3

sin θ cos θ sin2θ


sin θ cos θ sin2θ ∴∆
= − sin θ − cos θ − sin2θ
 2π   2π   4π   2π   2π   4π 
Sol 6: Let sin  θ +  cos  θ +  sin  2θ +  sin  θ −
 3   3   3   cos  θ −  sin  2θ − 
 3   3   3 
 2π   2π   4π 
sin  θ −  cos  θ −  sin  2θ − 
 3   3   3  = 0 (Since, R1 and R 2 are proportional).
1 7 . 8 0 | Determinants

Sol 7: (B) Method I


α 2 − 4 4 α 2 − 4 9α 2 − 4
5 3 5 3 5
Total no. of ways = 3 − C1 3 − 1 + C2 3 − 1 −2 −2 −2 = −648α

= 243 − 3 × 32 + 3= 246 − 96= 150 5 + 2α 5 + 4α 5 + 6α

Alternative Method α 2 − 4 4 α 2 − 4 9α 2 − 4
System I −2 1 1 1 648α
=
5 + 2α 5 + 4α 5 + 6α
Boxes I II III
Balls I 2 2 α2 − 4 3α2 8α 2
−2 1 0 0 = −648α
For this system no. of ways 5 + 2α 2α 4α
 5!
= 
 2!2!1!
1  5× 4 ×3× 2 
× ×
2!   2×2×2 
= × 6 90 (
−2 12α3 − 16α3 =−648α)
System II ( ) 3
⇒ 2 −4α =−648α

⇒ α ( α − 81 ) =0
2
Boxes I II III
⇒=
α 0,9, −9
Balls I 3 1

For this system no. of ways 1 1 1 + x3


Sol 9: (2) 2 4 1 + 8x3 =
10
 5! 1
=  ×  × 3! = 10 × 6 = 60 3 9 1 + 27x3
 3!1!1! 2! 
1 1 1 1 1 1
Total no. of ways = 90 + 60 = 150 3 6
x 2 4 1 +x 2 4 8 = 10
3 9 1 3 9 27
Sol 8: (B, C)

(1 + α ) (1 + 2α ) (1 + 3α )
2 2 2
1 0 0 1 0 0
3 6
x 2 2 −1 + x 2 2 6 = 10
( 2 + α ) ( 2 + 2α ) ( 2 + 3α )
2 2 2
−648α
=
3 6 −2 3 6 24
(3 + α ) (3 + 2α ) (3 + 3α )
2 2 2

6x3 + x3 − 5 = 0 ⇒ 6x6 + 6x3 − 5x3 − 5 = 0


1 + α2 + 2α 1 + 4α2 + 4α 1 + 9α 2 + 6α
4 + α2 + 4α 4 + 4 α 2 + 8α 4 + 9α2 + 12 R 2 → R 2 − R1
(6x − 5)( x
3 3
)
+1 =
0

9 + α2 + 6α 9 + 4α2 + 12α 9 + 9α2 + 18α 5


x3 = or x3 = −1 Two real distinct values of x.
6
R3 → R3 − R 2 =
−648α

1 + α2 + 2α 1 + 4α2 + 4α 1 + 9α2 + 6α
3 + 2α 3 + 4α 3 + 6α C2 → C2 − C3
5 + 2α 5 + 4α 5 + 6α
C2 → C2 − C2 =
−648α

You might also like